board vitals part II

Ace your homework & exams now with Quizwiz!

Bone marrow signal changes on MRI in osteomyelitis are characterized by: (SELECT 2 ANSWERS) A. Low signal on T1 B. High signal on T2 and STIR C. Normal T1 signal D. Low T2 signal

B - High signal on T2 and STIR,A - Low signal on T1. Correct answer: (A) (B) Low signal on T1 and high signal on T2 and STIR. Explanation: The bone marrow signal changes on MRI in osteomyelitis are low signal on T1 and high signal on T2 and STIR. This differentiates it from chronic neuroarthropathy, which is characterized by normal or low T1 and T2 signals. Acute neuroarthropathy is characterized by the same changes as in osteomyelitis.

Treatment of a IIIB open fracture: Can be flushed with saline in the ER, and scheduled for the OR within 48 hours. Should be treated with antibiotics based upon index ER cultures. Should not use penicillin G because of the high incidence of penicillinase-resistant bacteria. Should utilize antibiotics for 10-14 days. Should be considered for primary amputation if the MESS score is greater than seven.

Can be flushed with saline in the ER, and scheduled for the OR within 48 hours. Should be treated with antibiotics based upon index ER cultures. Should not use penicillin G because of the high incidence of penicillinase-resistant bacteria. Should utilize antibiotics for 10-14 days. Should be considered for primary amputation if the MESS score is greater than seven.

Which of the following kinds of suture material is not considered absorbable? A. PDS B. Polypropylene C. Polyglycolic acid D. Chromic gut

Correct Answer: B. Polypropylene. Explanation: Absorbable sutures are frequently used in foot and ankle surgery to approximate deep and subcutaneous tissue. Absorbable sutures lose the majority of their strength within 60 days following implantation. Non-absorbable sutures are used to re-approximate skin edges and in other locations where more durable strength is required. Non-absorbable sutures are made out of nylon, polyester, and other materials that are not degraded by the body. Polypropylene is also a non-absorbable suture material. PDS and polyglycolic acid are synthetic absorbable suture and chromic gut is a natural absorbable suture. The correct answer is B, since polypropylene is not consider absorbable.

Which of the following pressures is considered positive for acute compartment syndrome and is an indication for an emergent fasciotomy? A. 30 mmHg B. 10 mmHg C. 8 mmHg D. 25 mmHg

Correct answer: (A) 30 mmHg. Explanation: Compartment syndrome is due to increased pressure in a closed fascial space compromising the circulation to the nerves and muscles within the compartment. This can be caused by fractures, severe contusion, and drug overdose with limb compression, a burn, and vigorous exercise. Intracompartmental pressures are measured using a Wick catheter. The range of normal pressure is from zero to 8 millimeters of Mercury. A pressure of 30 mmHg is an indication for decompressive fasciotomy. (B) (C) (D) are all within limits of negative compartment pressure.

Which of the following is not an appropriate technique for popliteal blocks? A. Anterior B. Lateral C. Posterior D. All of the above are popliteal block techniques.

Correct answer: (A) Anterior. Explanation: Popliteal blocks are a common and effective means of anesthesia for many foot and ankle surgeries. They can provide long-lasting pain relief and are usually well-received by patients. Popliteal blocks are frequently performed in the pre-operative holding area. They can be performed using various devices to direct the injection and isolate the sciatic nerve in the popliteal fossa. Neurostimulation or ultrasound-guided blocks are the most commonly used nerve identification techniques. The nerve can be isolated with the patient in either the prone position or the supine position. When the patient is in the prone position, the nerve is accessed through a posterior approach and when the patient is supine, the nerve is located through a lateral approach. The nerve cannot be located through an anterior approach as that would require direct penetration through the tibia.

Which of the following is true of axillary crutches? A. Elbows should be flexed at 20-30 degrees when using B. Distance from the top of crutch to the axilla is 9 cm C. Hand grip should be at the level of the elbow D. Tip of crutch should be 10 inches from the front of the foot

Correct answer: (A) Elbows should be flexed at 20-30 degrees when using. Explanation: Axillary crutches are one of the most common assistive devices used after surgery. When utilizing these, it is important patient's get proper training. Elbows should be flexed 20-30 degrees when using. Elbows should NOT be fully extended. Distance from the top of crutch to axilla should be 2-3 finger widths (around 6 cm). The hand grip should either be at the level of the ulnar styloid process or at the level of the greater trochanter. The tip of the crutch should be 6 inches from the foot and not the ten inches like the incorrect answer choice in this question states.

Stance phase of gait is the weightbearing portion of each gait cycle. What initiates and ends the stance phase of gait? A. Heel contact and toe off of the same foot B. Heel contact and toe off of the contralateral foot C. Toe off of the support limb and heel contact of the same foot D. Toe off of the support limb and heel contact of the opposite limb

Correct answer: (A) Heel contact and toe off of the same foot. Explanation: Stance phase is initiated with the heel contact and ends with toe off of the same limb/foot. This period of time when the foot is in contact with the ground is divided into four functional phases, further subdividing the traditional three subphases of stance, the contact phase, midstance and heel off.

What is an advantage that the Chopart's amputation have over the Syme's amputation? A. Short distances can be walked without a prothesis B. Less effected by equinus C. Doesn't require and intact heel fat pad D. Less soft tissue complications

Correct answer: (A) Short distances can be walked without a prosthesis. Explanation: The Chopart amputation leaves a short stump whereas the the Symes amputation does not. The stump is just long enough to allow ambulation for short distances, such as the bathroom, without a prothesis. (B) As the lever arm of the Chopart amputation is greater than that of the Symes's the effect of equinus on the stump is greater as well. (C) The Chopart amputation does require an intact heel fat pad. (D) There is no difference in soft tissue complications with the Chopart versus the Symes amputation.

If a patient develops a new neurologic problem prior to a scheduled elective surgery, what will be the necessary recommended course of action? A. Surgery should be rescheduled B. Surgery should proceed C. Referral after surgery D. Have a neurologist in the OR during surgery

Correct answer: (A) Surgery should be rescheduled. Explanation: The purpose of the preoperative assessment is to identify modifiable risk factors and to optimize medically specific issues before surgery. Typically, if any patient develops a new neurologic problem or an exacerbation of an existing one, elective surgery should be rescheduled until the issue can be managed. (B) A new neurologic issue should call for a cancellation of the surgery. (C) Referral should be made to a neurologist, and communication with the surgeon and anesthesiologist to that effect should be made. (D) Surgery should be rescheduled and a neurologist should consulted.

What is the governing principles allows for gradual distraction of osseous and soft tissues? A. Tension-stress effect B. Lengthening effect C. Rigid immobilization D. Atraumatic technique

Correct answer: (A) Tension-stress effect. Explanation: The tension-stress effect, also known as the law of tension-stress, is the stimulation and active growth of osseous and soft tissue structures by gradual traction. The tension-stress effect, first described by Ilizarov, is the governing principle that allows for the gradual distraction of osseous and soft tissues to achieve lengthening of the skeletal system. Ilizarov discovered that bone could be lengthened through callous distraction. The tension-stress shows that if limb lengthening is performed correctly and is distracted at a proper rate, both osseous tissues and soft tissues will proliferate in the area of distraction.

Principles of tendon transfers include (SELECT ALL THAT APPLY): A. A transferred tendon does not lose strength/power upon transfer B. Arthrodesis procedures may still require tendon transfers C. In-phase transfers are preferred over out-of-phase transfers D. The peroneus longus-to-brevis transfer is indicated with a rigid plantarflexed first ray, with a rigid cavus deformity

C - In-phase transfers are preferred over out-of-phase transfers,B - Arthrodesis procedures may still require tendon transfers. Correct: B and C Explanation: A transferred tendon loses one grade of power; therefore, only a 4/5 or 5/5 tendon should be considered for a dynamic transfer. When performing arthrodesis procedures, a plantigrade foot may be accomplished initially. However, if deforming tendons remain, an example being a strong tibialis posterior tendon causing equinovarus post-operatively, appropriate tendon transfers should be considered. Although out-of-phase transfers can be considered, in-phase transfers require less post-operative rehabilitation. In many forms of hereditary sensory motor neuropathies, the peroneus longus is a strong deforming force. The longus-to-brevis transfer works best with a flexible, not rigid, plantarflexed first ray, in a flexible deformity in a younger patient population.

Clawing of the toes in polio is secondary to the long extensors being recruited to assist in ankle dorsiflexion. Tendon transfers that can be utilized to treat this clawing is (are): (Select all that apply) A. Transfer of the EDL to the midfoot. B. Transfer of the EHL to the first metatarsal base. C. Transfer of the FDL to the proximal phalanx. D. Transfer of the EHL to the first metahead.

Correct answer: (A) (B) (C) (D) Explanation: All of the above are options to treat digital clawing. The Hibbs type tenosuspension transfers the 2-5 EDL tendons to either the third metatarsal base or third cuneiform. Hansen transfers the 4th and 5th EDL tendons to the peroneus tertius and does not re-route or attach the second and third EDL tendons. Transferring the EHL to the first metatarsal base allows the EHL to act as an ankle dorsiflexor, and remove the deforming forces at the MPJ. This procedure effectively replaces the Jones teno-suspension (choice D above). An EHL re-attachment close to the tibialis anterior insertion most closely approximates normal ankle dorsiflexion. Transferring the FDL to the proximal phalanx effectively changes the deforming force of the FDL to one of stabilization and plantarflexion of the proximal phalanx at the level of the MPJ. The FDL is split along its natural raphe, and the split tendon is brought up each side of the phalanx and secured into the extensor hood. Hansen has a modification where, in addition to the longus, he also brings the brevis tendon up each side of the phalanx also. The split longus transfer is the Girdlestone-Taylor procedure. Finally, transfer of the EHL to the first metahead is the Jones procedure. The EHL can be attempting to aid in ankle dorsiflexion, but by doing so, can cause a hallux hammertoe. By transferring the tendon to the first metahead, and fusing the hallux IPJ, the hammering is resolved and the EHL becomes a more effective dorsiflexor of the ankle.

Which of the following statements correctly describe the Ponseti technique for clubfoot repair? (Select all that apply) The fist manipulation focuses on supinating the forefoot by dorsiflexing the first metatarsal head. The Achilles is the first step in the correction. The endpoint for the weekly manipulations is when the foot is abducted approximately 70°. The Achilles tenotomy is performed under a pre-incisional local anesthetic rather than a general anesthetic.

Correct answer: (A) (C) Explanation: The first manipulation does indeed focus on supinating the forefoot by dorsiflexing the first metatarsal head. This is the key component to the technique because if it is not performed correctly it will result in a stiff nonreducible foot. The endpoint for the weekly manipulations is when the foot is abducted approximately 70°. This usually will occur after three or four weekly manipulations. (B) The Achilles tenotomy is the final step in the Ponseti technique. (D) The Achilles tenotomy is not performed under a pre-incisional block as it would obscure the margins of the tendon from palpation and make the procedure difficult. Rather a topical anesthetic is initially used then after the procedure the area is infiltrated with local anesthetic.

Preoperative radiographic indications for a Lapidus fusion include: (Select all that apply) A. Increased cortical thickening of the second metatarsal shaft. B. -6mm metatarsal protrusion distance. C. First cuneiform-second metatarsal divergence. D. Divergence of the dorsal cortices of the first and second metatarsals.

Correct answer: (A) (C) (D) Increased cortical thickening of the second metatarsal shaft, first cuneiform-second metatarsal divergence and divergence of the dorsal cortices of the first and second metatarsals. Explanation: Increased cortical thickening of the second, sometimes third, metatarsal shafts is indicative of some degree of first ray hypermobility, as those two metatarsals endure more load. Increased load can also predispose those metatarsals to fracture. The normal metatarsal protrusion distance is +/- 2mm, with a positive indicating a longer first metatarsal; a negative indicating a longer second metatarsal. A -6mm MPD indicates a much longer second metatarsal (or shorter first). If the surgeon continues with the Lapidus as planned, bone graft would have to be considered to restore length. Know that for every 3mm of shortening of the first ray, there is 1mm of elevation. First cuneiform-second metatarsal divergence is indicative of hypermobility. To look at it another way, radiographically it appears the first metatarsal-cuneiform has separated from the second metatarsal-cuneiform; the "cuneiform split." Observe the dorsal cortex of the first metatarsal to that of the second metatarsal; a dorsal cortex of the first higher than the second may indicate mobility. As a final note regarding hypermobility of the first ray; thinking is evolving regarding what causes what. Does first ray hypermobility cause HAV, or vice versa? Discussion is beyond the scope of this question.

During contact phase in gait, what is the position observed with the tibia, and what is the motion observed with the tibia? A. 2 degrees internal, and 9 degrees internal rotation B. 4-5 degrees internal and 4-5 degrees internal rotation C. 0-2 degrees in external and 4-5 internal rotation D. 0 degrees in external and 4-5 degrees on internal rotation

Correct answer: (A) 2 degrees internal, and 9 degrees internal rotation.

At what age do talonavicular coalitions typically ossify? A. 3-5 years old B. 8-12 years old C. 12-16 years old D. 17-21 years old

Correct answer: (A) 3-5 years old. Explanation: Talonavicular coalitions ossify between the ages of 3 and 5 years old. Calcaneonavicular coalitions ossify between the ages of 8-12. Talocalcaneal coalitions ossify between the ages of 12 and 16. The times typically correlate with the onset of symptoms. The calcaneonavicular coalition is best seen on a oblique view of the foot, but may also be visualized on a lateral film. The talocalcaneal coalition is best viewed on a Harris projection of the calcaneus on plain films. Talonavicular coalitions are readily identified on AP views.

Base of gait is defined as the horizontal distance from one heel-strike to the next heel strike. What is the average base of gait? A. 3.5 in B. 2.5 in C. 4.5 in D. 1.5 in

Correct answer: (A) 3.5 in. Explanation: 3.5 in. Base of gait is defined as the horizontal distance from one heel-strike to the next heel-strike. The average base of gait is 3.5 in. in order for the lateral shift of the body to be properly accepted by the limb.

Which two muscles are antagonists in the gait cycle? A. Extensor digitorum longus and peroneus tertius B. Tibialis posterior and peroneus brevis C. Tibialis posterior and flexor digitourm longus D. Tibialis posterior and flexor hallucis longus

Correct answer: (B) Tibialis posterior and peroneus brevis. Explanation: The posterior tibial tendon is the strongest supinator of the foot in the gait cycle. The peroneus brevis is the strongest pronator of the foot in the gait cycle. They are antagonists and bring stability to the foot in the gait cycle. (A) The extensor digitorum longus is a bi-phasic muscle as is the peroneus tertius. They work together as agonists in the gait cycle. (C) The tibialis posterior and flexor digitorum longus are both stance phase muscles and are agonists. (D) The tibialis posterior and flexor hallucis longus muscles are both stance phase muscles and are agonists of each other.

There are two main radiographic angles that are used when look at Tailor's Bunions. The first is the intermetatarsal angle between metatarsals 4/5 and the other is the lateral bowing angle. Which of the following is the normal value of the IM angle between 4th and 5th metatarsals and the lateral bowing angle respectively? A. 6.4-9.1 degrees, 2.64 degrees B. 15 degrees, 21 degrees C. 2 degrees, 7.1 degrees D. 0 degrees, 0 degrees

Correct answer: (A) 6.4-9.1 degrees, 2.64 degrees. Explanation: The IM angle between the 4th and 5th metatarsals is measured as the angle between the bisection of the 4th metatarsal and then medial proximal aspect of the 5th metatarsal. The lateral bowing angle is measured as the angle between the medial proximal aspect of the 5th metatarsal and the bisection of the 5th metatarsal head and neck. Normal angle for IM is between 6.4-9.1 degrees and that of lateral bowing angle is approximately 2.64 degrees. It is important to realize in this question that we were not looking to get an exact answer to the nearest decimal point as there are clearly 3 incorrect answers with values that are way off from normal and then the correct answer.

Angle of gait is defined as the angle formed by the longitudinal axis of the foot and the line of progression. What is the average gait angle? A. 7 degrees toe-out per side B. An average of 18 degrees C. 18 degrees toe-out per side D. 3.5 degrees

Correct answer: (A) 7 degrees toe-out per side. Explanation: Angle of gait is defined as the angle formed by the longitudinal axis of the foot and the line of progression (sagittal plane). It is measured by a line bisecting the center of the heel and the first interspace with the sagittal plane. The average gait angle is 7 degrees toe-out per side, or an average of 12 to 15 degrees total. Torsional or positional changes in the hip, femur, and tibia have a major influence on this position.

What is the position of the head and neck of the talus relative to the body in clubfoot? A. Adducted and plantarflexed B. Abducted and dorsiflexed C. Adducted and dorsiflexed D. Abducted and plantarflexed

Correct answer: (A) Adducted and plantarflexed. Explanation: The foot is rotated medially beneath the talus; the head of the talus is palpable on the lateral aspect of the dorsum of the foot, owing to the inward and backward displacement of the navicular. The head of the talus is adducted and plantarflexed. The defect is in the head of the talus which is the most deformed bone in the clubfoot deformity.

Which of the following methods restores the primary restraint of the peroneal tendons in acute peroneal subluxation? A. Anatomic reattachment of the superior peroneal retinaculum B. Groove deepening of posterior fibula C. Fibular bone block D. Rerouting the peroneal tendons behind the calcaneofibular ligament

Correct answer: (A) Anatomic reattachment of the superior peroneal retinaculum. Explanation: Repair of the superior retinaculum will restore the primary restraint of the peroneal tendons to prevent subluxation. Many techniques are reported in literature. If retinaculum is available and of adequate integrity, a pants over vest suture technique may be used to restore the retinaculum. Several authors have reported drill holes within the distal fibula with reattachment of the retinaculum using sutures. (B) Groove deepening of the posterior fibula may be utilized when the groove is a flat or convex shape. However, recent literature suggests the shape of the groove may not be a significant predisposing factor to subluxation. A disadvantage of this technique is the deepening decreases the cartilaginous surface for gliding of the tendons. (C) An osteotomy of the fibula with fixation or a wedge shaped graft is used to create a physical barrier to prevent subluxation of the peroneal tendons. This has many reported complications and is not often used as a primary procedure. (D) Rerouting the peroneal tendons behind the calcaneofibular ligament may be performed if the superior peroneal retinaculum cannot be restored. The ligament serves as a constraint to the peroneal tendons and this procedure is utilized if the SPR cannot be repaired or utilized.

In a workup of a 14 year old male for left heel pain an MRI is obtained. Review the image below. What is the most likely diagnosis? A. Aneurysmal bone cyst B. Intraosseous lipoma C. Enchondroma D. Osteoid osteoma

Correct answer: (A) Aneurysmal bone cyst. Explanation: The aneurysmal bone cyst is typically a benign, expansile lytic lesion. MR imaging findings of ABC consist of an expansile, lobulated, multiseptated mass with varying signal intensity within numerous cavities. Fluid-fluid levels representing hemoglobin byproducts may be seen and are typical of ABC. A fluid-fluid level is the most striking feature of the lesion above. Although many tumors that undergo hemorrhage or cystic degeneration may cause fluid- fluid levels, the MRI findings are highly suggestive of ABC. Interosseous lipomas typically show a central calcification. Enchondromas typically occur in the phalanges. Osteoid osteomas are typically quite small (much smaller than the lesion above), and the pain worse at night is often relieved by aspirin or the NSAIDs. These lesions produce exuberant periosteal reactions.

Commonly physical therapists do mobilizations/manipulations of parts of the body that are causing the patient trouble. Which of the following is true? A. Ankle Open pack position is plantarflexion of ankle. B. Ankle Open pack position is dorsiflexion of the ankle. C. 1st MTPJ open pack position is plantarflexion of the MTPJ. D. 2nd MTPJ open pack position is dorsiflexion of the MTPJ.

Correct answer: (A) Ankle Open pack position is plantarflexion of ankle. Explanation: Mobilizations are always completed with the joint in the open pack position. Open pack position is described as the position with the least amount of contact between articular surfaces allowing for less resistance to motion and thus this is the position that mobilizations should be completed in. Closed pack position is when the joint surfaces are most lined up with the greatest amount of contact between them. The ankle open pack position is plantarflexion, the 1st MTPJ open pack position is dorsiflexion of the MTPJ and the 2nd MTPJ open pack position is plantarflexion of the MTPJ.

In preparation for the osteotomy for an Austin bunionectomy, the surgeon performs subperiosteal dissection of the dorsal and lateral aspect of the joint from the anatomical neck of the metatarsal to the shaft of the proximal phalanx. What potential complication's likelihood has been increased with this surgical technique? A. avascular necrosis B. nonunion C. malunion D. hallux varus

Correct answer: (A) Avascular necrosis. Explanation: With such aggressive soft tissue stripping around the first metatarsophalangeal joint, the probability of avascular necrosis significantly increases. Most of the vascular supply to the head of the first metatarsal originates from the periosteal and capsular tissues. When these tissue are stripped combined with the osteoeomty, located distal relative to the nutrient artery of the first metatarsal, the metatarsal head become void of any vascular supply. This sets the stage for an avascular process to occur. Nonunion, malunion and hallux virus while all being complications of distal bunioneictomies, there is not a correlation to extensive soft tissue stripping of the distal articular soft tissues.

In the dorsoplantar projection (plantar-dorsal view) the demonstrated structures are the phalanges, sesamoid bones, metatarsals, and tarsal bones. The patient is positioned standing on the X-ray cassette in the angle and base of gait. The tube angle is 15o from vertical. What anatomic structure is targeted by the X-ray beam? A. base of the second metatarsal B. base of the third metatarsal C. navicular D. lateral cuneiform

Correct answer: (A) Base of the second metatarsal. Explanation: The dorsoplantar or anterior posterior projections or plantar dorsal view is a very common projection used in podiatry. The phalanges, sesamoid bones, metatarsals and tarsal bones are well visualized. One-half of the film is either blocked with a led screen or collimated longitudinally. The patient states in the angle and base of gait. The X-ray tube is angled 15o cephalic or from vertical. The base of the second metatarsal is the anatomic target of the X-ray beam.

What is the closed kinetic chain pronation at the subtalar joint? A. Calcaneal eversion with talar adduction and plantarflexion B. Calcaneal inversion, with talar adduction and plantarflexion C. Calcaneal eversion, with talar dorsiflexion D. Calcaneal plantarflexion, inversion and adduction

Correct answer: (A) Calcaneal eversion with talar adduction and plantarflexion. Explanation: Closed kinetic chain pronation consists of calcaneal eversion with talar adduction and plantar flexion. This appears clinically as a foot facing inward, with a "stacking" of the talus over the calcaneus producing a "higher, thinner" appearance to the foot when viewed in the frontal plane. Conversely, closed kinetic chain pronation consists of calcaneal eversion along with talar adduction and plantarflexion. This appears clinically as a foot facing outward, with a "lowering" of the talus from the calcaneus producing a "shorter, wider" appearance of the foot when viewed in the frontal plane.

For a plantodorsal axial projection of the calcaneus, what angle should be used? A. Cephalic angle of 40 degrees to the long axis of the foot B. Caudal angle of 40 degrees to the long axis of the foot C. Caudal angle of 60 degrees to the long axis of the foot D. Cephalic angle of 60 degrees to the long axis of the foot

Correct answer: (A) Cephalic angle of 40 degrees to the long axis of the foot. Explanation: For a plantodorsal axial projection of the calcaneus, a cephalad angle of 40 degrees to the long axis of the foot should be used. The central ray (CR) should be directed to the base of the third metatarsal to emerge at a level just distal to the lateral malleolus.

A patient presents to your office for physical therapy for a painful 1st MTPJ. In addition to other modalities, you decide to use paraffin wax on this patient's foot. Which of the following is the correct description of how this heat is applied? A. Conduction B. Convection C. Conversion D. Radiation

Correct answer: (A) Conduction. Explanation: There are 4 main ways that heat can be applied to the body. Conduction is the transfer of energy from a hot surface to a cold surface. Examples of this are paraffin wax as well as moist heat packs. Convection is the transfer of heat through the movement of something over the body. An example of this is Whirlpool/Hydrotherapy. Conversion is described as the way to convert one energy to another. In the case of ultrasound, it is commonly sound energy to heat. Radiation is usually applied through UV rays- not commonly used in physical therapy any longer. In this case, the answer choice for paraffin would be conduction.

This is a prescription bar for clubfoot. The shoes attach to the bar with screws. The screws, or shoe clips, should be set at 70 degrees abducted to follow the treatment after serial casting for severe clubfoot. A. Dennis Brown Bar brace B. Bebax shoe C. Mitchell/Ponseti brace D. Wheaton clubfoot brace

Correct answer: (A) Dennis Brown bar brace. Explanation: The Denis Brown bar is a prescription bar for clubfoot, the shoes attach to the bar with screws, the screws or shoe clips should be set at 70 degrees abducted to follow the treatment after serial casting for severe clubfoot. It should be worn for 18-23 hours after casting. (See picture below) (B) It is a prescription brace, non-weight-bearing. It applies continuous corrective force, the angles are set by a prescribing physician. It has a plantar bar that can be adjusted up to three planes. The brace can be adjusted during treatment as needed using an Allen wrench, provided with shoes. This brace can be used for moderate and flexible to semi-flexible deformity or as a maintenance therapy, with little tibial deformity. (C) Developed by Dr. Ponseti and John Mitchell, the shoes attach to the bar with screws that attach to plate into shoe. Prescribed after serial casting cases, it should be worn for 23 hours, daily for 3 months, then while sleeping until 3.5 to 4 years old. The length of the bar is similar to the Denis Brown Bar. The bar should be between the heels. The shoe is set at 60 degrees abduction. The shoe is also in 15 degrees dorsiflexion when attached to the bar. (D) It is a prescription brace, thermoplastic knee AFO, applied to the lower limb with hook and loop closure, the connection is at the leg region and allows for growth in length or rotational deformities. The patient cannot walk with the brace due to knee flexion at 90 degrees, indicated for mild to moderate, flexible clubfoot as initial therapy, can also be used as maintenance after cast therapy.

When a lesion marker is placed on the skin for a radiographic study, which of the following are the only positioning techniques that should be used? A. Dorsoplantar view and Lateral view B. Dorsoplantar view and oblique view C. Oblique view and lateral view D. Lesion marker can only be used when taking the oblique view

Correct answer: (A) Dorsoplantar view and Lateral view. Explanation: If a lesion marker is placed on the skin, only dorsoplantar and lateral positioning techniques should be performed. Marker position cannot be accurately assessed with oblique views. Furthermore, the tubehead must be positioned vertically (0 degrees) for the dorsoplantar projection. Any other tubehead angulation will alter the position of the marker relative to the foot structures.

Which of the following inhaled general anesthetic agents is best used in patients with asthma? A. Sevoflurane B. Isoflurane C. Desflurane D. Propofol

Correct answer: (A) Sevoflurane. Explanation: Sevoflurane is a sweet-smelling, nonflammable inhalational anaesthetic for induction and maintenance of general anesthesia. After desflurane, it is the volatile anesthetic with the fastest onset and offset. Propofol is an IV anesthetic agent and is not used for inhalational so this can be excluded automatically. Sevoflurane is an agent that acts as a bronchodilator and does not have a pungent odor like other inhaled anesthetics. This best matches the answer choices. Isoflurane is also considered a bronchodilator but it has a pungent odor which can cause breath holding, coughing and laryngospasm. Desflurane has a pungent odor and by some is considered to be a bronchoconstrictor in patients who smoke so should be avoided in patients with lung problems.

Offset drilling did play a role in the development of the following plate? A. Dynamic compression plate (DCP) B. Neutralization plate C. Buttress plate D. Coverleaf plate

Correct answer: (A) Dynamic compression plate. Explanation: The DCP plate is a self-compressing plate. It makes axial compression possible, and this is done by combining screw geometry with screw insertion. Historically, self- compression plates were the following: a) a semi-tubular plate which is only 1mm thick and easily deformable, it owes most of its abilities to its borders which will "bite" into the underlying surface as the screws are tightened, b) the one-third tubular plate, only useful in the treatment of lateral malleolar fractures and fractures of the metacarpals and metatarsals there are also the quarter tubular plates, small T plates and L plates. The one third tubular plate is progressively being replaced by the dynamic compression plates (DCP). To achieve axial compression by means of self-compressing plates, the fracture has to be first reduced, the screws are then inserted eccentrically through the oval holes with corresponding drill sleeve as far from the fracture as possible. When the screw head contacts the edge of the plate, a force displaces the screw toward the center of the hole of the plate, a force displaces the screw toward the center of the hole in the plate. Displacement of the screw by eccentric plate contact transports the underlying bone as the is inserted, this is known as offset drilling, this concept resulted in the development of the Dynamic compression Plate (DCP). The neutralization plate is a protective device because it controls bending, translational, torsional forces. Use of lag screws only without a neutralization plate for long bone diaphyseal fixation will lead to failure. Lag screws insertion provides compression between fracture elements and is referred to as interfragmentary compression. The buttress plate is used to push bone fragments together, producing interfragmentary thereby producing interfragmentary compression with a pushing action. During screw application with this particular plate, elastic deformation of the plate is converted into anti-shear and compression forces. This plate can be applied without compression. Coverleaf mostly used in the fixation of a pilon fracture to buttress the fracture.

According to the American College of Chest Physicians Guidelines, what is the recommended venous thromboembolism prophylaxis for a 35 year-old healthy male undergoing a bunionectomy procedure? A. Early ambulation B. Aspirin C. Low molecular weight heparin D. Elastic stockings

Correct answer: (A) Early ambulation.

A 5 year old child presents to clinic with his mother. The child has had an elevated temperature the last several days and was diagnosed with strep throat by his pediatrician. The mother states that the child has refused to take his antibiotic and his symptoms are worsening. He is now limping and complains of pain in his right foot. Radiographs are obtained and the diagnosis of first metatarsal hematogenous osteomyelitis given. Based on this diagnosis, which area of the metatarsal will not exhibit radiographic signs of infection? A. Epiphysis B. Distal metaphysis C. Diaphysis D. Proximal metaphysis

Correct answer: (A) Epiphysis. Explanation: This question deals with routes of contamination. There are four main routes and these are: hematogenous, contiguous spread, direct inoculation and postoperative wound infections. In order to answer this question correctly, you need to remember the different vascular patterns which occur with infants, children and adults. In a child the normal vasculature does not penetrate the growth plate, therefore the blood-borne organisms cannot cross into the epiphysis of a tubular bone. Radiographic changes begin internally within the bone marrow and can affect the diaphysis and metaphysis, however the epiphysis remains free from infection.

Which of the following is not typically considered a characteristic of the pronated foot? A. Excessive calcaneal inversion B. Increased flexibility, unless peroneal spasm present C. Hallux valgus D. Uneven weight distribution

Correct answer: (A) Excessive calcaneal inversion. Explanation: The basic characteristics of the "pronated" foot are excessive calcaneal eversion, beyond 2-3 degrees; increased flexibility (unless peroneal spasm is present), uneven weight distribution, hallux valgus, contracted digits, neuroma, and postural symptoms involving the leg, knee, hip, and back. With pronation, adduction and plantarflexion of the talus, along with eversion of the calcaneus occur. As the subtalar joint becomes unstable, the midtarsal joint unlocks, and pathologic changes occur both proximally and distally. When this type of function is chronic, marked compensatory changes are noted.

A 31 year old patient with a high arch cavus foot presents to the office with a chief complaint of hammertoes right foot. Patient states that he has tried and failed conservative treatment options and is interested in surgery. On examination you notice a rigid contracture at the proximal interphalangeal joint. You also notice the patient primarily has the hammertoes during the swing phase of gait. Which of the following is the most likely etiology of above description of hammertoe and what is the best treatment option? A. Extensor Substitution, Arthrodesis PIPJ B. Flexor Stabilization, Arthroplasty PIPJ C. Extensor Substitution, Arthroplasty PIPJ D. Flexor Substitution, Arthrodesis PIPJ

Correct answer: (A) Extensor Substitution, Arthrodesis PIPJ. Explanation: There are three main etiologies of hammertoe deformities. Flexor stabilization is the most common and this is when flexors fire earlier and stay contracted longer overpowering the interossei. This is mainly seen in a pronated foot. Extensor substitution is mainly seen in cavus feet and this is when the long extensors fire earlier and stay contracted longer overpowering the lumbricals. This is mainly seen in the swing phase of gait. Flexor substitution is the last and least common and is typically seen after rupture of triceps surae musculature. Arthoplasty is primarily done when there is a reducible deformity when completing Kelikian push up test while arthrodesis is completed for rigid deformities as a whole.

What is the best treatment for a compartment pressure of 30 mmHg or higher? A. Fasciotomy B. Ice and elevate the limb C. Compressive therapy D. Immobilization

Correct answer: (A) Fasciotomy. Explanation: Once the diagnosis of acute compartment syndrome has been made, the goal of relieving pressure in the affected compartment(s) must be met. Fasciotomy is the definitive treatment for acute compartment syndrome. Numerous techniques have evolved with the discovery of additional foot compartments. The choice of fasciotomy is governed by surgeon preference, other planned procedures, and preexisting soft-tissue injury. Elevation of the affected extremity may reduce swelling, but should not exceed the level of the heart as doing so could further compromise tissue perfusion. (B) (C) (D) Will be incorrect as compartment syndrome does not abate with NSAID's, compression or elevation, or immobilization.

This orthotic type is designed to distribute plantar weight more evenly in a cavus foot structure: A. Shaffer plate B. Whitman plate C. UCBL D. Whitman-Roberts plate

Correct answer: (A) Shaffer plate. Explanation: Schaffer plates were originally made of metal in order to evenly distribute weight on the plantar foot in a cavus deformity. Modifications can be made to have it function as an accommodative device. (B) The Whitman plate was used in pediatric patients to apply pressure to the navicular with weightbearing in order to discourage pronation. The patient requires the ability to supinate their foot in order to use this device. (C) A UCBL device has a deep heel cup, arch support to stabilize the midtarsal joint and high lateral walls. It is used primarily for stabilization or correction of flexible rearfoot deformities. (D) The Whitman-Roberts plate has a high medial flange with a lateral clip and cupped heel seat to prevent adduction and plantarflexion of the talus.

When is the best time to schedule diabetic patients on insulin therapy who are scheduled for surgery? First thing in the morning Because of their high risk of infection, they should be scheduled as the last case of the day Should be schedule after ABI Should be scheduled if renal function are within normal limits

Correct answer: (A) First thing in the morning. Explanation: Preoperative evaluation of patients with diabetes mellitus should focus on some of the more common comorbidities and sequelae of the disease process like peripheral sensory neuropathy, cystopathy, gastroparesis, hypoglycemic unawareness, and cardiovascular autonomic neuropathy, and silent ischemia, angina without chest pain, retinopathy, nephropathy, and macrophage dysfunction with blood sugar > 150 mg/dL, hyperlipidemia and hypertension. These patients are at increased risk for cerebrovascular accidents, myocardial infarctions, acute renal failure, and postoperative wound complications. This risk may be mitigated with control of perioperative hyperglycemia. Musculoskeletal manifestations are common and may be predict difficulties with laryngoscopy and endotracheal intubation. Such complications are important to note in the perioperative period and provisions should be made avoid hyperglycemia, and minimize hypoglycemia. So in type 2 and type 1 diabetic patients on insulin therapy, these patients should be scheduled first thing in the morning.

Contact phase is a period in the stance phase of gait. What initiates and ends the contact phase? A. Heel contact to toe off of the opposite limb B. Heel contact to toe off of the same limb C. Toe off to heel lift of the support foot D. Heel off to heel strike of the opposite limb

Correct answer: (A) Heel contact to toe off of the opposite limb. Explanation: The contact phase is the period initiated by initial floor contact of the heel to toe off of the opposite limb. It occupies 10% of the gait cycle and 18% of the stance phase.

When considering lower extremity surgery in an RA patient, the correct sequence of surgical procedures is: A. Hip, followed by knee, followed by ankle. B. Ankle, followed by knee, followed by hip. C. Knee, followed by ankle, followed by hip. D. The sequence of deformity reduction is not important.

Correct answer: (A) Hip, followed by knee, followed by ankle. Explanation: Wilkinson looked at the "reliability" of procedures, and construction of a platform from which to build upon. THR and TKR have historically been more reliable and successful than TAR; many still view ankle fusion as the "gold standard." Restoring alignment of the hip/femur will allow subsequent better alignment and rotation of the knee components. Finally, with the suprastructural joints aligned, the ankle can be adequately addressed.

What is the primary cause of radiation induced death? A. Inability to fight infection because of damage to the bone marrow and bleeding through the GI tract. B. The dose effect and dose fractionation. C. When an individual is exposed to 1 mrem per day for a year. D. Is the radiation of the central nervous system of about 2.5 Sv.

Correct answer: (A) Inability to fight infection because of damage to the bone marrow and bleeding through the GI tract. Explanation: The mean lethal dose or lethal dose to kill 50% in 30 days for humans is estimated to be about 4.5 to 6 Sv and may be higher if autologous bone marrow transplants are done. The primary cause of radiation-induced death is the inability to fight infection due to damage of the bone marrow and bleeding through the GI tract. (B) When radiation is delivered at low doses, it is less effective than when exposure is at acute dose rates. And when a given total dose is delivered in two fractions, with a time interval between fractions, that fractionated dose is found to be less effective than the same total dose given at once. (C) It has been estimated that, on average, an individual in the USA is exposed to about 3.6mSv (360 mrem) per year, most from natural background. While the Mean Lethal dose (lethal dose to kill 50% in 30 days) for humans is estimated to be 4.5 to 6 Sv. (D) The central nervous system is the most resistant to radiation and the Mean lethal dose or lethal dose to kill 50% in 30 days for humans is estimated to be about 4.5 Sv to 6 Sv.

Shortly after performing a local hallux block in your office for a partial nail avulsion, your patient complains of perioral numbness and lightheadedness. You note that you injected approximately 8 cc of 1% preservative-free lidocaine without epinephrine. Your patient denied any paresthesias during the injection. Which of the following is the most likely explanation for your patient's symptoms? A. Intravascular injection of local anesthetic B. Intraneural injection of local anesthetic C. Toxic dose of lidocaine was exceeded D. Patient suffered an allergic reaction to lidocaine

Correct answer: (A) Intravascular injection of local anesthetic. Explanation: Local anesthetic blocks are frequently performed in an office setting for minor procedures. Complications are thankfully rare, but can occur. True allergic reactions to local anesthetics are not common. Allergic reactions are more common with ester anesthetics compared to amide anesthetics. Symptoms of allergic reactions can range from dermatologic manifestations to anaphylaxis. A true allergic reaction is very unlikely with preservative-free lidocaine. The amount of lidocaine injected in this scenario is well below the toxic dose if injected appropriately. The maximum dose for 1% lidocaine without epinephrine is 300 mg which equals a volume of 30 mL of anesthetic. Intraneural injection of anesthetic can cause pain as well as subsequent local damage to the nerves affected. Intraneural injections are often characterized by parasthesias or pain. However, as the patient did not report any paresthesias or excessive pain during the procedure, intraneural injection is less likely. Systemic effects of local anesthetics include circumoral numbness, lightheadedness, and tinnitus as lower doses, but can progress to seizures, coma and respiratory arrest at higher doses. These symptoms fit the description of the patient's symptoms in the question and is the most likely answer.

23 years old presents with osteomyelitis of the left heel. She has a past medical history significant for spina bifida. She will need surgical debridement. Preparing for the surgery, you are reviewing her allergies. Which one of the following is most common with this patient population? A. Latex allergy B. Antibiotics allergy C. Adhesive tape D. Pain medication

Correct answer: (A) Latex allergy. Explanation: Patient with a history of chronic urologic problems, spina bifida, and atopic dermatitis are considered at high risk for latex allergy. (B) Allergy to antibiotics are quite common, history is critical, but the main concern with spina bifida in the OR is latex allergy, so the entire team should be aware and latex free materials should be required. (C) Once again adhesive tape allergy is also quite common, history is equally critical here too, this is not known to be a primary concern for spina bifida, but latex allergy is. (D) Also a common allergy, it is important to gather proper information from the patient.

A 64 year old male presents to the clinic complaining of severe right ankle pain. He states the pain in the ankle worsens the longer he is on his foot. Icing and NSAIDs sometimes reduce, but do not eliminate the pain. The following radiograph was obtained (see image). What is the primary problem that has led to the condition producing the patient's symptomatology? A. Malunited calcaneal fracture with loss of calcaneal height below the knee B. Osteoarthritis of the ankle joint C. Anterolateral stage IV osteochondral defect of the talar dome D. Ankle equinus resulting in tibiotalar impingement

Correct answer: (A) Malunited calcaneal fracture with loss of calcaneal height below the knee. Explanation: The loss of calcaneal height has allowed dorsiflexion of the talus as it has sunken down into the calcaneus. This resultant dorsiflexion of the talus causes impingement of the anterior tibial plafond onto the talar neck. Repetitive impingement caused degenerative changes with joint space narrowing and spurring. Although this is an example of osteoarthritis of the ankle the question asks for the primary problem that has caused the patient's current condition. Additionally there is no radiographic evidence of an osteochondral defect. This combined with the dorsiflexed talus and the broken calcaneal hardware clearly point to the conclusion that the malunited calcaneal fracture is the primary problem. Additionally, ankle equinus does not cause tibiotalar impingement.

It is well accepted that the closing base wedge bunion procedure can result in significant metatarsal shortening. As a result a known potential complication of the closing base wedge procedure is transfer metatarsalgia. What other tendency inherent with the closing base wedge bunion procedure also can potentially result in transfer metatarsalgia? (Select all that apply) A. Metatarsal dorsiflexion B. Metatarsal supination C. Proximal sesamoid migration D. Metatarsal plantarflexion

Correct answer: (A) Metatarsal dorsiflexion. Explanation: Metatarsal dorsiflexion can easily occur with a closing wedge osteotomy simply due to the fact that the surgeon has a tendency to remove more bone an the dorsal aspect of the wedge than the plantar aspect of the wedge. (B) Metatarsal supination is not a complication of the closing wedge osteotomy and is not a potential cause of transfer metatarsalgia. (C) Proximal sesamoid migration does not occur with the closing wedge osteotomy and therefore is not a potential cause of transfer metatarsalgia. (D) Metatarsal plantarflexion would not cause a transfer metatarsalgia, but rather excessive plantar pressure under the first metatarsal head leading to possible sesamoiditis, callus and pain.

Which joint complex of the foot enables rotation of the leg and hindfoot to occur without the forefoot leaving the ground? A. Midtarsal Joint B. Subtalar Joint C. LisFanc's joint D. Metatarsophalangeal joint

Correct answer: (A) Midtarsal Joint. Explanation: The subtalar joint can be described as a mitered hinge that connects the talus to the calcaneus. When the axis of the hinge is 45 degrees, it creates a simple torque converter with every degree of vertical motion being translated into one degree of horizontal motion. With the addition of a pivot between the horizontal segments in our theoretical mitered hinge concept, the pivot prevents the entire horizontal segment from displacing in response to vertical segment rotation. This pivot corresponds to the midtarsal joint, allowing for rearfoot displacement with rotation of the leg, while keeping the forefoot flat to the surface.

A patient is scheduled for surgery. The patient has been on a recent course of glucocorticoid. You ordered an ACTH stimulation test which came back normal. What would dictate administration of steroids before surgery? A. Normal ACTH in the presence of unexplained hypotension B. ACTH stimulation test is unreliable C. If the deficiency in ACTH results in aldosterone deficiency D. Proceed with the surgery and monitor

Correct answer: (A) Normal ACTH in the presence of unexplained hypotension. Explanation: Test to detect perioperative adrenal suppression or, perhaps more importantly, identify patients who will respond to supplemental glucocorticoids have been neither sensitive nor specific. However, the short ACTH stimulation test is able to reliably assess adrenocortical function. If the test is abnormal preoperatively, supplemental perioperative glucocorticoid administration is justified. Even if perioperative ACTH stimulation is normal, and the patient is at high risk for perioperative adrenal insufficiency, if unexplained hypotension persists despite volume repletion, steroids should be administered in a dose consistent with the level of injury. Postoperatively, steroids should be continued until the stress response diminishes, usually 48 hours.

A 23 year old construction worker presents to the ED after a boulder fell on his left foot, damaging his steel toe boot. He denies any other trauma. The patient socks and shoes were removed before arrival to the fracture clinic. Past medical history: Unremarkable Past surgical History: None Social history: Non -smoker, no illicit drug abuse, no alcohol abuse Allergy: None What step is next? A. Physical exam B. Radiographic examination C. Casting D. Schedule open reduction internal fixation

Correct answer: (A) Physical exam. Explanation: After adequate history, the injured part should be examined. It is most helpful to compare the injured part to the opposite side to identify any deviation from normal in a particular patient. Inspection will identify ecchymosis and swelling as nonspecific signs of injury, wounds, indicating possible open fracture, dislocation, pallor or cyanosis indicative of impairment of circulation, and signs of preexisting disease or deformity. Gentle palpation may reveal crepitus, eliciting discrete area of pint tenderness. (B) After a complete history and thorough physical examination, x-rays of the injured foot, taken in the appropriate projections, enable the clinician to define the extent of osseous injury present. This is not the first step after a thorough history. (C) Treatment should be performed after diagnosis, after radiographic examination. (D) This answer choice is inappropriate at this time.

Which plane of the third stage of anesthesia is described by the following characteristics? - slow regular diaphragmatic breathing - eyelid reflex is present - amnesia - analgesia to mild painful stimuli - sedation A. Plane 1 B. Plane 2 C. Plane 3 D. Plane 4

Correct answer: (A) Plane 1. Explanation: When discussing the depth of anesthesia there are four separated planes contained within the third stage of anesthesia. Plane 1 is amnesia/analgesia, Plane 2 is delirium/excitement, Plane 3 is surgical anesthesia and Plane 4 is overdosage.

Which of the following medical conditions is most likely to contribute to wound failure in a diabetic patient undergoing digital amputation? A. Osteoarthritis B. Hypertension C. Renal failure D. Cerebral vascular disease

Correct answer: (C) Renal failure. Explanation: Renal failure along with poorly controlled diabetes, organ transplant, anemia and heart failure can contribute to failure of a wound to heal. Osteoarthritis, hypertension, and cerebral vascular disease are all significant co-morbitities, however they do not have any significant impact on the healing of digital amputations.

23 year old construction worker presents to the ED after a boulder fell on his left foot, damaging his steel toe boot, denies any other trauma. The patient socks and shoes were removed before arrival to the fracture clinic. Past medical history: Unremarkable Past surgical History: None Social history: Non -smoker, no illicit drug abuse, no alcohol abuse Physical exam yields the following: - No open wounds - Ecchymosis and swelling to the dorsum of the left foot - Swelling to the left foot - There is no pallor, or cyanosis - Gross sensations all intact to the left foot - Gentle palpation reveals tenderness and crepitus to the 2nd metatarsal He is diagnosed with an acute close fracture of the second metatarsal and incomplete fracture of the 3rd proximal phalanx. (See picture below) An attempt at closed reduction is performed with placement of fiberglass cast, (picture below). What will you do next? A. Post reduction x-ray B. Post reduction ultrasound C. Post reduction CT D. Post reduction MRI

Correct answer: (A) Post reduction x-ray. Explanation: If an attempt at post reduction is performed, always order post reduction plain film radiograph first. (B) Not applicable in this instance. (C) Plain film should be ordered first, if failure is noted at reduction, and there is surgical planning involved, CT can therefore be ordered. It is uncommon for metatarsal fracture. (D) Not applicable in this case. *** Post reduction plain film is below.

An external fixator will provide the greatest amount of bone repair if it presents with the following characteristics: A. Prevents bending, torsion, and permits small amount of movement in axial direction. B. Allows bending, shear and rigid fixation C. Less amount of movement in axial direction, less bending and torsion D. Greater amount of immobilization, less bending and more torsion

Correct answer: (A) Prevents bending, torsion, and permits small amount of movement in axial direction. Explanation: Several different designs of external fixators exist; each has unique biomechanical characteristics. A fixator that permits a small amount of movement in the axial direction, but prevents movement in torsion, bending, and shear stimulates the greatest amount of bone repair. Several studies compared the effect of a small amount of movement in the axial direction with rigid fixation; in all cases, significantly greater amounts of new bone formation was seen if axial micromovement was permitted.

A patient presents complaining of unilateral joint pain. Upon obtaining an xray, you identify an "ivory" phalanx of the distal hallux with periostitis or "whiskering." You suspect this patient may have: A. Psoriatic arthritis B. Rheumatoid arthritis C. Osteoarthritis D. Gouty arthritis

Correct answer: (A) Psoriatic arthritis. Explanation: On radiographic examination, psoriatic arthritis may be characterized by an "ivory" or sclerotic phalanx. There is often periostitis which is often referred to as "whiskering" due to the speculated appearance radiating away from the bone margin. (B) Rheumatoid arthritis is often symmetrical and is radiographically characterized by even joint space narrowing. Later in the disease process lateral deviation at the level of the metatarsophalangeal joint may be noted. (C) Osteoarthritis is often characterized by subchondral sclerosis, uneven joint space narrowing and small osteophytes referred to as joint mice. (D) Gouty arthritis is often characterized on x-ray with Martel's sign, which are well defined erosions with overhanging margins of bone. This occurs as new bone production attempts to encapsulate the gouty tophi.

Which surgical technique is most appropriate for a cystic osteochondral defect of the talus with the articular cartilage intact? A. Retrograde drilling B. Microfracture C. OATS D. ACI

Correct answer: (A) Retrograde drilling. Explanation: In large cystic lesions with the overlying articular cartilage intact, retrograde drilling is performed to bring blood supply to the site without disrupting the cartilage. Cancellous bone graft may be used to fill the cystic defect. (B) Small chondral lesions are often addressed with arthroscopic microfracture. This breaks the calcified subchondral layer to obtain growth factors from the bone marrow, creating a fibrin clot which eventually fills the defect. (C) Osteochondral autograft transfer system (OATS) is used to restore defects of articular cartilage. An arthrotomy is performed to gain access to the joint, the lesion is curettaged, and autogenous bone graft is harvested and tamped into the talar defect. (D) Autologous chondrocyte implantation (ACI) is used to address cartilage damage of the bone, in which cells are harvested, grown in a lab and re-implantated into the deficit.

Which of the following procedures below is indicated for the fifth toe deformity pictured below? A. Ruiz-Mora Procedure (Correction, cock-up fifth toe, with plastic skin closure) B. Thompson Procedure C. Post Procedure D. Lapidus Procedure (first tarsometatarsal arthrodesis)

Correct answer: (A) Ruiz-Mora Procedure. (Correction, cock-up fifth toe, with plastic skin closure) Explanation: The Ruiz-Mora procedure is the only procedure listed above indicated for a cock- up fifth toe. It consists of an elliptical plantar incision, resection of the proximal phalanx and suturing of the incision to medial deviate and plantar flex the toe with closure. (B) The Thompson procedure is indicated for an underlapping fifth toe. The picture above is, in contrast, a cock-up fifth toe deformity. It consists of a Z-type incision dorsally over the proximal phalanx, the resection of the fifth proximal phalanx in its entirety, reefing of the capsule to fill the dead space and corrective closure of the Z incision. (C) The Post procedure is for correction of the rectus hammertoe. It consists of resection of the head of the proximal phalanx through either a linear or transverse elliptical incision and reefing of the capsule with closure. (D) The Lapidus procedure (first tarsometatarsal arthrodesis) is indicated for severely overlapping fifth toes.

In a polytrauma patient with a long bone fracture, when IM nailing is considered with reaming, it could cause this phenomenon: A. Second hit phenomenon B. Infection C. Massive bleeding D. Massive hematoma

Correct answer: (A) Second hit phenomenon. Explanation: There is evidence highlighting that there is a local immuno-inflammatory response in the intramedullary canal after femoral fracture. Femoral shaft fracture results in a significantly raised local IL-6 concentration and that if you ream, this will result in a further significant rise in these pro-inflammatory molecules. We suggested that this provides evidence in support of the hypothesis that the femoral canal contributes to release of inflammatory reaction into the peripheral circulation resulting in a second hit phenomenon. You should take these findings into account when dealing with the management of polytrauma patients with femoral fractures.

The main contraindication to the Lapidus fusion is a (an): A. Short metatarsal B. Elevated metatarsal C. Open growth plate D. Top-level athlete

Correct answer: (A) Short metatarsal. Explanation: Regardless of how the cartilage is resected (including the subchondral bone plate), shortening will occur. Shortening can be minimized by avoiding planal resection with a sagittal saw, and instead using rongeurs, osteotome and mallet to fish-scale, or power burr. Additionally, if shortening does occur, bone grafting can be utilized. However, the problems associated with grafting can be avoided simply by avoiding performing the procedure on an already short metatarsal. An elevated metatarsal is not a contraindication; the fusion site can be fashioned to excise more bone plantarly than dorsally; in fact, intentional plantarflexion of the fusion site is frequently preferred to compensate for shortening and prevent second metatarsal overload although McGlamry's states: "care must be taken not to plantarflex the first ray as sesamoiditis could result." An open growth plate is a relative contraindication, and really is surgeon's philosophy. The literature does support the Lapidus fusion in the presence of an open growth plate if smooth wires or pins are used. Also, use of the Lapidus in a high level athlete is a relative contraindication, as post-operative stiffness could affect their performance.

During the gait cycle, this phase is initiated by opposite-foot toe- off and terminates with opposite heel strike? A. Single limb support B. Double limb support C. Initial double support D. Cadence

Correct answer: (A) Single limb support. Explanation: Single limb support, or single stance, is initiated by opposite-foot toe-off and terminates with opposite side heel-strike. Therefore, single limb support is identical to the period of swing of the opposite limb and so is responsible for support of the entire body. The duration of single limb stance is often a measure of an individual's capability to stabilize and support the body.

When performing single lag screw fixation of an oblique base wedge osteotomy what is the ideal relationship of the screw relative to the osteotomy? A. Slightly less than perpendicular B. Perpendicular C. Slightly beyond perpendicular D. 45° to the osteotomy

Correct answer: (A) Slightly less than perpendicular. Explanation: The ideal relationship of the lag screw relative to the osteotomy is insertion of the screw slightly less than perpendicular. This will create a distal shift of the distal fragment relative to the proximal fragments. In this scenario the intact cortical hinge is placed under tension. This acts much like a tension band in this circumstance and provides additional compression at the osteotomy and enhance the overall stability. (B) Orienting the lag screw perpendicular to the osteotomy will provide compression. With the intact cortical hinge 100% of the screw's effect will go towards compression. (C) If the lag screw is oriented beyond perpendicular to the osteotomy, as the screw is tightened a proximal shift in the distal fragment will occur and place pressure on the cortical hinge. If enough pressure is applied to the hinge it can cause the hinge to fracture. Even in the case where fracture does not occur the hinge is weakened. (D) Orienting the lag screw 45° relative to the osteotomy will provide compression and significant translation which will assuredly affect the hinge and the overall stability of the fixation.

In reference to MR imaging, what is meant by the time constant T1? The time it takes for the protons to realign with the static magnetic field, B0, following a radio frequency excitation pulse The time it takes for the protons to realign with the head-to-foot axis, Z-axis, following an radio frequency pulse The time it takes for the transverse magnetization to decay over time The time it take from the moment of the radio frequency pulse until transverse magnetization

Correct answer: (A) The time it takes for the protons to realign with the static magnetic field, B0, following a radio frequency excitation pulse. Explanation: Proton density is an important property that each tissue has. The second important property is how quickly the protons realign with the static magnetic field, B0, following a radio frequency excitation pulse. This describes the time constant called T1. A second time constant, T2, describes how quickly the transverse magnetization decays over time, reflecting the tendency of different protons, which are initially aligned together after a 90 degree pulse, to process quickly out-of-sync with each other.

What term below is best described by the following statement? This type of nonunion occurs when there is an intermediate fragment with decreased or absent blood supply. The intermediate fragment heals to one main fragment, but not to the other. A. Torsion wedge B. Comminuted C. Defect D. Atrophic

Correct answer: (A) Torsion wedge. Explanation: This question focuses on nonunions. There are two main categories of nonunions. Hypertrophic nonunions are those where union is possible in the right environment. Atrophic nonunions are those where union is not possible due to poor bone quality. These two main categories can be further broken down into subcategories. Hypertrophic nonunions are divided into three variants: elephant foot; horses hoof; oligotrophic. Atrophic nonunions are divided into 4 variants: torsion wedge; comminuted; defect; atrophic. The choices above are the four variants of atrophic nonunions. The one described above is the torsion wedge variant of an atrophic nonunion. Conversely, the comminuted variant has the presence of one or more necrotic intermediate fragments. The defect variant has gapping at the fracture site due to loss of a fragment. Lastly, the atrophic variant forms when non-osteogenic scar tissue fills a void at the fracture site.

Following successful callous distraction with an external fixator, how long should the frame be kept on for to ensure complete ossification? A. Twice the time it took to complete the lengthening B. The same amount of time it took the complete the lengthening C. Three times the time it took to complete the lengthening D. 14 days after the lengthening has completed

Correct answer: (A) Twice the time it took to complete the lengthening. Explanation: After the distraction period has been completed, the distraction gap is ossified incompletely. To allow satisfactory ossification, the frame is kept on for a period of time that is approximately twice that which was taken to complete the lengthening. The subject is encouraged to weight bear to stimulate bone healing.

In elective surgery, what is the standard for ordering an ECG during the preoperative testing? A. Women at 50 years of age, men at 40 years of age B. Women at 40 years of age, and men at 50 years of age C. Women at 35 years of age, men at 50 years of age D. Women at 45 years of age, and men at 50 years of age

Correct answer: (A) Women at 50 years of age, men at 40 years of age. Explanation: Part of the laboratory testing during the preoperative period, an ECG is generally recommended in men over the age of 40 years, in women older than 50 years and in those with known underlying cardiovascular disease. Preoperative laboratory testing should be selective and not routine. Current recommendations are for current laboratory tests based on specific signs, symptoms and diagnosis. (B) (C) (D) The statements fall outside of the guidelines for each category.

What is the average volume of local anesthetic injected for a hallux block? A. 1-2 mL B. 3-5 mL C. 5-7 mL D. 7-10 mL

Correct answer: (B) 3-5 mL. Explanation: For the hallux, often 3 to 5 mL of 1% lidocaine is used for a complete toe block; for a lesser toe block, considerably less volume is used. Less than 3 mL may not cause anesthesia and more than 5 mL may cause increased pressure and discomfort for the patient without an increase in anesthesia. Technique for a digital block involves raising a wheal on the dorsomedial aspect of the toe at its base. The dorsomedial and plantar medial sensory nerves are anesthetized as the needle is directed vertically from the dorsal to the plantar surface of the toe. The needle then is withdrawn into the subcutaneous tissue and is turned in a horizontal lateral direction. The dorsal aspect of the toe is then infiltrated with the anesthetic agent. The needle is withdrawn, and another wheal is raised on the dorsolateral aspect of the base of the toe. The dorsolateral and plantar lateral digital nerves are then anesthetized by a vertical injection on the lateral aspect of the toe.

What is the minimum amount of motion required from the longitudinal axis of the midtarsal joint during gait? A. No motion is required during gait B. 5 degrees C. 10 degrees D. 15 degrees

Correct answer: (B) 5 degrees. Explanation: A minimum of 4-6 degrees of longitudinal axis motion in gait is required to maintain the foot plantigrade when the subtalar joint pronates to that degree during the contact phase of gait. The longitudinal axis is supinated during contact phase of gait, and pronated in the midstance and propulsion stages of gait.

Which physical status class, as described by the American Society of Anesthesiologists, is best described by a patient having a severe non-incapacitating systemic disease? A. ASA Class II B. ASA Class III C. ASA Class IV D. ASA Class V

Correct answer: (B) ASA Class III. Explanation: The American Society of Anesthesiologists surgical physical status classification system is a very effective system at estimating risk of mortality. With that said, an ASA class III patient is one who has a severe non-incapacitating systems disease, such as a severe diabetic. Class III patients have a 4% mortality. To review the classification system: Class I is described as normal, healthy individuals. Class II is described as mild systemic disease such as well controlled diabetes. Class IV is described as a severe incapacitating disease that is a constant threat to life. An example patient that would fit this classification would be a patient with cardiovascular and renal disease with renal Impairment. Mortality for this class is 25%. Class V is described as a moribund patient who is not expected to survive 24 hours with or without operation. An example of this type of patient is one with a rupture aortic aneurysm. Lastly, there is a class VI which is best described as a brain-dead organ donor. One final note is that if a surgery is an emergency, the letter E accompanies the class i.e. IIE, IVE. As class V is nearly always an emergency, there is hardly a time that the E would be attached and by definition an ASA class IV cannot have an E attached.

What is the specific sequence of release to be performed during anatomic dissection in the first metatarsophalangeal joint space? Adductor hallucis tendon, fibular sesamoid ligament, excision of the fibular sesamoid, tenotomy of the lateral head of the flexor hallucis brevis Adductor hallucis tendon, fibular sesamoid ligament, tenotomy of the lateral head of the flexor hallucis brevis , excision of the fibular sesamoid Excision of the fibular sesamoid, adductor hallucis tendon, fibular sesamoid ligament, tenotomy of the fibular sesamoid Tenotomy of the lateral head of the flexor hallucis brevis, excision of the fibular sesamoid, adductor hallucis tendon, fibular sesamoid ligament

Correct answer: (B) Adductor hallucis tendon, fibular sesamoid ligament, tenotomy of the lateral head of the flexor hallucis brevis , excision of the fibular sesamoid. Explanation: Adductor hallucis tendon, fibular sesamoid ligament, tenotmy of the lateral head of the flexor hallucis brevis, extension of the fibular sesamoid. This technique begins with the release of the adductor tendon but progresses only as needed to release the contracture effectively and to allow unrestricted sagittal range of motion of the metatarsophalangeal joint. With each additional step in the release process is an increased potential for lateral instability. However, without these steps or other appropriate measures, lateral contracture may persist and may encourage recurrence of the hallux abducto valgus deformity. Therefore, the surgeon is attempting to recreate a more normal balance of the soft tissues at the joint level.

A patient with atrial fibrillation was scheduled to undergo a right ankle arthroplasty with implant. The patient's warfarin was stopped five days prior to the operation. Labs are checked the morning of surgery and the INR is 1.8. What is the most appropriate plan of action? A. Administer 1mg IV protamine sulfate B. Administer 1mg of oral vitamin K C. Administer 1g of oral vitamin K D. Administer 1g IV dantrolene

Correct answer: (B) Administer 1mg of oral vitamin K. Explanation: To reverse warfarin, oral vitamin K may be given. In a less urgent scenario, 1-2mg of oral vitamin K may be administered to reduce the INR to a safe operative level. (A) Protamin sulfate is used to reverse heparin and would not be appropriate to reverse warfarin. (C) 1g of oral vitamin K is a large dose and may be dangerus for the patient. 1-2mg of oral vitamin K may be administered in a less urgent situation. However, in very emergent operations, 5mg IV vitamin K may be given in conjunction with prothrombin complex with factors II,VII,IX and X. (D) Dantrolene is used in the treatment of malignant hyperthermia and would not be appropriate to reverse warfarin.

In 1990 Ian Taylor presented his landmark paper establishing the concept of angiosomes throughout the body. Which of the following statements regarding angiosomes is FALSE? An angiosome is defined as anatomic unit of tissue fed by a source artery. All angiosomes in the foot and ankle ultimately originate from the posterior tibial artery, anterior tibial artery and the arcuate artery. There are six angiosomes in the foot and ankle. Choke vessels link each angiosome to one another.

Correct answer: (B) All angiosomes in the foot and ankle ultimately originate from the posterior tibial artery, anterior tibial artery and the arcuate artery. Explanation: All six angiosomes in the foot and ankle ultimately originate from the posterior tibial artery, anterior tibial artery and the peroneal artery. The posterior tibial branches into the calcaneal artery, medial plantar artery and lateral plantar artery. The anterior tibial artery branches into the lateral and medial malleolar arteries. Finally the peroneal artery branches into the calcaneal branch and the anterior perforating branch. All the distal branches feed resultant angiosomes. The choke vessel system links the angiosomes creating a unified network so that one source artery can provide blood flow to multiple angiosomes beyond its immediate border. In addition, an angiosome is correctly defined as an anatomic unit of tissue fed by a source artery.

Radiographs have little clinical value in the work up of an intermetatarsal neuroma. Radiographs are generally most often used to rule out other osseous or biomechanics etiologies for the metatarsalgia complaint. However, there is one radiographic sign, while not always present, does suggest an intermetatarsal neuroma. What is that radiographic sign? A. a radio-opacity of the affected interspace B. an increased intermetatarsal angle of the affected interspace C. divergence of the adjacent digits D. an obscuring of the cortical margins of the adjacent metatarsal heads

Correct answer: (B) An increased intermetatarsal angle of the affected interspace. Explanation: In 1967, before the advent of magnetic resonance imaging, J. Sullivan presented a paper on the radiographic diagnosis of neuromas. In his paper he found that all the individuals afflicted with a neuroma showed an increase in the intermetatarsal angle of the affected interspace. The neuroma itself is not visible on plain film radiographs. A divergence of the digits, also known as Sullivan's sign is a clinical sign demonstrating divergence of adjacent toes. The physical mass of the neuroma, even if it does overlap the adjacent surfaces of the metatarsal heads, does not cloud or obscure the cortical margins of the metatarsal heads.

What would be the appropriate treatment for the fracture blister on the dorsum of the left foot? A. Aspiration and deroofing of the blister B. Aspiration of the blister C. Deroofing of the blister D. Leave the blister alone

Correct answer: (B) Aspiration of the blister. Explanation: Serous blisters are to be aspirated and the roof is allowed to collapse. It is not removed. It will then serve as a biologic dressing. After desiccation of the roof, it can be debrided. Deroofing of the these blisters increase infection rates. The presence of any type of fracture blister will delay the timing of surgical intervention. In the foot, calcaneal and ankle fractures are more likely to involve fracture blisters. The presence of fracture blisters lessens the incidence of compartment syndrome.

Where does the flexor digitorum brevis split? A. Near the midshaft of the metatarsal 2-5 B. At the base of the proximal phalanx 2-5 C. Near the middle of the distal phalanx 2-5 D. Near the middle of the middle phalanx 2-5

Correct answer: (B) At the base of the proximal phalanx 2-5. Explanation: The flat tendon divides into 2 slips at the level of the base of the proximal phalanx. The slips contour the long flexor tendon on each side, pass underneath and insert into the middle phalanx.

A 43 year old male patient presents to your office for a pre-operative evaluation for bunionectomy left foot. X-rays are taken and below are the measured angles. Which of the following procedure choices are the best? IM Angle: 12 degrees PASA: 7 degrees Sesamoid Position: 5 Hallux Abductus Angle (HAA): 13 Hallux Abductus Interphalangeal Angle (HAI): 7 Hypermobility- None Noted A. Reverdin Green Bunionectomy B. Austin Bunionectomy C. Lapidus Fusion D. Closing base wedge bunionectomy

Correct answer: (B) Austin Bunionectomy. Explanation: In going through a question like this, it is important to take the x-ray angles one at a time and determine whether they are normal or abnormal. In this case there is moderate elevation of the IM angle, normal PASA, abnormal sesamoid position, normal HAA, normal HAI and no hypermobility. Thus in this case, the Austin bunionectomy is the best choice. The Reverdin bunionectomy is primarily used for an increase in the PASA. A Lapidus fusion is primarily use for hypermobility of the 1st ray with elevated IM angle. The closing wedge bunionectomy is usually reserved for higher IM angles (some sources say greater than 15).

What is the mechanism of action of local anesthetics? A. Blockage of potassium channels on the nerve membrane inhibiting depolarization B. Blockage of sodium channels on the nerve membrane inhibiting depolarization C. Opening of potassium channels on the nerve membrane inhibiting depolarization D. Opening of sodium channels on the nerve membrane inhibiting depolarization

Correct answer: (B) Blockage of sodium channels on the nerve membrane inhibiting depolarization. Explanation: Local anesthetics stabilize the neural membrane and prevent depolarization and repolarization. The manner in which they do this is by reversibly blocking the sodium channels on the nerve membrane which, in turn, stabilizes the membrane, preventing depolarization. Motor function may be partially or totally affected depending on the size of the nerve, as well as the the concentration of local anesthetic used. Local anesthetic are lipid soluble. The larger the nerve is the lesser and slower the local's effect. The order of loss of sensations is usually pain, temperature, touch, proprioception, and finally skeletal muscle tone.

You are scheduled to performed an open reduction of a long standing Lisfranc injury in a 55 year old patient who is also complaining generalized fatigue, dyspnea on exertion, and transient tachycardia. What preoperative laboratory tests should be ordered? A. Chemistry profile B. Complete blood count C. Coagulation profile D. Renal and liver function tests

Correct answer: (B) Complete blood count. Explanation: Complete blood count should be performed in patients with a recent history of blood loss, a history of fatigue, dyspnea on exertion, liver disease, and signs of coagulopathy. (A) If the patient has a history of hypertension, diuretic use, chronic obstructive pulmonary disease or obstructive lung disease, blood loss, renal disease, or chemotherapy, a chemistry profile should be ordered. (C) Is indicated if the patient is receiving anticoagulant therapy, has a family or personal history that suggests bleeding disorders, or has evidence of liver disease. (D) Renal and liver function tests are indicated for patients who have a medical condition or medication use that would serve as indications for the tests.

Prolongation of the inflammatory phase of healing, increased pain, ideal medium for bacterial proliferation describes which complication of neuroma resection? A. vascular embarrassment B. hematoma C. stump neuroma D. digital contracture

Correct answer: (B) Hematoma. Explanation: All the above options are possible complications following neuroma resection. Post-operative formation of digital contraction, in the form of hammertoes, occurs from a biomechanical alteration of the lumbricale pull. Stump neuroma, a true neuroma, occurs from a hypersensitive traumatic nerve ending getting continually irritated from scar tissue, excessive movement and/or pressure. Appropriate proximal resection and/or burying the nerve ending in muscle are techniques used to mitigate this complication. Vascular embarrassment occurs from arterial injury to the nearby nerve. The signs and symptoms listed above are classic for a hematoma. Post operative hematomas result from accumulation of blood and/or serous fluid in the resultant dead space after neuroma resection. This prolongs the inflammatory phase of healing which increases the patient's pain and frustration. Also the collection of blood is an ideal medium and nidus for bacterial growth and infection.

Before choosing a metallic implant for a surgical patient, you review the breakdown process of the proposed biomaterial. Its breakdown is due to an oxidative process that is also known as? A. Creep B. Corrosion C. Ductility D. Passivation

Correct answer: (B) Corrosion. Explanation: Corrosion is known as the chemical reaction of a metal with its environment, resulting in continued degradation. All implanted metallic hardware will have some type of reaction with the surrounding material. The tissue fluid within the human body presents a very aggressive environment for corrosion. The surface-tissue interaction is dynamic rather than static and will develop into new stages as time passes, especially after initial implantation. Corrosion is divided into galvanic, crevice, fretting. There is galvanic corrosion when 2 dissimilar metals are placed in contact within a common electrolytic solution; the material with lower corrosion resistance will start to corrode, for example stainless steel and cobalt-chromium plate. The crevice corrosion happens when there is localized corrosion that occurs in confined spaces, and fretting is when corrosion occurs as a result of repeated relative micro motion between two surfaces. Creep is the process of a solid material to slowly and permanently migrate or deform while mechanical stresses that are below yield strength for the material. Ductility is the ability to withstand plastic deformation without breaking. Ductility is measured as either the percentage of elongation to failure or percentage of reduction in cross-sectional area at failure. Ceramics have little to no ductility. Passivation is production of corrosion resistance by a surface of reaction products. This layer is normally an oxide layer, which is impervious to gas and water.

Which of the following statement regarding digital syndactlization is correct? It is not an effective procedure for correction of a heloma molle. Deforming forces must be neutralized as the syndactilization is inadequate in providing long term correction. Alignment of the adjacent digits is irrelevant in a syndactlization procedure. Dissection for the syndactilization is through subcutaneous tissue.

Correct answer: (B) Deforming forces must be neutralized as the syndactilization is inadequate in providing long term correction. Explanation: It is imperative that for successful correction of the digital deformity the deforming forces must be eliminated. The flail toe must be syndactilized to a stable functioning rectus ad- jacent digit. The Syndactilization itself is not capable of provide any corrective influences. (A) Digital syndactlization has two primary indications. The first is for flail digits. The second is for heloma molle. In fact syndactilization is definitive and curative for heloma molle as once the skin is removed in the dermoplasty there is not a possibility for the soft corn. (C) The position of the adjacent toe that the flail to is being syndactilized to needs to be as close to rectus as possible in order to provide a stable splintage. (D) Syndactylization is performed only at the depth of the dermis.

Which term appropriate describes an orthotic material's ability to withstand repeated cycles of loading or unloading during activity? A. Stiffness B. Durability C. Density D. Corrosion

Correct answer: (B) Durability. Explanation: Durability describes the "fatigue resistance" of a material by its ability to withstand repeated cycles of loading or unloading during activity. (A) Stiffness is bending or compression that occurs when the material is loaded. (C) Density of a material is the weight per unit volume. (D) Corrosion is the amount of chemical degradation to which the material is susceptible.

Out of the choices below what is the most suitable sole fixation device for the fracture pictured below? A. tension band fixation B. dynamic compression plate C. intermedullary screw fixation D. Kirschner wire splintage

Correct answer: (B) Dynamic compression plate. Explanation: The most important aspect of this question is the recognition that the fracture is transverse in nature. This is the indication for dynamic compression plating. If not careful, one might think intermedullary screw fixation might be the best. However, the fracture pictured above is not a Jone's-type fracture. Tension band fixation, although it has its place in fifth metetarsal pathology, it is only helpful in the avulsion fractures of the tuberosity. Kirschner wire fixation would be very difficult given the orientation of the fracture and would lack stability and offer no compression.

A 31-year-old female presents with a complaint of pain of the dorsal distal aspect of the hallux. Radiographs reveal an oval radiolucent area at the distal phalanx with surrounding sclerosis. There is no sign of infection, no sinus tract, and no edema. What is the clinical diagnosis? A. anureysmal bone cyst B. enchondroma C. osteoid osteoma D. osteochondroma

Correct answer: (B) Enchondroma. Explanation: An Echondroma is a benign, hyaline cartilage tumor of intramedullary origin. It is the most common tumor of the phalanges with a higher ratio in the hands to feet (6:1). Radiographically it is presented with a central intrameduallary oval geographic lesion with sharp margination and a thin rim of relative sclerosis. Ostoid osteoma has radiolucent/opaque central nidus and is generally found in the femur, diaphysis of the tibia, and talus. Osteochondroma is pedunculated and has a cartilaginous cap. Aneurysmal bone cyst (ABC ) have an oval blown out appearance with soap bubble trabeculation in its stable phase.

Which IV anesthetic has the potential complication of causing adrenal insufficiency? A. Propofol B. Etomidate C. Ketamine D. Versed

Correct answer: (B) Etomidate. Explanation: The induction dosage of etomidate transiently inhibits enzymes which are involved in cortisol and aldosterone synthesis. Long-term use of etomidate, particularly in the very sick, can lead to adrenal suppression and associated morbidity. Etomidate suppresses corticosteroid synthesis by reversibly inhibiting 11-beta-hydroxylase. Studies have shown that using etomidate for sedation of critically ill trauma patients has been associated with increased mortality secondary to adrenal suppression. Prolonged periods of etomidate infusion can potentiate adrenocortical dysfunction. Etomidate is a rapid, short acting intravenous anesthetic that can be used for conscious sedation as well as rapid induction of general anesthetic. It is commonly used in emergency settings as this agent demonstrates a rapid onset and has shown to have a safe cardiovascular profile which makes it a preferable agent in patients who are hemodynamically unstable as it does not cause a drop in blood pressure.

Which type of polydactyly is characterized by medial digital duplication? A. Postaxial B. Preaxial C. Subaxial D. Inneraxial

Correct answer: (B) Preaxial. Explanation: Polydactyly is a common congenital deformity defined as excess of five digits in humans. It has racial and geographic predilections and may occur as an isolated deformity or less commonly, in association with certain congenitally inherited syndromes. Postaxial polydactyly is the most common hyperdactyly variant and denotes lateral digital duplication. Central polydactyly is the least common and involves duplication of the innermost digits, most often the second toe. Preaxial is applied to medial digital duplication.

Following a Post procedure for a hammertoe deformity, the proximal phalanx remains dorsiflexed at the MPJ. The correct sequence of reduction to resolve the dorsiflexion is: A. MPJ dorsal capsulotomy, plantar plate release, extensor hood recession B. Extensor hood recession, EDL lengthening, MPJ capsulotomy C. MPJ capsulotomy, EDL lengthening, extensor hood recession D. Extensor hood recession, plantar plate release, MPJ capsulotomy

Correct answer: (B) Extensor hood recession, EDL lengthening, MPJ capsulotomy. Explanation: Following a digital Post procedure, or before realigning the digital fusion site, residual dorsiflexion may remain at the MPJ. Frequently, when students are asked what they would do to realign the toe, the answer is "take more bone off the proximal phalanx", which is, of course, wrong. This is a soft tissue problem at the MPJ, and the correct sequence of reduction must be performed. The entire correct sequence is extensor hood recession, EDL lengthening/EDB tenotomy, dorsal MPJ capsulotomy with collateral release, plantar plate release with a McGlamry elevator, Girdlestone-Taylor flexor-to-extensor tendon transfer. Instead of a tendon transfer, some elect to drive a .062" K-wire across the MPJ because it is faster. However, problems associated with a wire across the MPJ include breakage of the wire, MPJ stiffness post wire removal, or recurrence of MPJ dorsiflexion after wire removal because all deforming factors were not addressed. Of note, every step does not have to be performed. After each step, perform a Kelikian push-up test; if the toe realigns after, say, hood recession, EDL lengthening, and capsulotomy, it is not necessary to continue; the toe is satisfactorily aligned. (A) (C) One cannot access the capsule or plantar plate without performing the recession first. (D) The plantar plate cannot be accessed without first performing a capsulotomy.

Baxter's neuritis is an entrapment of which nerve and at which level? Medial plantar nerve at the Porta Pedis First branch of the lateral plantar nerve between the Abductor hallucis and Quadrates plantae muscle Medial calcaneal nerve at the medial plantar aspect of the calcaneus Lateral plantar nerve as it courses between the flexor digitorum brevis and abductor digiti minimi muscles

Correct answer: (B) First branch of the lateral plantar nerve between the Abductor hallucis and Quadrates plantae muscle. Explanation: Baxter's neuritis is entrapment of the first branch of the lateral plantar nerve between the abductor hallucis and quadratus plantae muscle. Morning pain is not common. Instead, pain is typically noted at the end of the day or after prolonged activity. Can have parathesias laterally across the heel. Motor weakness of the abductor digiti minimi muscle might be noted. Pathognomonic sign is pain on palpation of the nerve between abductor hallucis and the quadratus plantae muscles along the medial border of the foot. Nerve conduction studies and EMGs are not diagnostically reliable. Diagnostic injection of the nerve is invaluable.

In cavus foot reconstruction, what is the indication for the peroneus longus transfer? A. Excessive subtalar eversion B. Flexible plantar flexed first ray C. Lateral ankle instability D. Rigid plantar flexed first ray

Correct answer: (B) Flexible plantar flexed first ray. Explanation: The peroneus longus transfer in the arena of cavus foot reconstruction is indicate for the flexible plantar flexed first ray deformity. This deformity is stemming from weakness of the opposing tibialis anterior muscle. The peroneus longs over powers the tibialis anterior and plantar flexes the first ray which contributes to the cavus foot deformity. Transferring the peroneus longus to the dorsal aspect of the foot converts in from a deforming force to a corrective one. (A) The peroneus longus transfer is not indicated for excessive subtalar eversion. The peroneus longus contributes minimally to eversion of the subtalar joint. (C) Transferring the peroneus longus tendon to the dorsum of the foot wouldn't aid in stabilization of the lateral ankle. (D) The transfer of the peroneus longus would have no effect on a rigidly plantar flexed first ray.

A construction worker was working outside on a job site and forgot to wear his steel toe boots. He stepped on a rusty nail that went through his shoe and pierced his foot resulting in a dirty wound. Patient states that he does not think that he had a tetanus shot in 20+ years and does not think he received the initial three vaccines as a child. In addition to x-rays and appropriate antibiotic therapy, which of the following is correct? A. Give patient tetanus toxoid 0.5ml. B. Give patient tetanus toxoid 0.5ml and 250 units tetanus immunoglobulin. C. Give patient nothing as you do not treat until symptoms begin. D. Give patient immunoglobulin 250 units alone.

Correct answer: (B) Give patient tetanus toxoid 0.5ml and 250 units tetanus immunoglobulin. Explanation: Tetanus status is an important thing to consider when seeing a patient with a puncture wound. In this case, the wound was dirty since it was a rusty nail that went through his boot. Patient also has not had a booster in 20+ years and status of 3 step vaccinations was unknown as a child. If the patient's tetanus vaccination status is unknown, and the wound is clean you only need to give 0.5ml toxoid. If the status is unknown and the wound is dirty, you should give toxoid and immunoglobulin. If the patient has had three initial injections and the wound is clean, you only have to give toxoid if it has been greater than 10 years since previous booster. If the patient has had three initial injections and the wound is dirty, you only have to give toxoid if the patient has not had a booster in > 5 years. If the wound is neglected > 24 hours you should always give both toxoid and immunoglobulin.

A patient has hallux rigidus deformity with 30 degrees of dorsiflexion, dorsal osteophytes visible radiographically, and mild pain on range of motion. According to the Coughlin and Shurnas classification, what is the grade of the deformity? A. Grade 0 B. Grade 1 C. Grade 2 D. Grade 3 E. Grade 4

Correct answer: (B) Grade 1. Explanation: The Coughlin and Shurnas classification of hallux rigidus combines clinical and radiographic features. Grade 0 has 40-60 degrees dorsiflexion, normal radiographs, no pain, and some stiffness. Grade 1 has 30-40 degrees dorsiflexion, dorsal osteophytes on radiograph, and mild pain on motion. Grade 2 has 10-30 degrees dorsiflexion, joint narrowing on radiograph, and moderate to severe pain near end range of motion. Grades 3 and 4 have 10 degrees or less dorsiflexion, the same radiographic findings as Grade 2 plus subchondral cystic changes, and constant pain at end range of motion (Grade 3) or at midrange of motion (Grade 4).

Bony sclerosis described as an "ivory phalanx" is most commonly associated with which arthritic disease process? A. Osteoarthritis B. Psoriatic arthritis C. Rheumatoid arthritis D. Septic arthritis

Correct answer: (B) Psoriatic arthritis. Explanation: One characteristic feature of psoriatic arthritis in the foot is the "ivory phalanx," which classically involves the distal phalanges,especially in the hallux, with sclerosis, enthesitis, periostitis and soft-tissue swelling.

Which over the following are the correct combinations of deformities exhibited in a patient with talipes equinovarus? A. Hindfoot calcaneus, Hindfoot varus, forefoot adduction B. Hindfoot equinus, hindfoot varus, forefoot adduction C. Hindfoot equinus, hindfoot varus, forefoot abduction D. Hindfoot calcaneus, hindfoot valgus, forefoot abduction

Correct answer: (B) Hindfoot equinus, hindfoot varus, forefoot adduction. Explanation: Hindfoot equinus, hindfoot varus, forefoot adduction. Clinical examination of an infant with talipes equinovarus (TEV), also known as clubfoot, reveals hindfoot equinus, hindfoot varus, forefoot adduction, and often cavus forefoot with varying degrees of rigidity. These are often the secondary result of medial displacement of the navicular on the talar head. Other pathoanatomical abnormalities seen in TEV include: lateral rotation of the talus in the ankle mortise, medial and plantar deviation of the talar neck, medial subluxation of the cuboid on the calcaneus.

The following is essential and central to patient's ability to generate a surgical stress response: A. Increase vascular response B. Hypothalamic-pituitary-adrenal axis C. Renal angiostensin system D. Blood glucose less than 200 mg/dL

Correct answer: (B) Hypothalamic-pituitary-adrenal axis. Explanation: The hypothalamic-pituitary-adrenal axis is central to patient's ability to generate a surgical stress response. A defect anywhere in this cycle has dramatic consequences in the perioperative period. Tuberculosis used to be the main cause of primary adrenal insufficiency, but autoimmune adrenalitis is now the most common cause. Other causes of primary adrenal insufficiency include infections, adrenectomy, and sepsis.

A neuropathic patient presents with hallux malleus and recurrent ulcerations at the dorsal interphalangeal joint. On examination, the first ray does not appear plantarflexed. Which is an appropriate surgical intervention to prevent further ulceration at this site? A. Jones tenosuspension B. IPJ arthrodesis with Jones tenosuspension C. Hibbs tenosuspension D. STATT

Correct answer: (B) IPJ fusion with Jones tenosuspension. Explanation: The Jones tenosuspension is a transfer of EHL from its insertion, through the neck of the first metatarsal and re-approximated back on itself. The procedure is beneficial for correction of hallux malleus or clawed hallux. The soft tissue procedure is commonly performed in conjunction with IPJ arthrodesis to stabilize the hallux and prevent future mallet toe. (A) Although Jones tenosuspension is an appropriate procedure for correction of hallux malleus, it is most effective in conjunction with hallux IPJ fusion in the case of recurrent ulcerations. Without addressing the IPJ, a mallet toe may result due to the pull of FHL. (C) Hibbs tenosuspension is used to correct digital contracture from overpowering EDL. The EDL is transferred into the peroneus tertius or lateral cuneiform, and should be used in conjunction with PIPJ arthrodesis if the contractures are semireducible or rigid. This procedure does not address the hallux IPJ deformity. (D) Split tibialis anterior tendon transfer (STATT) is commonly utilized to correct flexible equinovarus deformity. This would not correct the hallux IPJ deformity causing the chronic ulcerations.

Please refer to the clinical photograph below. Which of the following statement regarding digital syndactilization is correct? It is not an effective procedure for correction of a heloma molle. If deforming forces of a flail toe are not corrected then the syndactilization itself is inadequate to provide long term correction. Alignment of the adjacent digits are not important in a syndactilization procedure. Dissection is through subcutaneous tissue.

Correct answer: (B) If deforming forces of a flail toe are not corrected then the syndactilization itself is inadequate to provide long term correction. Explanation: It is imperative that for successful correction the deforming forces be eliminated from the flail and that the adjacent toe be a stable functioning rectus digit. Rather than the syndactlized toe stabilizing the adjacent toe from the deforming force, it is more likely that the two toes would now deform together. Syndactyly is the side-to-side anastomosis of two neighboring toes by resection dermoplasty. It is indicated for flail digits and heloma molle. It is an extremely effective procedure and is performed only at the depth of the dermis.

All of the following statements about bone wax listed below are correct except which? A. It may develop a foreign body granuloma. B. It promotes bone healing. C. It is contraindicated in individuals with an allergy to bees. D. It provides hemostasis by mechanical tamponade.

Correct answer: (B) It promotes bone healing. Explanation: Bone wax is a very common commercially available product that provides a mechanical tamponade of the bleeding vessels following a bone procedure and has the benefit of low cost, easy preparation and handling. However, bone wax has been shown to inhibit bone healing and may develop foreign body granuloma. Additionally it is contraindicated in the presence of an allergy to bees. As it inhibits bone healing the notion of promoting bone healing is inaccurate.

Which of the following skin closure techniques is not for circular defects in the skin? A. O-T closure B. M-Plasty C. O-Z closure D. Mercedes Benz closure

Correct answer: (B) M-Plasty. Explanation: Whenever possible an elliptical excision is best utilized for lesion excision. However, in some instances the excision of the lesion results in a circular defect. Constraints that my cause this include anatomic constraints, relaxed skin tension lines, lines of maximum extensibility and line of minimum movement. The M-Plasty procedure is used when a full length fusiform excision is not possible due to an important anatomic or cosmetic line. By converting one or both of the fusiform ends to an M-Plasty, tissues can be preserved and the extension line is reduced. The O-T Closure, O-Z closure, and Mercedes Benz closure are all techniques used to close a circular skin defect. The O-T closure consists of making horizontal incisions on opposing sides of the base of the defect. The flaps are dissected in the subcutaneous plane and are directed towards the circular defect with the final closure assuming the shape of a T. The O-Z flap, also known as a double rotational flap, disperses the line of tension along three lines (rather than just one line in an elliptical closure). Two arc-like incisions are made on opposite sides whose radius contains the defect- whose sides are subsequently approximated giving rise to the characteristic Z shape. During a Mercedes Benz closure a three-point anchoring suture is placed after advancing the surrounding tissue toward the center, creating a tripod closure.

Which of the following is the most common complication of correcting a clubfoot deformity by serial casting? A. Calcaneal gait B. Rocker-bottom deformity C. Hallux abductovalgus D. Peroneal spasm

Correct answer: (B) Rocker-bottom deformity. Explanation: Dorsiflexing the foot prematurely causes stress at the midfoot which causes a rocker bottom deformity. If equinus is corrected prior to the cavus, adductus, or varus deformities, a rocker-bottom deformity may occur. Therefore it is suggested that the equinus deformity be corrected last as not to cause excessive dorsiflexion of the forefoot.

The most common intraoperative complication seen when performing a Lapidus fusion is: A. First metatarsal base fracture B. Multiple broken fragments of the resected surfaces C. Infection D. Non-union

Correct answer: (B) Multiple broken fragments of the resected surfaces. Explanation: Pieces of the resected surfaces, especially the lateral/plantar aspect of the joint which seems to be the most difficult portion to remove due to soft tissue attachments, is the most common intraoperative complication. Difficulty also arises due to the shape of the articulation and depth of the bone, especially the first cuneiform side. (A) A fist metatarsal fracture can indeed occur; this is usually due to placement of the screws too close to the fusion site. Taking too much of the lateral condyle of the first metatarsal can also contribute to fracturing. (C) Infection is a post-operative complication, not intra-operative. The infection rate is no greater for this procedure, as compared to other forefoot procedures. (D) Non-union is also a post-operative complication. Chang states non-union is rare, is more of a radiographic finding than clinical, and feels these are usually not symptomatic clinically.

Which of the following is true regarding the use of Ropivacaine? A. Cost is comparable to bupivacaine. B. Myocardial toxicity is less than that of bupivacaine. C. Duration of anesthesia is longer than that of bupivacaine. D. Potency is greater than that of bupivacaine for sensory blockade.

Correct answer: (B) Myocardial toxicity is less than that of bupivacaine. Explanation: A disadvantage is the substantially increased cost of ropivacaine over readily available and inexpensive agents. The main advantage of ropivacaine is a more favorable toxicity profile than bupivacaine. The direct myocardial toxicity is substantially less than that of bupivacaine. The duration of anesthesia and analgesia is comparable to bupivacaine's and has been reported to last 16 to 19 hours. Compared with bupivacaine, ropivacaine has an anesthetic potency that is clinically equivalent with respect to sensory anesthesia but that is slightly less potent with respect to motor block when used in lower concentrations.

In fixing fractures affecting the cortical bone, lag screws must be protected with a plate or a plate equivalent. This plate is referred to as the following: A. Lag plate B. Neutralization plate C. Static tension plate D. Tension plate

Correct answer: (B) Neutralization plate. Explanation: The lag effect using screw is adequate as the only fixation for some metaphyseal or epiphyseal fractures. Particularly in cortical bone, lag screws must be protected with a plate or plate equivalent, such as external fixator. The protective device is referred to as neutralization device/plate because it controls the bending, translational, and torsional forces.

This accessory bone is situated superiorly between the first and second metatarsal bases. It is best seen in the dorsoplantar view; it is typically superimposed on the 1st metatarsal base: A. Os interphalageus B. Os intermetatarseum C. Os vesalianum D. Os supratalare E. Os peroneum

Correct answer: (B) Os intermetatarseum. Explanation: Situated superiorly between the first and second metatarsal bases, it is best seen in the dorsalplantar view. It can be seen in the lateral view superiorly, although it typically is superimposed on the first metatarsal base. This ossicle may be round, oval, kidney shaped, or linear, or may even resemble a rudimentary metatarsal. Its size also varies. It may articulate with the medial cuneiform or be attached to the first metatarsal base. Calcification of the perforating branch between the dorsal and plantar metatarsal arteries may simulate an os intermetatarseum. Occurs in 1% to 10% of the population.

A 52 year old male presents complain of severely painful callus under the right first metatarsal head. He states has been present for over fifteen years. He has tried numerous over the counter and custom orthotics, shaving the lesion, emollients. He is very frustrated and wants a definitive procedure performed. Physical exam show intact neuromuscular status on the right foot. A flexible severely plantar flexed first metatarsal deformity is noted. Weight bearing exam shows a varus hindfoot. Coleman Block test results show neutralization of the hindfoot varus when the first ray is unloaded. A clinical picture is provided below. Which of the following procedures would be best indicated? A. Jones tenosuspension B. Peroneus longus to peroneus brevis transfer C. Lapidus procedure D. Tibial sesamoidectomy

Correct answer: (B) Peroneus longus to peroneus brevis transfer. Explanation: The condition described above is much more a functional plantar flexed first ray rather than a structural plantar flexed first ray. In other words from an osseous standpoint there is nothing abnormal or pathological. The problem lies in an overactive peroneus longus. In instances such as this, the peroneus longus to peroneus brevis transfer is indicated. In this procedure the longus tendon is transected at the level of the cuboid and tenodesed in the brevis tendon. The distal remnant of the longus tendon is tenodesed into the periosteum and/or bone of the cuboid. Indications for the procedure include overload beneath the first metatarsal with painful callus, peroneus brevis weakness or tear, or replacement for a peroneus brevis tendon that's been inappropriately used in a lateral ankle stabilization.

Concerning the depth of anesthesia, stage 3 is further subdivided into 4 planes. Which planes is best described by the following characteristics? - corneal reflex disappears - changes in breathing - exhalation time becomes greater than inhalation time - beginning of intercostal paralysis - secretion of tears - loss of laryngeal reflex A. Plane 1 B. Plane 2 C. Plane 3 D. Plane 4

Correct answer: (B) Plane 2. Explanation: When discussing the depth of anesthesia there four separated stages. Stage 1 is amnesia/analgesia, stage 2 is delirium/excitement, stage 3 is surgical anesthesia and stage 4 is overdosage. Stage 3 lasts from the onset of regular breathing to respiratory arrest. Stage 3 is further subdivided into four planes. Plane 2 is most notably characterized by a change in the patient's breathing pattern. The patient starts to have longer durations of exhalation than those of inhalation due to the initiation of the intercostal paralysis in the latter portion of the plane. Also of note is that the once active ocular muscle in plane 1, now have become inactive. This results in the eyes becoming immobile. The secretion of tears increases. The laryngeal reflex is lost.

What clinical entity is shown in the MR image below? A. Baxter's neuritis B. Plantar fasciitis C. Achilles tendinosis D. Posterior facet arthritis

Correct answer: (B) Plantar fasciitis. Explanation: On MR imaging, plantar fasciitis is manifested by increased signal in both T1- and T2-weighted sequences and thickening of the fascia up to 6 to 10 mm with the greatest involvement closest to the calcaneal attachment. Inflammatory change in the surrounding subcutaneous tissue can also be present on MR imaging and is manifested by diminished and increased signal intensity on T1- and T2-weighted sequences, respectively.

23 year old construction worker presents to the ED after a boulder fell on his left foot damaging his steel toe boot. He denies any other trauma. The patients socks and shoes were removed before arrival to the fracture clinic. Past medical history: Unremarkable Past surgical History: None Social history: Non -smoker, no illicit drug abuse, no alcohol abuse You proceed with the physical exam. The following are pertinent findings which you observed: - No open wounds - Ecchymosis and swelling to the dorsum of the left foot - There is no pallor, or cyanosis - Gross sensations all intact to the left foot - Gentle palpation reveals tenderness and crepitus to the 2nd metatarsal What will be your next step? A. Physical exam B. Radiographic examination C. Casting D. Schedule open reduction internal fixation

Correct answer: (B) Radiographic examination. Explanation: After a complete history and thorough physical examination, x-rays of the injured foot, taken in the appropriate projections, enable the clinician to define the extent of osseous injury present. This will help the clinician to define the extent of osseous injury. (A) After adequate history, the injured part should be examined. This is the first step. It is most helpful to compare the injured part to the opposite side to identify any deviation from normal in a particular patient. Inspection will identify ecchymosis and swelling as nonspecific signs of injury, wounds, indicating possible open fracture, dislocation, pallor or cyanosis indicative of impairment of circulation, and signs of preexisting disease or deformity. Gentle palpation may reveal crepitus, eliciting discrete area of pint tenderness. (C) This is part of the treatment plan after thorough physical exam and testing. (D) This is part of the treatment, not applicable yet for this patient.

Mechanical control of the pathologically pronated foot is essential for effective treatment. A stiff shanked shoe is essential not only for improved control of the pronated foot, but also effective orthotic and ankle foot orthotic therapy. What is another quality that is important for a shoe used for treatment of an adult-acquired flatfoot? A. Wide toe box B. Reinforced heel counters C. High-top style shoe D. Lace shoe

Correct answer: (B) Reinforced heel counters. Explanation: Shoe with stiff shanks and reinforced heel counters are an essential element in the treatment of an unstable flatfoot. These elements are essential in as shoe not only for the flatfoot by itself, but also with orthotic and ankle foot orthotic therapy. (A) A wide toe box is inconsequential in regards to the elements of a shoe for an unstable flatfoot. (C) While a high top shoe is not a bad idea with an unstable flatfoot as it captures some control of the ankle, it is not considered and essential element in a shoe for an unstable flatfoot. (D) An effective shoe for an unstable flatfoot can be lace, velcro or an alternative closure.

Which of the diseases is considered a "true" avascular necrosis? A. Blount's (proximal-medial tibial epiphysis) B. Renandier's (tibial sesamoid) C. Sever's (posterior calcaneus) D. Osgood Schlatter's (tibial tubercle)

Correct answer: (B) Renader's (tibial sesamoid). Explanation: Avascular necrosis or osteonecrosis is defined as bone death due to lack of blood flow. This interruption in blood flow can be due to arterial damage/injury or venous obstruction. In order to be considered a "true" AVN, cell death needs to be evident. When blood flow is compromised to the epiphysis of a long bone, the bone will go through several stages in its healing process ultimately resulting in altered morphology. In the initial "avascular phase", radiographic signs are minor. Typically the joint will distend due to inflammation and effusion, however there are minimal bone density changes as the necrotic bone remains the same density as the viable bone. It isn't until the second stage, "revascular phase", when osseous density changes become evident. With hyperemia to the area, the healthy bone will appear washed out or of decreased density compared to the dead bone. Subchondral fractures may occur causing the bone collapse. During the third phase, "repair and remodeling", the area around the necrotic tissue begins to be resorbed and new bone is laid down. This new bone causes an actual increase in density, making the necrotic bone area bright white. "Residual deformity" is the final stage and it is reached when adaptive changes are noted at the articular surface on the opposite side of the joint. Of the diseases listed above, the only "true" AVN is Renandier's or osteonecrosis of the tibial sesamoid. With this disease, bone death occurs and causes mottling/density changes, fragmentation and abnormal size/shape. All the other diseases cause transient fragmentation and changes in bone density due to variations in endochondral ossification.

A patient undergoes right ankle ORIF and is placed in a below-knee cast. Three days post- operatively, the patient complains of calf pain, tenderness and redness. On examination, there is concern for a deep vein thrombosis. In deciding an appropriate pharmacologic course, which would be a contraindication for using low molecular weight heparin (LMWH)? A. Cirrhosis B. Renal failure C. Peptic ulcer D. Stroke 12 months prior

Correct answer: (B) Renal failure. Explanation: Low molecular weight heparin (LMWH) is often used in the treatment of DVTs. LMWH is renally excreted and therefore should not be used in patients with renal failure. (A) The use of LMWH in patients with cirrhosis appears to be safe. (C) Peptic ulcers are often a reason for concern when using aspirin as DVT prophylaxis. Aspirin may cause GI irritation and bleeding. (D) Stroke within the past 6 months prior to surgery may be a relative contraindication, however 12 months prior to surgery is not an absolute contraindication for use of LMWH.

A 31 year old female patient presents to your clinic with a chief complaint of generalized pain. On future questioning it appears that she has pain in multiple joints in the body including the hands and feet. She states that she has also started seeing nodules under her skin. X-rays were taken and showed erosive changes with uniformed joint space narrowing. She was also complaining that after she gets out of bed in the AM, it takes her a while to get moving and she feels stiff. What is the most likely diagnosis? A. Osteoarthritis B. Rheumatoid Arthritis C. Paget's Disease D. Scurvy

Correct answer: (B) Rheumatoid Arthritis. Explanation: The correct answer in this question is B. A patient with rheumatoid arthritis has x- ray changes consistent with erosions as well as uniformed narrowing of the joints particularly in the hands and feet (small joints of the body). It is also common for these people to complain of morning stiffness. Rheumatoid nodules are also commonly seen in these patients. Osteoarthritis shows non-uniform joint space narrowing and generally would not show morning stiffness. Paget's disease is a disorder of bone that shows bone breakdown and then remodeling forming poorly oriented bone. Scurvy is a deficiency in vitamin C. On x-ray typically you would see "Wimberger's ring sign" which is when the epiphysis appears as an outer shell of increased density surrounding a central lucency.

What is the name of the genetically mutated receptor on the sarcoplasmic reticulum in a skeletal muscle cell that that is responsible for malignant hyperthermia? A. acetylcholine receptor B. ryanodine receptor C. inositol triphosphate receptor D. adrenergic receptor

Correct answer: (B) Ryanodine receptor. Explanation: Malignant hyperthermia is a genetic condition involving the ryanodine receptor in skeletal muscle sarcoplasmic reticulum. The sarcoplasmic reticulum is the calcium-storing organelle within the skeletal muscle cell. Furthermore, the ryanodine receptor modulates the calcium release from the sarcoplasmic reticulum. When an individual with a genetic predisposition for malignant hyperthermia is exposed to a trigger such a volatile inhalation anesthetics or depolarizing muscle relaxants, a massive intracellular release of calcium into the skeletal muscle occurs. This causes severe muscle contractions and cellular destruction. Inositol triphosphate and adrenergic receptors both occupy smooth muscle, and although acetylcholine is a receptor on skeletal muscle, it is not associated with the sarcoplasmic reticulum and/or malignant hyperthermia.

A 68 year old diabetic patient presents to your office for a surgical consult for elective neuroma excision. On examination, patient does not have palpable pulses and capillary refill time is sluggish to the toes. ABI completed in the office is 0.55. Which of the following is the best option moving forward? A. Schedule neuroma surgery- ABI of 0.55 is fine for healing B. Send patient to vascular surgeon for evaluation C. Schedule neuroma surgery and have patient take aspirin before to increase blood flow D. ABI has no effect on healing potential

Correct answer: (B) Send patient to vascular surgeon for evaluation. Explanation: The vascular examination is very important before moving forward with surgery. A patient without adequate blood flow to the lower extremities may have difficulty healing surgical incisions. The ABI (ankle brachial index) is measured as ankle pressure/ brachial pressure. Values of greater than 1.4 are abnormal and indicate calcification of the vessels. Values of 1-1.4 are considered normal, 0.9-1.0 is acceptable, 0.8-0.9 is mild vascular disease, 0.5-0.8 moderate vascular disease and <0.5 is severe vascular disease. In this question, the ABI is 0.55 and thus the patient should not be scheduled for surgery at the current time but instead should be sent to a vascular surgeon for evaluation.

A 64 year old male patient is referred to a physical therapists office for a chief complaint of inflammation in the left shoulder. The patient is generally healthy other than a history of a myocardial infarction 5 years ago. Patient now has a pacemaker. Which of the following would be the least favorable choice for rehabilitation of this specific case? A. Moist Heat Packs B. Short Wave Diathermy C. Whirlpool D. Paraffin Wax

Correct answer: (B) Short Wave Diathermy. Explanation: The key factor that needs to be picked up on in this case would be the importance of seeing that the patient has a pacemaker. Short wave diathermy uses high frequency current to heat tissues using electromagnetic energy. It is thus contraindicated in a patient with a pacemaker as it creates an electromagnetic field. There would be no contraindication to the use of moist heat packs in this case, whirlpool (although may be difficult to rehabilitate a shoulder in the whirlpool) or paraffin wax.

The best surgical option utilizing the tibialis posterior tendon in the treatment of equinovarus in the CP patient is: A. Tibialis posterior tenotomy B. Split tibialis posterior tendon transfer (SPOTT) C. Transfer through the interosseous membrane to the foot dorsum D. Transpose anterior to the medial malleolus

Correct answer: (B) Split tibialis posterior tendon transfer (SPOTT). Explanation: A complete detachment of the tendon without transfer to effect re-balancing is doomed to failure, and will create a new deformity, especially if its antagonist, the peroneus brevis, is functional. Transferring the tibialis posterior tendon through the interosseous membrane is an out-of-phase transfer; although many state the transferred tendon can be "re- educated", this is more likely true for lower motor neuron lesion diseases. Tendons cannot be re- educated in upper motor neuron lesion diseases like CP; long-term results are poor. Transferring the tendon anterior to the medial malleolus has simply fallen out of favor due to its unpredictability. Split tendon transfers have worked better than transfer of the entire tendon. Split transfers allow better re-balancing of the foot. In this transfer, the tibialis posterior tendon is split longitudinally to the musculotendinous junction and the medial half transferred to the peroneus brevis on the lateral side of the foot. This works well in spastic hemiplegia.

A 28-year old male patient sees you regarding a painful "growth" on their toe (see images below). He states he dropped a large boulder on this toe over a year ago and this lesion has been slowly growing since that time. A radiograph shows cancellous bone growing from the base of the distal phalanx with no destructive changes. You suspect: A. Osteochondroma B. Subungual exostosis C. Osteosarcoma D. Chondrosarcoma E. Bone fragment

Correct answer: (B) Subungual Exostosis. Explanation: Subungual exostosis is a bony growth most commonly found on the hallux. It is most common in young adults and is often associated with a history of trauma or infection. Radiographically they appear as cancellous bone growing from the base off the distal phalanx with no underlying destructive changes. Subungual osteochondromas, on the other hand, are more common in adolescents and radiographically appear as confluent with that of the underlying bone.

A 21 year old baseball catcher presents to the clinic complaining of pain to the back of his left ankle. States it has been present and gradually getting worse over the last 3 weeks. He states that when he squats low in his catchers stance the pain increases greatly. Also, he reports that he notices increased pain when going down stairs and sometimes it feels as if it is going to give out on the stairs. Denies any known trauma. Physical exam reveal tenderness on palpation to the posterior lateral ankle joint. The pain is reproduced when the foot is plantar flexed and an axial load is applied to the heel. Active plantarflexion and dorsiflexion of the hallux causes pain. What is the diagnosis? A. Fracture of the lateral talar process B. Symptomatic os trigonum C. Stenosing flexor hallucis longus tenosynovitis D. Non-insertional Achilles tendinopathy

Correct answer: (B) Symptomatic os trigonum. Explanation: The diagnosis in the scenario above is that of a symptomatic os trigonum. The patient with a symptomatic os trigonum typically presents with pain in the posterior lateral aspect of the ankle. The symptoms are accentuated by resisted plantar flexion or dorsiflexion of the hallux. This can be explained by movement of the adjacent flexor hallucis longus tendon against the ossicle. Likewise pain can be elicited with forced plantar flexion of the foot, such as squatting and walking downhill. The patient may complain of pain and mild swelling when running, jumping or descending stairs, and symptoms of giving way may be present. On physical examination, in addition to the tenderness at the posterior lateral aspect of the ankle, plantar flexion of the foot with an axial load to the heel can reproduce symptoms.

What is one of the most frequently encountered congenital conditions affecting the foot, and most commonly affecting the 2nd and 3rd digits? A. Macrodactyly B. Syndactyly C. Brachymetatarsia D. Ectrodactyly

Correct answer: (B) Syndactyly. Explanation: Syndactyly is the most frequently encountered congenital conditions affecting the foot, and most commonly affecting the 2nd and 3rd digits. Syndactyly is defined as congenital or acquired webbing between adjacent digits from birth or secondary to injury. Most investigators agree that the deformity is the result of rapid arrest of embryological development during the 6th to 8th week of intrauterine life. The skin between the 2nd and 3rd digits is the last to develop which may explain the predication of these digits being affected. It is nearly ten times more common in Caucasians than African Americans, with males and females being similar affected. (A) Macrodactyly is not the most frequently encountered congenital conditions affecting the foot, and most commonly affecting the 2nd and 3rd digits. It is an abnormally hypertrophied digit. This congenital deformity often presents unilaterally, rarely seen in the fourth and fifth digits, and is more commonly seen in males. Its exact cause is unknown. It can involve enlargement of the bones, skin, nerves, blood vessels, and nails. (C) Brachymetatarsia is not the most frequently encountered congenital conditions affecting the foot, and most commonly affecting the 2nd and 3rd digits. Brachymetarsia results from premature closure of a growth plate of a metatarsal. The reason for this premature closure of the epiphyseal plate is not known. It typically occurs at the fourth metatarsal most often and typical presents bilaterally. When more than one metatarsal is involved it is termed brachymetapody. The condition is largely hereditary but can be caused surgically, idiopathically, developementally, or traumatically. It is associated with a number of disorders such as Down's Syndrome, Albright hereditary osteodystrophy, and Turner syndrome to name a few. (D) Ectrodactyly is not the most frequently encountered congenital conditions affecting the foot, and most commonly affecting the 2nd and 3rd digits. Ectrodactyly is congenital absence of two or more central rays. There are more than 175 terms used to describe the skeletal dysplasia describing ectrodactyly including "cleft foot" and "lobster foot." The tarsal bones are also typical deformed with this condition. The condition is usually present along with "lobster clawing" of the hand and other deformities such as cleft lip and cleft palate.

This ligament primarily limits inversion of the subtalar joint: A. The interosseous ligament. B. The cervical ligament. C. The anterior talofibular ligament. D. The posterior talofibular ligament. E. The spring ligament.

Correct answer: (B) The cervical ligament. Explanation: The lateral talocalcaneal ligament or cervical ligament runs from the lateral tubercle of the talus obliquely, obliquely, inferiorly, posteriorly, and parallel to the intermediate band of the lateral collateral ligament of the ankle joint and inserts on the lateral surface of the calcaneus. This ligament, along with the inferior extensor ligament limits inversion of the subtalar joint.

The extensor brevis tendons and extensor digitorum longus tendons form this important apparatus at the level of the MTPJ? A. The expansion hood apparatus B. The extensor hood apparatus C. The extension expansion D. The expansion-extension hood apparatus

Correct answer: (B) The extensor hood apparatus. Explanation: The extensor hood apparatus is an extremely important and unique anatomical structure. At the level of the MTPJ, its medial and lateral fibers unite plantarly with the capsule, deep transverse metatarsal ligament, and the flexor tendon sheath, thereby functionally inserting into the MTPJ. The extensor hood is composed of 2 parts that together cover the metatarsophalangeal joint and extend to the proximal interphalangeal joint. The extensor expansion is sometimes referred to the extensor hood or the extensor hood apparatus. The extensor expansion may be absent at the 1st metatarsophalangeal joint.

As a preference you decide to use epinephrine for hemostasis instead of an ankle tourniquet. Which of the following is true concerning the use of epinephrine as a hemostatic agent? Epinephrine decreases bleeding but also increases the rate of absorption of the anesthetic. The most common side effect is transient tachycardia after inadvertent intravascular injection. Circulation is completely occluded when using epinephrine in the involved digits. Safe to use in peripheral vascular disease, the dose cannot exceed 1:100,000.

Correct answer: (B) The most common side effect is transient tachycardia after inadvertent intravascular injection. Explanation: Epinephrine shortens the onset of anesthesia, prolongs the effect, and produces vasoconstriction. It not only decreases bleeding but also slows the rate of absorption of anesthetic. This allows more time to metabolize the anesthetic and further prolongs its action. The most common side effect is transient tachycardia after inadvertent intravascular injection. (A) In fact epinephrine decreases bleeding with vasoconstriction, but also slows the rate of absorption of anesthetic, not increasing the rate of absorption like stated in the question. (C) It has not been proven that epinephrine will completely occlude circulation in the involved digits. (D) It is not safe to use in peripheral vascular disease, as it is a contraindication in this patient population.

Regarding distal osteotomies of the fifth metatarsal for bunionette deformity, which is the most important advantage? A. They provided minimal intermetatarsal angular correction. B. They preserve the function of the fifth metatarsophalangeal joint. C. They can result in transfer lesions if the capital fragment dorsiflexes. D. They can shorten the metatarsal. E. They can narrow the forefoot.

Correct answer: (B) They preserve the function of the fifth metatarsophalangeal joint. Explanation: Distal osteotomies preserve the function of the fifth metatarsophalangeal joint. This is their primary advantage. (A) This is a disadvantage of the procedure. (C) This is a potential disadvantage of the procedure. (D) This is a potential disadvantage. (E) Although they narrow the forefoot to an extent, this advantage is minimal compared to the narrowing obtained in proximal osteotomies, which also more effectively reduce the intermetatarsal angle.

A 43 year old female presents with a complaint of pain in the ball of her left foot. Pain on palpation is noted in both the second and third interspaces of the left foot. A diagnostic block of 1 cc 1% lidocaine without epinephrine's administered to the left third interspace. On exam after the diagnostic block you note that the pain to palpation is now absent from both the second and third interspaces. What should the surgical plan be? A. second interspace neurectomy B. third interspace neurectomy C. second and third interspace neurectomy D. distal tarsal tunnel release

Correct answer: (B) Third interspace neurectomy. Explanation: The key to this question is the realization that pain from one interspace can cause the patient to feel similar sensations in the adjacent interspaces. For this reason, diagnostic local anesthetic blocks are invaluable in a metatarsalgia work up. The patient above has pain in both interspaces relieved with only injection of the third interspace, which in turn confirms that the pathology is in the third interspace. With this fact confirmed, only the third interspace would therefore be addressed surgically. If the isolated third intermetatarsal block would not have relieved pain in both interspaces, then a more proximal diagnosis would need to be explored.

A palpable posterior tibial pulse is associated with ___% healing of digital amputations? A. 60 B. 70 C. 80 D. 90

Correct answer: (D) 90. Explanation: Although absence of palpable pedal pulse does not serve as an indicator of healing potential, it has been found that a palpable posterior pedal pulse is associated with successful healing of digital amputations at a rate of 90%. (A) 60% healing rate with palpable posterior tibial pulses is too low. (B) 70% healing rate with palpable posterior tibial pulses is too low. (C) 80% healing rate with palpable posterior tibial pulses is too low.

Upon dorsiflexion of the metatarsophalangeal joints, the plantar fascia is pulled distally around the metatarsal heads increasing the arch height of the foot. The name for this is: A. Elftman's Theory B. Windlass Effect C. Coleman Effect D. Kelikian Test

Correct answer: (B) Windlass Effect. Explanation: Due to the strong attachments of the plantar aponeurosis to the distal aspects of the plantar pads of the metatarsophalangeal joints, the rearfoot will supinate as the pull of the aponeurosis causes the distance between the calcaneus and metatarsal heads to shorten. The supination of the rearfoot, along with intrinsic muscle (those originating on the medial aspect of the calcaneus) contraction aids in the supination of the oblique axis of the midtarsal joint during propulsion. Furthermore, the foot has commonly been described as acting much like a beam and truss. The plantar aponeurosis, acting like the truss, relieves the tensile forces when placed under tension so that the skeleton only experiences compressive forces. Vertical loading and forward motion of the leg increase the tension on the aponeurosis and initiate the truss phenomenon.

What is the toxic dose of bupivacaine with epinephrine for a 56 kg female? A. 106 mg B. 174 mg C. 196 mg D. 225 mg

Correct answer: (C) 196 mg. Explanation: Complications from local anesthetic typically occur from toxicity or allergy. Allergy is relatively rare. Toxicity is seen when an excessive dose based on mg/kg of body weight is given, accidental intravascular injection or rapid absorption from a highly vascular tissue. The traditional memorized toxic dose for bupivacaine with epinephrine is 225 mg. The critical problem with this is that the 225 mg number is based on a 70 kg individual. Our female in the case above is 56 kg. The toxic dose of bupivacaine is determined by a ratio 3.5mg/kg. So the 56 kg female would have a toxic dose of 196 mg. 56 x 3.5 = 196.

What is the average range of motion of the subtalar joint in normal gait? A. 15 to 30 degrees, 2/3 inversion, and 1/3 eversion B. 15 to 30 degrees, 2/3 eversion, and 1/3 inversion C. 25 to 30 degrees, 2/3 inversion, and 1/3 eversion D. 25 to 30 degrees, 2/3 eversion and 1/3 inversion

Correct answer: (C) 25 to 30 degrees, 2/3 inversion, and 1/3 eversion. Explanation: The reported normal range of motion of the subtalar joint is highly variable, reported values of inversion and eversion range from 5 to 50 degrees. Reported values of eversion range from 5 to 26 degrees. The average subtalar joint range of motion has been reported as 25 degrees (20 degrees inversion and 5 degrees eversion). Root et al report an average range of subtalar motion to be 30 degrees with two thirds (20 degrees) in the direction of inversion and one third (10 degrees) in the direction of eversion. Wright et al. found that throughout the stance phase of gait, the average excursion of the subtalar joint was only 6 degrees and labeled this the "functional range" of motion. Root et al. state that normal locomotion requires an average minimum range of 4 to 6 degrees of inversion with supination and 4 to 6 degrees of eversion with pronation. Although there is some disagreement in regard to how much subtalar joint motion is required for normal locomotion, it appears that considerably less motion is actually required for normal function than is available in the average joint.

A 42 year old male patient underwent neuroma excision in the left foot 3rd interspace. One year after surgery, he noticed his fourth toe begin to deviate. Which of the following may have been damaged during the surgery leading to this toe deviation? A. 1st Lumbrical B. 5th dorsal interossei C. 3rd Lumbrical D. 3rd plantar interossei

Correct answer: (C) 3rd Lumbrical. Explanation: During a neuroma surgery, it is very easy to accidentally sever muscles in the interspace you are working in thinking that it is really the nerve. Lumbricals are plantar to the deep transverse intermetatarsal ligament while interossei are above this ligament. The 1st lumbrical inserts into the soft tissue around the MTPJ of the 2nd toe so this is not the answer as the dissection occurred in the 3rd interspace. There is no such thing as 5th dorsal interossei- there are only 4 and the fourth inserts into the 4th toe proximal phalanx. The 3rd plantar interossei inserts into the base of the proximal phalanx of the 5th toe. The correct answer is 3rd Lumbrical. This inserts into the soft tissue around the MTPJ of the fourth toe and muscle belly is in the 3rd interspace.

Vicryl® is a commonly used suture in podiatry. Approximately how much of its original strength does it retain after being in the body for 2 weeks? A. 100%, it's nonabsorbable. B. 80% C. 60% D. 20%

Correct answer: (C) 60%. Explanation: After being the in body for about 2 weeks, Vicryl® has approximately 60% of its original tensile strength remaining. Tensile strength is the first property to decrease in the absorption process followed by the mass of the material. Vicryl®, also known as polyglactin 910, is a braided synthetic, absorbable, multifilamentous polymer of lactide and glycoside. Sythetic materials, like Vicryl®, are known for their more reliable loss of tensile strength and absorption when implanted in the body compared to natural suture materials like catgut. Vicryl®, like all synthetic absorbable suture material, are broken down via the process of hydrolysis, which is initiated when the material is exposed to the water of soft tissues. Hydrolysis has much less tissue reaction than the enzymatic process that natural materials go through for breakdown in the body. The end products of hydrolysis are removed from the body via urination and expiration. (A) 100%, it's nonabsorbable is incorrect as Vicryl® is a synthetic, absorbable material. Even some nonabsorbable materials will go through some form of breakdown within the body but this typically takes many months. For example, silk may only retain approximately 40% of its original tensile strength after 8 months of implantation within the body. (B) (D) After being the in body for about 2 weeks, Vicryl® has approximately 60% of its original tensile strength remaining.

When caring for diabetic patients with limb threatening infections, statistics have shown that early surgical treatment for forefoot infection will limit the risk of amputation proximal to the ankle. When medically possible, this surgical care should take place within ____ of diagnosis. A. 12 hours B. 24 hours C. 72 hours D. 96 hours

Correct answer: (C) 72 hours. Explanation: It has been shown that surgery performed more than 72 hours after admission leads to higher risk of amputation proximal to the ankle. Intervention must consist of surgical decompression, drainage, and debridement. Deep wounds cultures should be obtained. Likely the wound should be left opened and empiric antibiotic therapy initiated. Once it is performed, a more controlled workup can begin and further decisions on treatment of the wound can be made.

In correcting an abnormal metatarsal parabola deformity, the surgeon employs an obliquely oriented osteotomy. The osteotomy has been fixated with a single small diameter screw using a lag technique oriented midway perpendicular to the longitudinal axis of the metatarsal and perpendicular to the osteotomy. A second point of fixation should be employed for the following reason: A. A secondary point of fixation should be used to increase compression across the osteotomy site. B. A secondary point of fixation is not necessary as the orientation of the screw provides optimum resistance to all deforming forces. C. A secondary point of fixation provides resistance to rotational force that is not countered by the single point of fixation construct. D. A secondary point of fixation will facilitate secondary bone healing.

Correct answer: (C) A secondary point of fixation provides resistance to rotational force that is not countered by the single point of fixation construct. Explanation: When fixating an oblique osteotomy, two points of fixation is the ideal construct as it provides the best combination of compression and resistance to deforming forces. Deforming forces include shear, bending, torsion and rotation. Only two points of fixation can counter the deforming force of rotation thereby making the correct answer C. The second point of fixation would not increase compression across the osteotomy. Additionally the single fixation point construct does not provide optimal resistance to all deforming forces as it does nothing to counter rotational forces. The secondary point of fixation provides an effective compliment to the stability and compression provided by the solitary lag screw. It provides the resistance rotational force that the solitary point cannot. By providing optimal stability and rigid fixation, primary bone healing can occur as opposed the secondary bone healing that would likely occur with the instability afforded by the single fixation construct.

What is an advantage of the dorsal and plantar flap approach versus the medial to lateral flap approach in digital amputation surgery? A. Better visualization B. Better cosmesis C. Ambulation on more durable plantar skin D. Lowe rate of dehiscence

Correct answer: (C) Ambulation on more durable plantar skin. Explanation: Digital amputation utilizing dorsal and plantar allows the distinct advantage of weight bearing on the thicker, more durable plantar skin. In fact, the medial and lateral flap exposure of leads to a painful plantar scar. (A) Better visualization is provided by the medial and lateral flap exposure in digital amputation. (B) Cosmesis is quite acceptable with both exposures. (D) Dehiscence rates are comparable between both the dorsal and plantar flap exposure as they are with the medial and lateral flap exsposure.

Which of the following agents are appropriate for use in PMMA antibiotic beads? A. Cefazolin and cefotetan B. Vancomycin and Bactrim C. Aminoglycosides and Vancomycin D. Gentamicin and Bactrim

Correct answer: (C) Aminoglycosides and Vancomycin. Explanation: The aminoglycosides and vancomycin are the best choices for antibiotic bead use because of their broad spectrum of activity, low incidence of reaction, heat stability and extensive research. Other antibiotics used in PMMA beads include cefazolin, cefotetan, ceftriaxone, ceftazidime, penicillin V, nafcillin, amoxicillin, minimum inhibitory concentration (MIC) PMMA-antibiotic laden cement (PMMA-ALC), ticarcillin, piperaciIIin, and doxycycline. Antibiotic choice should be culture directed when possible. However, some authors believe that MIC s are not valid with local antibiotic therapy because otherwise resistant organisms will be sensitive at the high tissue concentrations obtained with antibiotic beads. The antibiotic selected for incorporation into PMMA should possess a broad spectrum of microbial activity, have a low allergenic rate, and be heat stable. Gentamicin has been the most extensively studied antibiotic used in PMMA because it possesses all of these characteristics. Antibiotics are incorporated into PMMA either by the manufacturer or by the surgeon on the surgical table, with the latter requiring off-label use in the United States. Release of antibiotic from PMMA-ALC occurs in a biphasic or bimodal profile. The bulk of the antibiotic is leached from the cement rapidly over the first few hours to days postimplantation, with the remaining antibiotic release being slower and sustained with the potential to persist for several years. Wahlig and Dingeldein found that the gentamicin sulfate in Palacos cement (Biomet, Inc., Warsaw, Indiana) had sustained release for up to 5 years.

Which statement regarding diabetic patients and the effects of anesthesia is true? A. Non-diabetics and diabetics respond to anesthesia the same. B. Any type of sedation may impair symptom recognition of hyperglycemia. C. Any type of sedation may impair symptom recognition of hypoglycemia. D. A preoperative blood glucose check is all that is needed until after the operation.

Correct answer: (C) Any type of sedation may impair symptom recognition of hypoglycemia. Explanation: Modern inhalation anesthetics have relatively little effect on how the body regulates metabolism. Spinal, epidural and peripheral nerve blocks have very little effect of blood glucose levels. With that said an ankle block is the preferred method of anesthesia for diabetics. In the diabetic population particular attention must be payed to the use of induction medications and muscle relaxants, particularly in the geriatric population. The justification for the last statement is that any type of sedation will impair the medical team's recognition of hypoglycemia. Ant type of sedation may impair recognition of sings and symptoms of hypoglycemia. It has been well documented that asymptomatic hypoglycemia is common in diabetic patients who are not receiving central nervous system depressants.

In modern day medical imaging, MRI has essentially replaced nuclear medicine bone scanning for the evaluation of radiographically occult fractures. Indications for computed tomography (CT) have also diminished due to MRI. CT examination is still the gold standard for evaluating overall alignment. In which of the following situations is CT also more appropriate than MRI? A. Osteomyelitis B. Bone tumors C. Articular surface interfaces D. Stress fractures

Correct answer: (C) Articular surface interfaces. Explanation: In modern day imaging, MRI is a superior modality for the evaluation of stress fractures, infectious process such as osteomyelitis, as well as bone tumors. Computed tomography tops MR imaging in the arena of evaluation of articular surfaces, cartilage and the interface of two joint surfaces. Additionally, computed tomography reigns supreme in evaluation of skeletal alignment and deformity correction planning.

What is the first step in treating frostbite of the toes? A. Gradual rewarming B. Protection of the toes from further injury C. Rapid rewarming D. Placement of the feet in a dependent position with heat pad placement behind the knees

Correct answer: (C) Rapid rewarming. Explanation: The first step in the treatment of frostbite is to rapidly rewarm the digits in water of 104 to 111° F. It is of the highest priority to "thaw" out the tissue to reestablish perfusion and limit tissue necrosis. (A) Gradual rewarming is simply too slow and it is likely that more extensive tissue necrosis would occur as perfusion of the frostbitten tissues would be delayed. (B) While protecting the fragile toes from further injury does carry a degree of importance, it does not garner the priority of rapid rewarming. (D) Placement of the feet in a dependent position and placement of hearing pads behind the knees would cause a vasodilation of the popliteal arteries but would not increase end perfusion as the distal tissue would remain frozen.

You are examining the radiographs of a 24 year old white female with a chief complaint of pain in her right ankle. Radiographs are obtained and a lesion is noted in the distal fibula. The lesion is oval, measuring 3 cm x 2 cm with a thin sclerotic rim surrounding the lesion. There is no evidence of cortical destruction or periosteal activity. The lesion has a matrix with punctate areas with marked increase in density surrounded by areas of marked radiolucency. The lesion appears to be centered within the distal epiphyseal region. Which of the following is the most likely diagnosis for the lesion described above? A. Enchondroma B. Osteoid Osteoma C. Chondroblastoma D. Chondrosarcoma

Correct answer: (C) Chondroblastoma. Explanation: A chondroblastoma is a benign cartilaginous lesion. Radiographically it is a lytic lesion located in the epiphysis. It may present as jont pain due to its typical periarticular location. The incidence is higher in males than in females 2:1. Typically it shows up in the late teens to 20s. On the other hand, an enchondroma is located in the medullary canal of small tubular bones. It may have punctate calcifications and be expansile with cortical thinning ad possible fracture. An osteoid osteoma is an osseous lesion that is highly symptomatic with pain worse at night. Classically the pain is relieved with aspirin. It similarly has a male to female ratio of 2:1. Radiographically the lesion is < 1 cm with a central nidus surrounded by a reactive sclerosis. Lastly a chondrosarcoma is a malignant lesion. Radiographically it is often a large lucent lesion with punctate calcifications. It may exhibit an associated soft tissue mass as well as cortical disruption and periosteal reaction.

64 year old African American female with a 60 pack year tobacco history of smoking presents to the office to discuss the amputation of the fourth and fifth digits of her right foot. The podiatrist states that her strong smoking history increases her risk for complications with this procedure. What is the best advice you can give her to help improve her outcomes? A. Cut down her smoking to a half a pack/day B. The use of albuterol for 1 week prior to surgery C. Complete smoking cessation D. Exercising 30 minutes three times/week prior to surgery

Correct answer: (C) Complete smoking cessation. Explanation: Smoking Cessation is the single most important prognostic factor when it comes to surgical outcomes with smokers. There has not been an exact amount of time pinpointed to exactly how long one must stop smoking tobacco, but it has been shown the longer the smoking cessation the better the outcomes typically are (C). As exercise (D) may in theory help, there has not been any strong evidence to prove this. (B) Albuterol usage, and (A) cutting down to a half pack a day has not been proven to show better surgical outcomes.

A Cole procedure for cavus foot involves an osteotomy of which osseous structures? A. Talonavicular and Calcaneocuboid joints B. Talus and Calcaneus C. Cuneiforms, Navicular, and Cuboid D. Lis Franc's joints

Correct answer: (C) Cuneiforms, Navicular, and Cuboid. Explanation: The Cole procedure is a closing wedge osteotomy with removal of a dorsally based wedge. The wedge is removed from a distal cut through the cuboid and cuneiforms coupled with a proximal cut through the cuboid and navicular. This elevates the foot out of equinus. It is indicated when the apex of the deformity is in the midfoot and likewise contraindicated whenever the apex of the deformity is somewhere else. It is only indicated for the skeletally mature foot. The wedge created can be made wider medially laterally to further correct deformity in the transverse or frontal planes.

What type of instruments contain a small scoop-like blade on a handle and are used for things such as gouging rough bone to debriding of soft tissues depending on the type used? A. Periosteal elevators B. Gouges C. Curettes D. Traction forceps

Correct answer: (C) Curettes. Explanation: Curettes contain a small scoop-like blade on a handle and are used for things such as gouging rough bone to debriding of soft tissues. Heavy curettes are typically used for gouging in bone as well as debridement of necrotic bone and removing articular cartilage when performing arthrodesis. Dermal curettes are used for debridement of soft tissues as they are made lighter construction. (A) Periosteal elevator is incorrect. The instruments have smooth and wide blades that vary in sharpness. They are typical used for reflecting the periosteum from the bone to which it is connected, as well as being occasionally used retracting deeper tissues. Examples of periosteal elevators include the freer elevator, Sayre elevator, key elevator, and Crego elevators. (B) Gouges are incorrect. Gouges are very similar to chisels in that they both only have one surface that tapers to a fine edge. However, gouges have a curved cutting edge making them useful for smoothing curved surfaces and rounding flat surfaces of bone. They come in a variety of sizes. One common gouge used in podiatric surgery is the McGlamery metatarsal elevator. (D) Traction forceps are incorrect. Traction forceps are hinged instruments made up of a set of jaws for grasping tissues, finger rings, and a locking mechanism. The grasping aspect of the jaws is designed to provide traction without unnecessary pressure that may crush the object being grasped.

What radiographic finding would one expect to find on a lateral foot projection of a cavus foot deformity after a plantar fascial release or a Steindler stripping? A. First metatarsal elevatus B. Midfoot fault C. Decrease of calcaneal pitch D. Distal migration of the sesamoids

Correct answer: (C) Decrease of calcaneal pitch. Explanation: Both a plantar fascial release and a Steindler stripping would create a decrease in the calcaneal pitch or calcaneal inclination angle. This is due to the release of the windlass mechanism. (A) Neither a plantar fascial release nor a Steindler stripping would result in a a first metatarsal elevatus as the peroneus longus is the main stabilizer of the first ray. (B) Neither a plantar fascial release nor a Steindler stripping would result in a midfoot fault or sag. (D) Neither a plantar fascial release nor a Steindler stripping would result in distal migration of the sesamoids as they would be locked into position by the flexor hallucis longus tendon.

What is the most serious complication of digital interphalangeal arthrodesis in the rheumatoid patient? A. Infection B. Pin breakage C. Digital ischemia D. Nonunion E. Pseudarthrosis

Correct answer: (C) Digital ischemia. Explanation: Digital ischemia is a potential complication of the distal interphalangeal joint arthrodesis in the rheumatoid patient, due to vascular embarrassment from stretching and kinking of the vessels when attempting to reduce severe joint contractures. (A) Infection is a more common complication than digital ischemia, but is generally more easily treated. (B) Pin breakage is a much more common complication than digital ischemia, but is not as serious and is more easily treated. (D) Nonunion is an uncommon complication of digital interphalangeal arthrodesis. (E) Pseudarthrosis is an uncommon complication of digital interphalangeal arthrodesis.

A 31 year old female patient is going to the operating room for bunion surgery as well as 4 hammertoe corrections under sedation anesthesia. Patient takes both metformin and regular insulin daily and is not well controlled per patient. Which of the following is correct regarding this patients pre-op course? Continue metformin through day of surgery and take 1/3-1/2 of regular insulin the morning of surgery. Discontinue metformin and double regular insulin day of surgery. Discontinue metformin and administer 1/3-1/2 patients AM insulin dose morning of with frequent blood glucose checks. Make the patient the last surgery of the day.

Correct answer: (C) Discontinue metformin and administer 1/3-1/2 patients AM insulin dose morning of with frequent blood glucose checks. Explanation: The most important thing to pick out of this question is that the patient is on metformin and insulin daily. Metformin is usually stopped 2 days prior to surgery to prevent lactic acidosis. Metformin in general should also not be utilized in patients over the age of 80 or with a serum creatinine of >1.5 mg/dl in males or >1.4 mg/dl in females. Regular insulin is usually given up until the morning of surgery and on the morning of surgery only 1/3-1/2 of normal dose is given as the patient should be NPO since midnight. Frequent accuchecks are done with further coverage as needed.

What complication can result from medial rotation of the saw blade during the osteotomy for a crescentic bunion procedure? A. Plantarflexion of the metatarsal B. Supinated hallux position C. Elevatus D. Pronated hallux position

Correct answer: (C) Elevatus. Explanation: Medial rotation can lead to metatarsal elevates or dorsiflexion at the osteotomy site. It is for this reason that it is imperative that that neither medially or lateral rotation ope the saw blade occurs during performance of the crescentic osteotomy. (A) Neither medial nor lateral rotation of the saw blade during the crescentic osteotomy cause plantarflexion of the first metatarsal. (B) Neither medial nor lateral rotation of the saw blade during the crescentic osteotomy cause a supinated hallux position. (D) Neither medial nor lateral rotation of the saw blade during the crescentic osteotomy cause a pronated hallux position.

Pedal xrays demonstrate an oval expansile geographic lesion with sharp margins and a thin rim of sclerosis within the middle phalanx of the second digit. This is most consistent with: A. Osteoid osteoma B. Osteochondroma C. Enchondroma D. Ewing sarcoma

Correct answer: (C) Enchondroma. Explanation: Enchomdroma is a benign bone tumor characterized by an intramedullary geographic appearance with clear margins and a thin rim of sclerosis. They are typically located in the metaphysis or less commonly the diaphysis of tubular bones. (A) Osteoid osteoma is an osteoblastic tumor with three main types: cortical, medullary and subperiosteal. The cortical lesions are intracortical lucencies marked by a central nidus of new bone formation. (B) Osteochondroma is a benign cartilaginous-capped bone tumor. Radiographically it resembles a protruding mushroom-shaped exostosis with the cortex flaring into the cortex of the underlying bone and directed away from a joint. (D) Ewing sarcoma is a malignant tumor often occurs in long bones of the lower extremities. Radiographically there is permeative bone destruction with mottled appearance. Often there is periosteal reaction with the "onion skin" type appearance.

While performing isolated extensor tenotomies on the second and third digits on a geriatric patient, the surgeon notices immediate dorsal contracture of the fourth and fifth digits. What is the most appropriate action to take at this point? A. No action is required. B. Plantarflexory bandaging of the fourth and fifth digits. C. Extensor tenotomies of the fourth and fifth digits. D. Post arthroplasty procedure to the fourth and fifth digits.

Correct answer: (C) Extensor tenotomies of the fourth and fifth digits. Explanation: With the immediate dorsal contracture of the fourth and fifth after extensor tenotomies, the most appropriate action to take would be matching extensor tenotomies of the fourth and fifth digits. The surgeon must keep in mind that although the extensor tenotomy seems innocuous enough, severing one or two slips of the tendon will proportionately increase the tension of the remaining slips. (A) It would be inadvisable to take no action at all. The action of the surgeon severing the extensor tendons is what caused the immediate contracture of the fourth and fifth toes. (B) Bandaging alone would not have any lasting effect of the newly-formed contracture of the fourth and fifth digits. (D) The cause of the contracture is the overload of the extensor tension slips of the fourth and the fifth digits. That is where the corrective action should be focused rather than the proximal interphalangeal joints.

A 66 year old female presents to the clinic complaining of sharp pain under her first metatarsophalangeal joint for approximately eight months. She has a mostly sedentary lifestyle. The following radiograph was performed at her visit. What would be the most appropriate nonsurgical intervention? A. corticosteroid injection B. pulsed electromagnetic bone stimulation C. functional/accommodative orthotic with first metatarsal head cutout D. accommodative orthotic with a morton's extension

Correct answer: (C) Functional/accommodative orthotic with first metatarsal head cutout. Explanation: This is a chronic, comminuted fracture of the fibular sesamoid. The sclerotic margins are consistent with avascular necrosis. Due to the patient's age and lifestyle, nonsurgical solutions would make the most sense. The nonsurgical treatment of choice in this instance would be a functional/accommodative orthotic with a first metatarsal head cutout. An accommodative orthotic with morton's extension would increase pressure under the sesamoid fracture and cause more pain so it would not be a good choice. Corticosteroid injection in the area would not only be very painful, but could lead to rupture of the lateral slip of the flexor hallucis longus tendon. Pulsed electromagnetic bone stimulation would not be indicated as this is clearly an atrophic nonunion.

A patient presents with painful hallux abductovalgus deformity. There is pain-free range of motion of the first metatarsophalangeal joint (MTPJ) with hypermobility of the 1st ray. Radiographs reveal: hallux abductus angle: 37 degrees intermetatarsal angle: 19 degrees proximal articular set angle: 22 degrees distal articular set angle: 4 degrees What is the most appropriate procedure? A. Closing wedge with McBride B. Proximal Akin with McBride C. Lapidus with Reverdin-Green D. Austin with proximal Akin

Correct answer: (C) Lapidus with Reverdin-Green. Explanation: With the increased intermetatarsal angle (greater than 15), a base metatarsal procedure is indicated, which includes choices A/C. An Austin is a metatarsal head procedure and an akin is a proximal phalanx base procedure. Therefore B-proximal akin with mcbride and D-austin with proximal akin are wrong. With the proximal articular set angle (PASA) being 22 degrees, an additional procedure is needed to correct the PASA. A McBride is a metatarsal head procedure that decreases the exostosis on the metatarsal head and decreases the contracture on the 1st metatarsal head, but does not correct the PASA. However, the Reverdin-Green procedure does decrease the PASA. Therefore, the answer choice is Lapidus with Reverdin-Green.

Your 24 y/o patient is scheduled for a triple arthrodesis. She relates a family member who died during surgery from a diagnosed case of malignant hyperthermia. The best screening test to determine her susceptibility to MH is: A. History B. CK levels C. In vitro halothane or caffeine contracture test D. DNA analysis

Correct answer: (C) In vitro halothane or caffeine contracture test. Explanation: This is, for now, the "gold standard" in the diagnosis of MH. Two forms of this test have been developed; in Europe by the European Malignant Hyperthermia Group (EMHG) and the North American Malignant Hyperthermia Group (NAMHG). In the EMHG protocol, a person is susceptible to MH when both caffeine and halothane tests are positive. When both tests are negative, the person is not susceptible. Using this protocol, there is 94% specificity and 99% sensitivity. The test is performed on muscle strips from surgically removed muscle biopsies. A downside is finding a lab to perform the test. (A) Although incorrect, always remember the history is your first tool that will lead you down the right road, to determine the best test to order. You should always take an appropriate history with your NLDOCATS in any oral board exam. (B) Based upon the patient's history, a simple blood test for creatinine kinase levels should be performed; again, to lead you to the next appropriate test. From Anesth. Analg.1997 May. 84(5). 1038-41: "Unexplained persistently elevated CK levels in an otherwise healthy patient should alert the anesthesiologist to the possibility of MHS(susceptible) and/or myopathy." (D) Although incorrect, it is an up and coming test. It is an alternative to IVCT, requiring only a blood sample. 50-70% of MHS (susceptible) has been linked to RYR1, with over 400 variants associated with MH being identified within this gene.

With regards to performing surgery on diabetic patients, which statement below is correct? A. Hyperglycemia promotes phagocytosis. B. It is best to do surgery late morning to afternoon. C. Inhalation agents suppress insulin secretion. D. Inhalation agents upregulate insulin secretion.

Correct answer: (C) Inhalation agents suppress insulin secretion. Explanation: Performing surgery on diabetic individuals bring several unique considerations to the table. The disease impacts preoperative fluid, electrolyte and nutritional balance. Cardiovascular function as well as renal function are also impacted. The patient's immune system is impacted as well as their ability to heal wounds. A very important consideration to make is that inhalation anesthetics suppress the pancreas's ability to secrete insulin. In contrast, hyperglycemic states inhibit phagocytosis which leads to an increase in post-operative infection rates. In regards to timing, it is ideal to perform surgery on a diabetic early in the morning.

Please refer to the MR image below. What is the likely cause of the calcaneal tongue fracture? A. brake pedal injury from a motor vehicle accident B. fall from a height C. insufficiency fracture from osteomyelitis D. blunt force trauma to the posterior heel

Correct answer: (C) Insufficiency fracture from osteomyelitis. Explanation: The key to this question is the recognition of the infectious process. The direct extension osteomyelitis stemming from the posterior heel ulcer has weakened the calcaneus to the extent that the contraction from the Achilles tendon was able to shear the superior one third off. The increased signal intensity of the T2-weighted image with a contiguous connection to the ulceration is indicative of osteomyelitis.

Which of the following is associated with an increased risk of malignant hyperthermia and should be avoided in patients with a family history of malignant hyperthermia? A. Propofol B. Dantrolene sodium C. Isoflurane D. Sodium pentothal

Correct answer: (C) Isoflurane. Explanation: Malignant hyperthermia (MH) is a genetic disorder that results in increased intracellular calcium levels following exposure to a variety of agents potentially used during induction and maintenance of anesthesia. The halogenated inhalational agents used for induction of anesthesia including sevoflurane and isoflurane are known to be triggers of MH. Succinylcholine, a depolarizing muscle relaxant, is also a known trigger of MH. (A) Propofol is a commonly used intravenous anesthetic, however one of its advantages is that it is not known to trigger MH. (B) Dantrolene sodium is the treatment of choice for MH due to its ability to rapidly reduce the intracellular calcium load that occurs during MH. (D) Sodium pentothal is an anesthetic that is no longer commonly used, but it is also not associated with episodes of MH.

What happens to the center of rotation of the first metatarsophalangeal joint in the picture below? A. It remains the same as before distraction. B. It moves proximally. C. It moves distally. D. It moves plantarly.

Correct answer: (C) It moves distally. Explanation: Whenever distracting a joint the center of rotation of that joint is affected. In the case of the first metatarsophalangeal joint, the center of rotation moves distally within the head of the bone. As seen above a hinge can be placed at the new center of rotation. This allows passive range of motion to the distracted joint. Full weight-bearing is allowed.

Which option listed below would be the best procedure as part of the surgical plan to correct the hallux malleus? A. Girdlestone procedure B. Hibbs procedure C. Jones Tenosuspension with an IPJ arthrodesis D. MTP and IPJ arthrodesis

Correct answer: (C) Jones Tenosuspension with an IPJ arthrodesis. Explanation: Hallux malleus is commonly encountered in cavus and anterior caves foot types. Due to the plantarflexed nature of the first ray the extensor hallucis longus contracts for a longer period of time in the gait cycle to facilitate the clearance of the toes to the ground. This causes an uneven tug-o-war phenomenon and the extensor hallucis longus overpowers the flexor hallucis longus. This deforms the hallux and leaves very prominent extensor tendon tenting the skin on the dorsum of the foot. The Jones tenosuspension involves transferring the extensor hallucis longus tendon into the neck of the first metatarsal and fusion of the hallux interphalangeal joint. This creates a rigid lever in the hallux for propulsion and a deforming extensor tendon into a dorsiflexor of the first ray.

What complication is unique to an interspacectomy (unrestricted cleaning out of the interspace) that is not even potentially encountered with a well-dissected neurectomy? A. hammertoe formation B. hematoma C. loss of the plantar fat pad D. stump neuroma

Correct answer: (C) Loss of the plantar fat pad. Explanation: Cleaning out the contents of the interspace, an interspacetomy. is not a recommended practice as it is inherently fraught with complications. If critically considered, this would sever the artery, lumbricale tendon, inter metatarsal ligament as well as the nerve. It would also remove the interspace adipose tissue and potentially the organized lattice of the fat pad. With this traumatic technique it os clear to see how a hammertoe, hematoma and stump neuroma can be formed. The alteration of the plantar fat pad can lead to severe metatarsalgia. In contrast, the chance of these complications are minimized. Furthermore, the chance for altering the plantar fat pad is nonexistent with atraumatic technique.

A 27 year old patient presents to office to discuss bunion surgery. She says that she previously had surgery completed on the contralateral foot and would like surgery on the other foot. States that during the previous surgery there was a complication. Patient cannot remember the name of the complication but says that she has elevated creatinine phosphokinase and EKG abnormalities. She also had fever and rigidity. What is the most likely complication this patient had per clinical history? A. Benign intra-operative fever B. Severe Hypertension C. Malignant Hyperthermia D. Reynaud's Disease

Correct answer: (C) Malignant Hyperthermia. Explanation: All of the symptoms that are discussed in the stem of the question lead to malignant hyperthermia being the correct answer. Malignant hyperthermia can be life threatening at times leading to high fever and muscle rigidity. It is also common to see EKG changes. Benign intra- operative fever is sometimes seen during or just after surgery, but does not usually present with EKG changes or increased CPK. Hypertension can also be seen intra-operatively but would most likely not show increased CPK. Reynaud's disease is a disorder that causes reduced blood flow to vessels in the feet in the presence of cold or stress.

A 28 year old male presents complaining of frequent ankle sprains of his right foot. Upon examination the loss of the Achilles reflex on both feet was noted. Patellar reflex was present bilateral. Decreased vibratory and proprioceptive sensations were also noted bilaterally. He reports no significant medial history. Below is a clinical photograph. From the options listed below, what would be the next most logical treatment decision? A. Custom foot orthotic B. Custom ankle foot orthotic C. Neurological consultation D. Coleman block test

Correct answer: (C) Neurological consultation. Explanation: The next most logical treatment for the patient described above would be a neurological consult. The patient described above likely has Charcot Marie Tooth (CMT) disease or a similar entity and prompt and complete neurological evaluation is warranted. The absence of the Achilles reflex combined with the decrease in vibratory and proprioceptive sensation combined with the appearance of the foot is highly suggestive of CMT. (A) Although a custom foot orthotic might play a role in the treatment of the individual above, the fact that an underlying undiagnosed neuromuscular condition exists, a neurological consolation is better justified. (B) Although a custom ankle foot orthotic might play a role in the treatment of the individual above, the fact that an underlying undiagnosed neuromuscular condition exists, a neurological consolation is better justified. (D) The Coleman block test would aid in potential orthotic and/or surgical planning. However, since an underlying undiagnosed neuromuscular condition exists, a neurological consolation is better justified.

Which of the following operative procedures can be done for the treatment for Sever's disease? A. Subtalar joint fusion B. PRP injection C. Nothing, the problem is self-limiting D. Removal of ossification center

Correct answer: (C) Nothing, the problem is self-limiting. Explanation: Sever's disease or calcaneal apophysitis is caused by inflammation of the secondary ossification center of the calcaneus. It is due to traction apophysitis and repetitive microtrauma experienced during gait, seen often in children aged 8-14. Due to it being an overuse injury it is considered a self-limited process that resolves with maturation and closure of the apophysis, therefore no surgical intervention is indicated. All the other answers include invasive measures.

Which of the following risk factors carries the highest risk of infection in class one (clean) surgical wounds? A. Steroid use B. Age of 53 years C. Obesity D. Diabetes

Correct answer: (C) Obesity. Explanation: Although all choices elevate risk of infection in a surgical patient, obesity raises it the highest. The reason for this remains uncertain. In fact it raises it 3% higher than diabetes. Both the patient with chronic steroid use and those with diabetes are in an immunocompromised state. In other words they do not have the ability to fight the contaminating organisms with a functioning immune system. Age is definitely a factor for multiple reasons. First as we grow older we are more likely to have an underlying medical condition and immune senescence. In even later years there can be long-term exposure to resistant pathogens especially in long-term care settings. Malnutrition may also be seen.

Which one of the following is a radiographic sign of osteoarthritis? A. Soft tissue swelling B. Periarticular osteopenia C. Osteophyte formation D. Uniform joint space narrowing

Correct answer: (C) Osteophyte formation. Explanation: Inflammatory arthritis is associated with marginal bone erosion, uniform joint space narrowing and soft-tissue swelling. The hyperemia involved with acute synovitis leads to periarticular osteoporosis and early radiographic examination might reveal only soft tissue swelling and diffuse juxtaarticular osteopenia. Osteophyte formation and bone sclerosis are key features of degenerative arthritis.

Which of the following is FALSE regarding a common peroneal block? A. Commonly given 2-3 finger breadths distal to fibular head in the leg B. Can be used to help with diagnosis of peroneal spastic flat foot C. Patient will get numbness over the entire bottom of the foot with this block D. The common peroneal nerve is a branch of the sciatic nerve

Correct answer: (C) Patient will get numbness over the entire bottom of the foot with this block. Explanation: The common peroneal block is a common block for podiatrists when working on the ankle. It is commonly given 2-3 finger breadths distal to the head of the fibula. It is commonly used in patients who may have peroneal spastic flat foot as a diagnostic tool. With a common peroneal block, the patient will not get numbness over the entire bottom of their foot. In order for that to occur, you would need to do posterior tibial nerve block. The common peroneal nerve is one of the two major branches of the sciatic nerve with the tibial nerve being the other of the two.

Which procedure is indicated for treatment of bunionette deformity with significantly elevated intermetatarsal 4-5 angle? A. Minimal incision oblique osteotomy B. Distal metatarsal osteotomy C. Proximal base osteotomy D. Dorsal-lateral cheilectomy E. Chevron osteotomy

Correct answer: (C) Proximal base osteotomy. Explanation: Only the proximal base osteotomy will decrease a significantly elevated intermetatarsal 4-5 angle. (A) This osteotomy provides minimal intermetatarsal correction. (B) This osteotomy provides only minimal intermetatarsal correction. (D) The cheilectomy simply removes bony prominences with no intermetatarsal angle correction. (E) The Chevron osteotomy provides minimal intermetatarsal correction and is only indicated in cases of minimal intermetatarsal deviation.

The patient pictured below is four months post transmetatarsal amputation. How could the deformity pictured below have been prevented at the time of the amputation? A. Performance of a concomitant subtalar arthrodesis B. Application of a rigid below knee cast for eight weeks C. Performance of a split tibialis anterior tendon transfer and equinus reduction D. Performance of a tibialis poster tendon transfer through the interosseous membrane

Correct answer: (C) Performance of a split tibialis anterior tendon transfer and equinus reduction. Explanation: A varus foot deformity is almost a certainty if not addressed at the time of the transmetatarsal amputation. Two of the three ankle dorsiflexors are sacrificed (extensor digitorum longus and extensor hallucis longus). This leaves a very imbalanced tug-o-war situation were the strongest tendon in the body, achilles tendon, as well as four other ankle plantar flexors pull against the lone anterior tibial tendon. This causes an overactive anterior tibial tendon which causes a varus deformity due to its medial attachment relative to the midtarsal and subtalar joints. The split tibialis anterior tendon transfer would equalize the dorsiflexory pull and eliminate the varus deformity. Furthermore the equinus release either through a gastrocnemius recession or a tendoachillles lengthening will weaken the plantarflexory pull and decrease the load on the tibialis anterior tendon. Subtalar arthrodesis is not indicated and likely would not be well tolerated from a patient requiring an amputation like this. A cast would have no influence. It would keep the foot straight while the cast was on, but once the cast was removed the deformity would manifest itself. A transfer of the tibialis posterior tendon through the interosseous membrane does not address the imbalance biomechanics and therefore is not indicated.

What is the radiographic hallmark of vertical talus? Persistent dorsal dislocation of the navicular on the talus in the lateral projection Hindfoot in equinus on the lateral projection Persistent dorsal dislocation of the navicular on the talus in both maximum dorsiflexion and maximum plantarflexion lateral projections Increased tibiocalcaneal angle

Correct answer: (C) Persistent dorsal dislocation of the navicular on the talus in both maximum dorsiflexion and maximum plantarflexion lateral projections. Explanation: The radiographic hallmark of the vertical talus foot is the persistent dorsal dislocation of the navicular on the talus in both maximum dorsiflexion and maximum plantarflexion lateral projections. It is for this reason that it is very important to get plantarflexed and dorsiflexed lateral projections. (A) Persistent dorsal dislocation of the navicular on the talus in the lateral projection is only partially correct. This dislocation must be confirmed in both a plantarflexed and a dorsiflexed attitude. (B) Although the hindfoot is identified in equinus on the lateral projection, this is not the radiographic hallmark. (D) Although an increased tibiocalcaneal angle is seen on the lateral radiogram it is not the radiographic hallmark of vertical talus.

Concerning the depth of anesthesia, stage 3 is further subdivided into 4 planes. Which plane is best described by the following characteristics? - eyelid reflex absent - diaphragmatic breathing - complete abdominal relaxation A. Plane 1 B. Plane 2 C. Plane 3 D. Plane 4

Correct answer: (C) Plane 3. Explanation: When discussing the depth of anesthesia there are four separated stages as described by Guedel in 1937. Stage 1 is amnesia/analgesia, stage 2 is delirium/excitement, stage 3 is surgical anesthesia and stage 4 is overdosage. Stage 3 lasts from the onset of regular breathing to respiratory arrest. Stage 3 is further subdivided into four planes. Plane 3 can best be characterized by complete absence of the eyelid reflex, dialed pupils and the light reflex is absent. Furthermore, the patient will exhibit diaphragmatic breathing as progressive intercostal paralysis occurs until the intercostals are completely paralyzed.

Referring to the diagnostic image below of a patient's left heel, what working diagnosis can be reached? A. Calcaneal stress fracture B. Heel spur C. Plantar fasciitis D. Baxter's neuritis

Correct answer: (C) Plantar fasciitis. Explanation: The image below is a sagittal plane ultrasound image of the left heel. Clinically two distinct features are present. The first is a slight calcaneal exostosis. The second, which is highlighted by the yellow arrow, is the severely thickened plantar fascia. This plantar fascia is 3- 4 times thicker than normal. This is clearly indicative of plantar fasciitis. A calcaneal stress fracture is not a possibility here. Although a heel spur is present, that alone is not enough to substantiate a diagnosis. Rarely is a heel spur symptomatic. The presence of one merely suggests that the instrinsic musculature attachment site on the anterior aspect of the calcaneal tubercle is under great sustained tension. Furthermore, the first branch off the lateral plantar nerve is not visible in this image therefore excluding a diagnosis of Baxter's neuritis.

What is open kinetic chain supination at the subtalar joint? A. Dorsiflexion, eversion, abduction B. Dorsiflexion, inversion and plantarflexion C. Plantarflexion, inversion and adduction D. Plantarflexion and adduction

Correct answer: (C) Plantarflexion, inversion and adduction. Explanation: The motion available at the subtalar joint is triplanar, as the axis is deviated between all three body planes. The triplanar motions are known as pronation and supination. In open kinetic chain, with the foot free at the end of the leg, the components of supination are plantarflexion, adduction, and inversion. The resultant position of an open chain supinated subtalar joint is a foot pointing down, toward the midline, and facing inward.

Which of the following radiation monitoring devices can provide an immediate reading? A. TLD B. OSL C. Pocket ionization chamber device D. Film badges

Correct answer: (C) Pocket ionization chamber device. Explanation: A device with an ionization chamber monitoring device, such as a pocket chamber or pocket dosimeter, can provide an immediate reading. TLD devices must be returned to the commercial supplier for a reading each month. The commercial supplier reads OSL dosimeters bimonthly. Film badges must also be processed by a commercial laboratory and are not reusable.

This anomaly of the digits is characterized by the presence of supernumerary digits of the feet. A. Hemimelia B. Amelia C. Polydactyly D. Sympodia

Correct answer: (C) Polydactyly. Explanation: Polydactyly is supernumerary digits on the hands or feet. Syndactyly is the fusion or fleshy webbing between the digits of the hands or feet. Hemimelia is the absence of all or part of the distal limb, the proximal limb is normal. Amelia is a congenital absence of a limb or limbs. Phocomelia is a congenital absence of the proximal limbs such that the feet or hands are attached to the trunk by an anomalous segment. Sympodia is the fusion of the lower extremities.

A physician learns that one of his patients has tested positive for HIV infection. Which of the following is true concerning HIPAA and required reposting of communicable diseases? The reporting requirements for HIV vary in different locations across the country, so the physician should check local reporting requirements for HIV. The physician must obtain consent from the patient before reporting. Reporting may be done by the laboratory or by the clinician. Patient protected health information is not included in information reported to public health authorities. Information may not be disclosed to partners without patient consent.

Correct answer: (C) Reporting may be done by the laboratory or by the clinician. Explanation: Reporting may be done by the clinician or by the testing laboratory. (A) Reportable communicable diseases in every state include HIV, AIDS, syphilis, gonorrhea, chlamydia, and chancroid. Other disease reporting requirements may vary by region. (B) Consent is not necessary for a covered entity to disclose protected health information about certain diseases to public health authorities authorized by law to carry out public health interventions or investigations. The information reported assists public health authorities to assess morbidity trends, plan resource allocation, and notify individuals who are at risk. (D) Protected patient health information is disclosed to enable public authorities to notify individuals at risk of contracting or spreading a disease. (E) A covered entity may disclose information about HIV and other communicable diseases to the extent authorized by law to individuals who are at risk of spreading or contracting the disease. Patient consent is not required.. Partner notification allows providers or public health authorities to contact partners of infected individuals to arrange evaluation and treatment and to interrupt spread of the disease. Providers may contact the partner directly or through cooperation with state and local health departments.

A 43 year old male patient on chronic anticoagulant therapy (Coumadin) is going in for elective surgery on his right foot. The podiatrist refuses to do the surgery if the patient is on Coumadin due to risk of bleeding and discusses the case with the cardiologist. The cardiologist feels the patient is at too high of a risk to stop the anticoagulation. Patient's INR 5 days before surgery is 3.1. Which of the following can occur which will allow the patient to get anticoagulation and also have surgery? Stop Coumadin the morning of and do surgery Stop Coumadin 5 days prior and bridge with heparin or lovenox and stop those three days before Stop Coumadin 5 days before and bridge with heparin or lovenox and stop hours before surgery Stop Coumadin the day before and it is ok to do surgery with INR 3.1

Correct answer: (C) Stop Coumadin 5 days before and bridge with heparin or lovenox and stop hours before surgery. Explanation. Coumadin has an approximate half life of about 40 hours. Stopping it the morning of surgery would not work because the patient's INR would still be too high and make the patient high risk. Coumadin should be stopped about 5 days before surgery. Heparin and Lovenox have half lives that are quite a bit shorter than Coumadin (1-6 hours) and thus the patient is typically stopped on Coumadin 5 days before surgery and bridged onto lovenox or heparin which can be stopped hours before surgery. Heparin is monitored by PTT and Lovenox does not need to be monitored generally but can be monitored by factor 10A.

The Harris-Beath projection is best utilized for which type of pathology? A. Talonavicular coalition B. Calcaneocuboid coalition C. Talocalcaneal coalition D. Fracture of calcaneal anterior process

Correct answer: (C) Talocalcaneal coalition. Explanation: A Harris-Beath projection is used to assess the middle and posterior facets of the subtalar joint when there is high suspicion of talocalcaneal coalition. When there is no pathology, the joint surfaces should be parallel to each other and to the ground. However, in the presence of an osseous coalition the joint space may not be visible, if a non-osseous coalition the joint space may appear angulated. (A) A talonavicular coalition would be better visualized with projections focusing on the medial column. (B) A calcaneocuboid coalition would be better viualized with projections focusing on the lateral column. (D) This answer choice is incorrect. The Harris-Beath projection allows visualization of the middle and posterior facets of the subtalar joint and the posterior aspect of the calcaneus. It would not be beneficial in evaluating the anterior aspect of the calcaneus.

Both the Mitchell and the Hohmann bunionectomy incorporate a transverse osteotomy in the distal metaphysics of the first metatarsal. What is the primary difference between the two procedures? A. the Mitchell bunionectomy involves a fibular sesamoidectomy B. the osteotomy in the Hohmann is angled from dorsal distal to plantar proximal C. the Mitchell bunionectomy incorporates a lateral cortical shelf of bone D. the Mitchell bunionectomy has more intrinsic stability as compared to the Hohmann

Correct answer: (C) The Mitchell bunionectomy incorporates a lateral cortical shelf of bone. Explanation: The primary difference between the Mitchell and the Hohmann bunionectomy is the modification of the osteotomy in the Mitchell incorporates a lateral cortical shelf which projects proximally and abuts against the lateral cortex of the proximal first metatarsal segment. The width of the lateral cortical shelf is dependent on the degree of metatarsus primus varus. The Roux modification involves making a trapezoidal wedge to correct for PASA. Neither procedure involves an obligatory fibular sesamoidectomy as well as neither osteotomy has an angulation as classically described. With that being said, the transverse osteotomy that is shared between both bunioneictomies does not afford any degree of stability to either the Mitchell or the Hohmann bunionecotomies.

During contact phase of the gait cycle, what is the main shock-absorbing mechanism? A. The hip flexion B. The ankle plantarflexion C. The knee flexion D. Heel contact E. Knee extension

Correct answer: (C) The knee flexion. Explanation: Knee flexion is the main shock-absorbing mechanism during this phase. As the heel strikes, plantarflexion at the ankle rapidly moves the tibia forward while the femur lags behind, resulting in flexion of the knee. The quadriceps restrains rapid flexion as the hamstrings resist hyperextension.

A point of clinical confusion sometime lies in the fact that the MR imaging characteristics of geodes may be identical to an intraosseous ganglion. In order to come to an accurate diagnosis, some clinical information must be considered to differentiate the two clinical entities. First, consideration of the clinical setting is important. Geodes are a manifestation from a degenerative arthritic process which, in most instances, will develop in later decades. In contrast, intraosseous ganglions are usually seen in young and middle-aged adults. What is another important clinical "clue" that can assist the physician in making the distinction between geodes and intraosseous ganglions? A. The two entities have different uptake qualities of gadolinium. B. Geodes tend not to be painful in contrast to intraosseous ganglions. C. The presence or absence of associated arthritic findings. D. Intraosseous ganglions are located in long bones where geodes are not.

Correct answer: (C) The presence or absence of associated arthritic findings. Explanation: Geode, or subchondral cyst, is a well-defined lytic lesion in the periarticular surfaces that occur in arthritic disorders. A geode is one of the common differential diagnoses of an epiphyseal lesion. This is originally a geological term referring to rounded formations in igneous and sedimentary rocks. Presumably, one method of geode formation takes place when synovial fluid is forced into the subchondral bone, causing a cystic collection of joint fluid. Another etiology is following a bone contusion, in which the contused bone forms a cyst. The intraosseous ganglion (IOG) is a subchondral lesion, reactive in nature, usually seen in young and middle-aged adults. It is most common in the ankle in the distal tibial plafond near the medial malleolus. The MR imaging characteristics of geodes may be identical to an intraosseous ganglion. However, the lack of an associated arthritis suggests the diagnosis of intraosseous ganglion.

Bones are classified as long when: A. They consist of thicker marrow. B. They consist of condensed marrow. C. They are more twice as long than wide. D. They are as wide as they are long.

Correct answer: (C) They are more twice as long than wide. Explanation: Long bones usually have greater length than width with slight posterior/ plantar concavity for strength. They are composed of a diaphsis, shaft, body, and a medullary cavity which contains bone marrow. The long bones also have epiphyses which are the ends of the bone. Their core consists of cancellous bone with a thin covering of cortical bone. Areas of articulation are covered with cartilage. The metaphysis of the bone is the flared area between the diaphysis and epiphyses. It has a core of cancellous bone with a thin covering of cortical bone.

Unresolved symptoms following third interspace Morton's neuroma excision may be secondary to an un-diagnosed "double-crush syndrome." The nerve involved in a double crush of a third interspace neuroma is the: A. Saphenous nerve B. Sural nerve C. Tibial nerve D. Superficial peroneal nerve

Correct answer: (C) Tibial nerve. Explanation: A double crush results from a proximal nerve lesion contributing to a distal nerve lesion. Thus, the proximal and distal nerve must be anatomically connected. A Morton's neuroma can be the result of a connection between the lateral and medial plantat nerves, both of which are branches of the tibial nerve. A tarsal tunnel syndrome proximally can be the "double crush" causing the distal nerve lesion. (A) The saphenous nerve has two branches, one of which courses from the medial malleolus distally to innervate the medial and dorsal foot, terminating at the base of the hallux. (B) The sural nerve has two terminal branches, the lateral calcaneal branch and the lateral dorsal cutaneous nerve, supplying the lateral side of the foot. (D) The superficial peroneal nerve has two branches; the medial dorsal cutaneous nerve and intermediate dorsal cutaneous nerve, both supplying the dorsum of the foot.

34 year old African American female patient presents to your office as a second opinion from another podiatrist for bunion surgery. She has Sickle Cell Disease but really wants the bunion to be operated on as it is causing her discomfort. Which of the following is true regarding peri- operative management of the Sickle Cell patient? Should always use general anesthesia. Patients with sickle cell are at no higher risk than the general population for surgical complications. Tourniquet should be avoided. Osteomyelitis in Sickle cell patients is usually caused by Pseudomonas.

Correct answer: (C) Tourniquet should be avoided. Explanation: Tourniquets should be avoided as much as possible in patients with Sickle Cell disease to avoid hypoxia and increased sickling. In these patients, you should try to use local anesthetic as this is the least invasive anesthesia type. Patients with sickle cell are at a higher risk than the general population for intra-operative and post-operative surgical complication. Osteomyelitis in Sickle cell patients tends to be causes by salmonella.

The concept of planar dominance is a quintessential concept in the arena of pediatric flexible pes planovalgus. Radiographic findings of an increased talocalcaneal angle, increased calcaneocuboid angle, decreased talonavicular congruency and decreased forefoot to rearfoot adduction are hall marks of what plane of dominance? A. Sagittal B. Frontal C. Transverse D. Multi-planar

Correct answer: (C) Transverse. Explanation: The radiographic findings of an increased talocalcaneal angle, increased calcaneocuboid angle, decreased talonavicular congruency and decreased forefoot to rearfoot adduction are hall marks of a transverse plane dominant pes planovalgus deformity. The means that the motion of the foot occurs at an imaginary hinge at the junction of the sagittal and frontal plane. (A) The radiographic findings described above do not describe a sagittal planar dominant pes planovalgus deformity. A sagittal plane dominate flatfoot is describe radiographically by an increased talar declination angle, increased talocalcaneal angle, decreased calcaneal inclination angle and a naviculocuneiform breach. (B) The radiographic findings described above do not describe a frontal planar dominant pes planovalgus deformity. A frontal plane dominate flatfoot is describe radiographically by decreased first metatarsal declination angle, decreased height of the sustentaculum tali, increased superimposition of the lesser tarsus and widening of the lesser tarsus. (D) There is not a multiplanar designation in the topic of planar dominance.

A patient with rheumatoid arthritis is undergoing pan metatarsal head resections with 1st metatarsophalangeal joint fusion. When is the appropriate time to stop methotrexate prior to surgery? A. Methotrexate does not need to be discontinued prior to surgery B. One day prior to surgery C. Two weeks prior to surgery D. Six weeks prior to surgery

Correct answer: (C) Two weeks prior to surgery. Explanation: Methotrexate is a disease modifying anti-rheumatic drug (DMARD) commonly used in the treatment of rheumatoid arthritis. In several studies, it has been shown to increase risk of post-operative infections and delayed wound healing. It is recommended that the medication be discontinued two weeks prior to elective surgery. (A) Due to the risk of post-operative complications including infection and delayed wound healing, methotrexate should be discontinued prior to elective surgery when possible. (B) Holding Methotrexate one day prior to surgery is likely not sufficient to significantly prevent post-operative complications. (D) Although this may be required in very severe disease processes, the medication is typically not withheld for this length.

When discussing MR imaging, it is well known that fat is one of two primary carriers of protons in the body. What is the other? A. muscle B. collagen C. water D. bone

Correct answer: (C) Water. Explanation: To produce an MR image, the anatomic body part to be examined is placed within a powerful, directional, homogenous magnetic field. This large, static magnetic field, called B0, is oriented along the patient's head-to-foot axis, the z-axis. The protons in the body, chiefly those in water and lipids, generate their own small magnetic fields and align themselves with the B0 field. The portion of the collective conglomeration of magnetized protons that is aligned with the B0 field is called longitudinal magnetization.

What deformity will likely result following a complete 5th ray amputation? A. Hallux varus B. A "pronated" foot C. A "club" foot D. A "supinated" foot

Correct answer: (D) A "supinated" foot. Explanation: In the event a complete 5th ray amputation was to be performed, a "supinated" foot is likely to result due to the loss of the pull of the peroneus brevis. The resultant "supinated" or varus deformity will significantly increase the pressure on the 4th metatarsal head and likely lead to breakdown unless the deformity is aggressively off-loaded. Likewise, if a complete first ray amputation was to be performed and the pull of the tibialis anterior was lost, a pronated foot would result. This would increase the pressure to the 2nd metatarsal head.

23 year old construction worker presents to the ED after a boulder fell on his left foot, damaging his steel toe boot. He denies any other trauma. The patient socks and shoes were removed before arrival to the fracture clinic. Past medical history: Unremarkable Past surgical History: None Social history: Non -smoker, no illicit drug abuse, no alcohol abuse The physical exam reveals the following: - No open wounds - Ecchymosis and swelling to the dorsum of the left foot - Swelling to the left foot - There is no pallor, or cyanosis - Gross sensations all intact to the left foot - Gentle palpation reveals tenderness and crepitus to the 2nd metatarsal Among the differential in a crush injury is the possibility of compartment syndrome. Even though most compartment syndromes can be ruled out clinically, which of the following can help in assessment? A. Ultrasound B. MRI C. Wick catheter D. Bone scan

Correct answer: (C) Wick catheter. Explanation: Compartment syndrome is due to increased pressure in a closed fascial space compromising the circulation to the nerves and muscles within the compartment. This can be caused by fractures, severe contusion, and drug overdose with limb compression, a burn, and vigorous exercise. Intracompartmental pressures are measured using a Wick catheter. The range of normal pressure is from zero to 8 millimeters of Mercury. A pressure of 30 mmHg is an indication for decompressive fasciotomy. The wick catheter was developed for measurement of subcutaneous pressures; the wick catheter is capable of critical diagnosis and treatment.

Which of the following is the most appropriate excisional technique for a malignant skin lesion of the midfoot? A. Marginal resection B. Intralesional resection C. Wide resection D. Radical resection

Correct answer: (C) Wide resection. Explanation: Wide resection is performed on malignant lesions with removal of the tumor and healthy viable borders. The borders of the lesion can be tagged with various sutures prior to excision and marked in the pathology report. (A) Marginal resection is excision of the lesion through the reactive zone and is often performed for benign lesions that have high risk of recurrence or does not have good healing potential. (B) Intralesional excision may be performed on benign lesions that have low risk of recurrence and have good healing potential. (D) Radical resection is excision of the entire compartment. This may result in functional impairment depending on the structures that are excised and therefore is not performed unless necessary.

Which of the following is tri-planar but not pronatory/supinatory? A. Subtalar joint B. Midtarsal joint C. 5th ray D. 1st ray

Correct answer: (D) 1st ray. Explanation: The 1st ray is the only answer above that is tri-planar but not pronatory/supinatory. This ray passes medial, posterior, dorsal instead of medial anterior and dorsal as the above. (A) (B) (C) The subtalar joint, midtarsal joint, and 5th ray are all tri-planar and pronatory/supinatory. A tri-planar axis occurs when there is motion in the frontal/transverse/ and sagittal plane. The amount of motion that occurs in these planes depends on how far the axis is from each plane. The closer the axis is perpendicular to a plane, the more motion you get in that plane. All of the above joints show that. There are two types of tri-planar motions, pronatory/supinatory and non pronatory/ supinatory. Pronatory/supinatory means that the axis passes through the joint in a direction of medial, anterior, and dorsal.

The first metatarsophalangeal joint incorporates how many articular facets: A. 1 articular facet B. 2 articular facets C. 3 articular facets D. 4 articular facets E. There is no articular facets at the first metatarsophalangeal joint.

Correct answer: (D) 4 articular facets. Explanation: The first metatarsophalangeal joint incorporates the articular facets of four bones within a single synovial joint capsule: the first metatarsal head, the base of the proximal phalanx, and the superior surfaces of the medial and lateral sesamoid bones.

How many lumbricales are found in the foot and what is the origin of each lumbricale? 3 small muscles and originate from the medial side of the corresponding flexor digitorum longus slip to the lesser toe 3 small muscles and originate from the lateral side of the corresponding flexor digitorum longus slip to the lesser toe 4 muscles and originate from the lateral side of the corresponding flexor digitorum longus slip to the lesser toe 4 muscles and originate from the medial side of the corresponding flexor digitorum longus slip to the lesser toe

Correct answer: (D) 4 muscles and originate from the medial side of the corresponding flexor digitorum longus slip to the lesser toe. Explanation: The lumbricales are 4 small muscles that have an unusual tendon origin. Most muscles originate from bone. Each of the lumbricales originates from the medial side of the corresponding flexor digitorum longus slip to the lesser toe. They course plantar to the deep transverse intermetatarsal ligment and insert medially into the base of the proximal phalanx and the oblique portion of the extensor hood apparatus.

What is the annual whole body radiation occupational dose limit? A. 10 rem (100 mSv) B. 15 rem (150 mSv) C. .5 rem (5 mSv) D. 5 rem (50 mSv)

Correct answer: (D) 5 rem (50 mSv).

For how long will most absorbable orthopedic implants retain sufficient strength? A. 4 weeks B. 10 weeks C. 14 weeks D. 8 weeks

Correct answer: (D) 8 weeks. Explanation: Most absorbable orthopedic devices will retain sufficient strength for up to 8 weeks, before there is a significant loss in strength. If one considers the healing process in bone, by the 8th week, bone callus has formed, and the fracture or osteotomy site is stable. In theory the implant is no longer needed.

What is the recommended lavage volume for a Gustilo grade III fracture? A. 1L B. 3L C. 6L D. 9L

Correct answer: (D) 9 L. Explanation: Gustilo grade III fractures should receive between 9 and 18 L of high-pressure pulse lavage. The same is true for Gustilo grade II fractures. Grade I fractures need only 3 L of fluid. High-Pressure lavage is essential in these situations. The mechanical effect is what is important more than the contents of the irrigation fluid. The use of antibiotic in the irrigation fluid is also debatable. If they are used, however, it only makes sense to use them in the final bag of irrigation fluid. (A) High-Pressure pulse lavage of 1 L is inadequate to provide effective debridement for a Gustilo grade III fracture. (B) High-Pressure pulse lavage of 3 L is inadequate to provide effective debridement for a Gustilo grade III fracture. However, it is the recommendation for a Gustilo grade I fracture. (C) High-Pressure pulse lavage of 6 L is inadequate to provide effective debridement for a Gustilo grade III fracture.

A 35 y/o patient presents with a history of a tendo-Achilles rupture incurred eighteen months ago, treated with an A/K cast for ten weeks. He now has problems with push-off, the leg tires easily, and there is pain in the leg. An MRI shows an 8cm defect in the tendon. The best surgical procedure is: A. Direct end-to-end repair B. V-to-Y repair C. Peroneus brevis tendon transfer D. Achilles tendon turndown with FHL augmentation

Correct answer: (D) Achilles tendon turndown with FHL augmentation. Explanation: Defects greater than 5-7cm require an Achilles tendon turndown procedure with FHL transfer to augment, or an allograft replacement of the Achilles tendon. The FHL is a long tendon (10-12cm), able to bridge large defects. Also, the FHL axis is close to the axis of the tendo-Achilles, and the FHL fires in phase with the gastrocnemius-soleus. In the Kuwada classification, a defect greater than 6cm is a Type IV injury (I-IV), requiring a gastrocnemius recession, free tendon graft, and/or a synthetic graft. Free tendon grafts include the fascia lata, patellar tendon-bone graft and rectus femoris tendon-bone grafts. Synthetics or allografts include polyglycol threads, marlex mesh, dacron vascular graft, carbon fiber, or allograft tendon substitute. (A) A gap of 1-2cm can usually be treated with end-to-end repair w/ or w/o tenodesis augmentation. "If the defect is less than approximately 3cm and the injury occurred within the past three months, end-to-end repair is often possible" (Wapner, K.). (B) A gap of 2-5cm can be treated with a V-to-Y advancement with or without tenodesis augmentation. Measure the gap, then double the length of the portion to be advanced. The apex of the V is proximal. Some surgeons have used this procedure for defects greater than 5cm; if done, an FHL tendon transfer is required. (C) The tendon is harvested at its insertion. Once brought back to the tendo-Achilles, a medial- to-lateral calcaneal drill hole is created, through which the tendon is passed and secured to the Achilles tendon. Criticism for this technique includes loss of lateral stability of the ankle, and an overpowering of its antagonist, the tibialis posterior.

Which of the following statements regarding phenol is correct? A. Phenol 84% is most commonly used. B. It has low toxicity. C. It is insoluble in alcohol and soluble in water. D. Alcohol dilutes, but does not neutralize phenol.

Correct answer: (D) Alcohol dilutes, but does not neutralize phenol. Explanation: It is a common misconception that alcohol neutralizes phenol. This in fact is not the case. The reason alcohol is is used in a phenol matrixectomy procedure is that like phenol, alcohol is hydrophobic. This is in stark contrast to saline which is hydrophilic. In other words phenol is soluble in alcohol and insoluble in water. Therefore, alcohol can effectively dilute and wash away the phenol from the surgical site. That is also why it is recommended to use copious amounts of alcohol. If in fact alcohol neutralized phenol then only enough alcohol to equal the amount of phenol used would be necessary. The most commonly used concentration of phenol is 89%. In contrary, it is highly toxic and must be handled with care.

Which of the following mechanisms is responsible for the systemic hypotension seen in patients receiving propofol as an anesthetic? A. Direct myocardial depression B. Increased arterial vasodilation C. Increased venodilation D. All of the above

Correct answer: (D) All of the above. Explanation: The use of propofol as an anesthetic causes multiple systemic effects outside of the desired anesthetic effects. Propofol produces dose-dependent respiratory depression, it has anticonvulsant properties, and also leads to a systemic reduction in blood pressure. The hypotensive effects of propofol are produces through three main mechanisms. Propofol has a direct effect on the myocardium, thought to be related to a decrease in intracellular calcium availability. Propofol also causes decreased vascular resistance to both arterial and venous smooth muscle through both direct effects on vascular smooth muscle as well as a decrease in sympathetic activity.

Female patient with a recent hallux valgus surgery of the left foot, in which you used a stainless screw, 3 weeks ago, presents complaining of a rash about the left foot emanating from the surgical site which is fully healed, as sutures were removed without incident. The rash is extending proximally, the rash started 2 weeks. Now she reports that the rash is now spreading systemically to the entire body. The rash consists of erythematous patches and plaques. No blisters or other systemic reaction present. What is your diagnosis and what is your treatment plan? Allergic contact dermatitis secondary to nickel allergy, oral prednisone Erysipelas, admit to the hospital and start IV antibiotics Suspect psoriasis referral to a dermatologist Allergic contact dermatitis surgical secondary to Nickel allergy removal of the stainless steel screw

Correct answer: (D) Allergic contact dermatitis surgical secondary to Nickel allergy removal of the stainless steel screw. Explanation: An allergic contact dermatitis to stainless steel implants is an infrequent and poorly understood occurrence. Stainless steel screws are commonly used for internal fixation in an array of podiatric procedures. These screws normally contain 14% nickel, 20% chromium, and 27% cobalt. Of these components, nickel is the most likely to cause an immune response. Nickel is the most common human metal allergen, representing 10% of all contact dermatitis cases worldwide. A greater prevalence has been seen in women. This has been attributed to the greater exposure to nickel-containing metals used in custom jewelry, makeup, belt buckles, zippers, perfumes, and garments. The screw should be removed immediately, and the rash will start to subside, usually within 48 hours. A patch test should be ordered, as it is the reference standard to diagnose metal sensitivity. It is not practical to perform a patch test on every patient who is a surgical candidate but only to those with a history of nickel allergy. The use of prednisone will not eliminate the rash, with the implant still present in the body, it may be considered for 21 days if the removal of the implant is contraindicated.

Which of the following are required to appear on an informed consent form for surgery? A. Copy of photo ID B. Blood type C. Approval for blood transfusion D. Alternatives to the procedure E. Three witness signatures, a patient, and a surgeon signature

Correct answer: (D) Alternatives to the procedure. Explanation: Informed consent should contain: • The diagnosis, if known • The nature and purpose of a proposed treatment or procedure • The risks and benefits of proposed treatment or procedures • Alternatives (regardless of costs or extent covered by insurance) • The risks and benefits of alternatives • The risks and benefits of not receiving treatments or undergoing procedures • Name and signature of the patient, or if appropriate, legal representative • Name of the hospital • Name of procedure(s) • Name of all practitioners performing the procedure and individual significant tasks if more than one practitioner • Risks • Benefits • Date and time consent is obtained • Statement that procedure was explained to patient or guardian • Signature of person witnessing the consent • Name and signature of person who explained the procedure to the patient or guardian

If a patient experiences local anesthetic-induced ventricular dysrhythmias, what is the drug of choice? A. Clonidine B. Procainamide C. Lidocaine D. Amiodarone

Correct answer: (D) Amiodarone. Explanation: Amiodarone is the drug of choice for LA-induced ventricular dysrhythmias. Clonidine is a drug used to treat hypertension. It is not an antiarrhythmic drug. Procainamide and lidocaine are both antiarrhythmic drugs for ventricular arrhythmias but are second-line treatments to amiodarone for this condition.

Taking a critical look at the post op radiograph below, which of the following statements is true? A. Tibial sesamoid position is position 3. B. The first ray is too short. C. Alignment of the first metatarsophalangeal joint is congruous. D. An interpositional bone graft is present.

Correct answer: (D) An interpositional bone graft is present. Explanation: The radiograph above shows a salvage of a Lapidus nonunion. An interpositional bone graft was utilized and the extended medial column fusion was fixated with a monolateral external fixator. In this case the tibial sesamoid position is a 5-6. In terms of the length of the first ray, the interpositional graft has allowed an excellent maintenance of length. In the instance above the first and third metatarsal are approximately the same length which is ideal. In contrast, when the articulation of the first metatarsophalangeal joint is examined critically it can be seen that although the proximal phalanx is well aligned the valgus deviation of the metatarsal head leads to an incongruous articulation.

The tension band wiring technique would be an indicated fixation technique in which of the following? A. Dorsiflexory wedge osteotomy of the first metatarsal B. Akin Osteotomy C. Sesamoid fracture D. Avulsion fracture of the fifth metatarsal

Correct answer: (D) Avulsion fracture of the fifth metatarsal. Explanation: Tension band wiring is a preferred method of fixation for an avulsion fracture of the fifth metatarsal. With this example of an avulsion fracture as well others the key point is that the fracture fragment is so small that it does not allow for compression screw fixation or tension band plating. Although cerclage wire can be used in the fixation of the dorsiflexory wedge osteotomy of the first metatarsal and akin osteotomy it is not a tension band technique. Tension band wiring fixation is neither indicated nor possible in fixating a sesamoid fracture. The critical requirement of the fracture complex is a definite mechanism of counterpull or resistance to the compressive forces of the tension band wire. This is not afforded in the sesamoid fracture.

During fixation of a long bone fracture, this plate is used to push bone fragments together, producing interfragmentary compression with a pushing action. During screw application with this particular plate, elastic deformation of the plate is converted into anti-shear and compression forces. This plate can be applied without compression. Which of the following is the appropriate plate? A. LCD-plate B. Delta plate C. DCP- plate D. Buttress plate

Correct answer: (D) Buttress plate. Explanation: The ideal application of a buttress device in fracture fixation acts to push bone fragments together, thereby producing interfragmentary compression with a pushing action. Appropriate contouring facilitates the compression effect of a buttress plate as, during screw application, elastic deformation of the plate is converted into antishear and compression forces.

A 25 years old female presents for surgical correction of elongated 2nd metatarsal, with past medical history of muscular dystrophy, the following test should be considered in the perioperative period: A. MRI of the brain B. The Nerve conduction studies C. Electromyography D. Cardiac testing

Correct answer: (D) Cardiac testing. Explanation: Muscular dystrophies are a series of muscles affecting joint and tendons, causing limitations of passive and active movements. Pain will result in abrupt give-away pattern. The lesions are commonly found in the upper and lower motor neurons. Muscular dystrophy frequently manifests in young people who may otherwise have a reason to have cardiac testing. These myopathies frequently have primary cardiac manifestations which may compromise cardiac function. (A) MRI of the brain can be used to diagnose conditions like Multiple sclerosis, a chronic illness, which presents with multiple areas of white matter inflammation, demyelination, and sclerosis. Manifests clinically as at least two neurological problems separate in space and time; this is quite different than a muscular dystrophy. The MRI usually shows "target appearance" on T1, bright signal on T2, MRI is the gold standard for multiple sclerosis. (B) The nerve conduction studies are serial testing for peripheral nerve injury; they test sensory, motor and mixed components. (C) Electromyography also studies peripheral nerve injury, studying motor unit action potentials.

Which type of study works by first identifying the diseased and non-diseased individuals and then ascertain the frequency of certain exposures? A. Case report B. Cohort study C. Randomized trial D. Case-control study

Correct answer: (D) Case-control study. Explanation: A case control study works backwards in the sense that it first identifies the diseased and non-diseased individuals and then they ascertain the frequency of certain exposures. They are useful in studying diseases with rare outcomes as well as those with long latency periods. They are also useful in studying multiple risk factors for a disease. Their disadvantages include being an observational study and the recall bias that may distort associations.

Which of the following are benefits of wound debridement? (select all that apply) A. Removal of necrotic tissue B. Cicatrix formation C. Increasing platelet and growth factor migration into wound D. Choices A & C are correct

Correct answer: (D) Choices A & C are correct. Explanation: The benefits of wound debridement include removal of necrotic tissue, removal of bacteria, and migration of platelets and endogenous growth factors into wound through local stimulation.

During pre-admission testing for a flatfoot reconstruction, the laboratory results show increased levels of T3, T4, and T3 and suppressed levels of TSH. You referred the patient to a primary care provider, and postponed the surgery until the patient has had further studies and started on a regimen for hyperthyroidism. You explain to your patient the importance of treating hyperthyroidism as it may increase mortality due to the following effect it has on the cardiovascular system. A. Deactivate the renin-angiotensin- aldosterone system. B. Decrease cardiovascular output. C. Decrease circulating blood volume. D. Chronically elevated thyroid hormones could cause cardiovascular collapse during surgery because of increased stress. E. Chronically elevated thyroid hormone causes an increment in systemic vascular resistance and blood pressure.

Correct answer: (D) Chronically elevated thyroid hormones could cause cardiovascular collapse during surgery because of increased stress. Explanation: T3 and T4 have direct inotropic and chronotropic effects on the heart. In addition, they have a direct effect on vascular smooth muscle causing decrement in systemic vascular resistance and blood pressure. As a result, the renin-angiotensin- aldosterone system is activated, enhancing sodium reabsorption and increasing circulating blood volume therefore increasing cardiac output by 50 to 300%. So chronically elevated thyroid hormones may limit the ability of patients undergoing surgery to respond to stress and lead to cardiovascular collapse. So it is important to diagnose and treat hyperthyroidism, and be well controlled prior to surgery, as failure to identify and treat can drastically increase mortality.

A 9 year old male is requiring a first metatarsal-cuneiform arthrodesis for a congenital met primus varus. Suitable fixation for the above arthrodesis would include all of the following except? A. Smooth Steinmann pins B. Crossing Kirschner wires C. Plate and screw fixation D. Compression screw fixation

Correct answer: (D) Compression screw fixation. Explanation: The key point to this question is to recognize at 9 years of age an open growth plate is present at the base of the first metatarsal. Compression across the growth plate is contraindicated as it will damage the growth plate and cause an early cessation of metatarsal growth. This is why answer D is the correct choice. Steinman pins and Kirschner wires only stabilize the arthrodesis site and do not compress the growth plate. Likewise, the plate and screw construct can be fashioned as to only stabilize the joint and to not compress or violate the open growth plate.

All the following are true statements regarding fractures of the lateral process of the talus except: A. If untreated carries a high incidence of ankle and subtalar joint arthritis. B. Results from an acute dorsiflexion and eversion of the foot. C. Is associated with the sport of snowboarding. D. Conservative treatment consists of non-weight-bearing for 4 weeks followed by early range of motion.

Correct answer: (D) Conservative treatment consists of non-weight-bearing for 4 weeks followed by early range of motion. Explanation: Fractures of the lateral process of the talus has been termed "snowboarder's fracture" as a result of the high occurrence of these fractures in this patient population. The mechanism of injury involves forced dorsiflexsion and eversion of the hind-foot, which is a common maneuver in aerial maneuvers in snow boarding. These injuries should be treated as soon as possible to avoid complications, some of which include arthritis, non-union, avascular necrosis, and bone impingement.

A 7 year old boy reports with his parents to the ER six hours after stepping on a rusty nail. The nail punctured his left foot. The wound created is classified as which type of wound? A. Clean B. Clean-contaminated C. Contaminated D. Dirty

Correct answer: (D) Dirty. Explanation: Dirty wounds are those that include retained revitalized tissue, foreign bodies, fecal contamination, or are greater than 4 hours old. The wound above is 6 hours old and therefore is classified as a dirty wound. (A) Clean wounds are primarily surgical wounds. (B) Clean-contaminated refers to areas of the respiratory, genitourinary system and gastrointestinal tract where although the tissues are within the body there is communication with the external environment. (C) Contaminated wounds are defined as wounds created by a fresh traumatic injury less than 4 hours old.

A 50 y/o patient with RA is scheduled for hammertoe surgery; 2-5 left foot. One of his medications currently utilized is methotrexate. The proper protocol for use in the peri-operative period is: A. Discontinue one week pre-op; resume one week post-op. B. Discontinue one week pre-op; resume two weeks post-op. C. Discontinue two weeks pre-op; resume two weeks post-op. D. Do not discontinue methotrexate.

Correct answer: (D) Do not discontinue methotrexate. Explanation: In the past, the concerns with methotrexate were impaired wound healing and increased incidence of infection. Current thinking is the patient can continue on the prescribed dosing pattern, without interruption; there is not an increased incidence of these two complications. In fact, patients fare better in the post-operative period being maintained on methotrexate, as opposed to discontinuing.

A 39 year old female presents complaining of pain and swelling in her right foot for 2 weeks duration. She denies any trauma, change in activity or shoe gear. Approximately one year earlier a Lapidus procedure was performed on her for a hallux valgus deformity. After clinical and radiographic exam a diagnosis of a second metatarsal stress fracture has been confirmed. What is the likely position of the first ray? A. varus (undercorrected) B. valgus (overcorrected) C. plantarflexed D. dorsiflexion

Correct answer: (D) Dorsiflexion. Explanation: The most common malposition following a Lapidus procedure is a position of dorsiflexion. Sagittal plane malposition is fraught with problems. In the question above the first ray is most likely positioned in a dorsiflexed position. The key to this question is realizing the sequela of a dorsiflexed malposition. The elevated position of the first ray will lead to transfer metatarsalgia. It is this increased load onto the second metatarsal, as a result of the elevated first ray, that fatigues and creates the stress fracture of the second metatarsal. In contract, the undercorrected intermetatarsal angle or varus malposition of the first ray is generally well tolerated, but not always acceptable by the patient. Plantarflexion of the first ray can lead to plantar callosities and/or sesamoiditis.

A calcaneocuboid distraction arthrodesis is a modification of the Evans osteotomy. What is the major advantage of the calcaneocuboid arthrodesis over the Evans osteotomy? A. The arthrodesis is more suitable for the obese population. B. More correction can be obtained. C. Less chance of nonunion. D. Eliminates the possibility of cacalneocuboid joint arthrosis.

Correct answer: (D) Eliminates the possibility of cacalneocuboid joint arthrosis. Explanation: A modification of the Evans osteotomy is the calcaneocuboid arthrodesis. This procedure, like the Evans, also corrects the forefoot abduction and the rearfoot valgus while maintaining significant amount of hindfoot motion, unlike the fusion of the subtalar joint or the talonavicular joint. This technique eliminated the complications of arthritic changes in the calcaneocuboid joint with the Evans osteotomy. The chances of a malunion with the arthrodesis are higher than that of the osteotomy since you are working on two separate bone to graft fusion sites versus a bone graft within an osteotomy of a highly vascular bone. The amount of correction that can be obtained is similar for both procedures.

The runout is the weakest portion of the AO/ASIF screw. What should be done regarding the runout and fracture fixation using lag screw technique? Place the runout at the site of increased bending force, which will inherently reinforce the runout Allow shear force to stabilize the screw Employ a short enough screw to place the runout as close as possible from the fracture line Employ a long enough screw to place the runout as far as possible from the fracture line

Correct answer: (D) Employ a long enough screw to place the runout as far as possible from the fracture line. Explanation: The site where the shaft meets the threads is referred to as the runout, and it is the weakest point of a partially threaded screw. The runout should be shielded from bending and shear forces that may arise at a fracture or osteotomy site. The surgeon should employ a long enough screw to place the runout portion as far as possible from the fracture line. The core of the screw refers to the portion of the shaft extending within the thread pattern. Increasing the width of the core allows the screw to be more resistant to bending loads.

Which of the following procedures addresses the mid tarsal "locking mechanism"? A. Arthroereisis of the subtalar joint B. Medial displacement calcaneal osteotomy C. Young's tenosuspension D. Evans calcaneal osteotomy

Correct answer: (D) Evans calcaneal osteotomy. Explanation: The Evans calcaneal osteotomy improves the mid tarsal locking mechanism as well as restores the medial longitudinal arch and reduces handoff valgus. To evaluate this pre- operatively, stabilize the heel with one hand and attempt to load the forefoot. If the mid tarsal joint doesn't lock then the Evans calcaneal osteotomy is indicated. If it does lock than another procedure is better suited. The Evans anterior calcaneal osteotomy is indicated of late stage II adult acquired flatfoot and flexible adolescent flatfoot. It is contraindicated in those demonstrating radiographic evidence of degenerative changes within the tritarsal complex and in patients with rigid, non reducible deformities.

Critical analysis of the bunion correction in the picture below reveals which of the following? A. Adequate correction of the hallux valgus deformity B. Nonunion of the phalangeal osteotomy C. First metatarsal length was maintained post bunionectomy D. Inadequate correction of the hallux valgus deformity

Correct answer: (D) Inadequate correction of the hallux valgus deformity. Explanation: The radiographs above are pre and post Lapidus with phalangeal osteotomy. Clearly demonstrated above is inadequate correction of the hallux valgus deformity. This is indicated by a lack of sesamoid reduction. Tibial sesamoid position in the preoperative photo is a 6 and in the post operative photo has only been reduced to a 4. The is due to the lack of intermetatarsal angle correction. There is successful union of both the phalangeal osteotomy and the first metatarsal-cuneiform arthrodesis. There also has been significant shortening of the first metatarsal.

In the practice of callus distraction for bone lengthening there is some debate on the placement of the osteotomy being placed in the metaphyseal versus the diaphyseal portion of the bone. Which statement regarding placement of the metaphyseal portion of the bone is true? A. offers easier manipulation of the periosteum B. less mechanical strength C. less metabolic activity D. increased vascularity and osteogenic potential

Correct answer: (D) Increased vascularity and osteogenic potential. Explanation: When performing callus distraction for bone lengthening the osteotomy can be placed either in the diaphysis or metaphysis of the bone. Points of consideration when placing the osteotomy in the diaphysis include: easier access; allows for a well defined osteotomy; offers easier manipulation of the periosteum; less mechanical strength; less metabolic activity. Points of consideration for placing the osteotomy in the metaphysis include: greater surface area means greater stability and strength; increased vascularity and osteogenic potential; more difficult to manipulate the periosteum as it is much thinner.

During fracture fixation using lag screw technique, you perform an aggressive countersinking. What step can you take to prevent excessive advancement of the screw? A. Drill a second hole B. Add a kirschner wire C. Add an extra week of post-operative non weight bearing D. Insert a washer

Correct answer: (D) Insert a washer. Explanation: Aggressive countersinking can weaken cortical bone; this may allow screw advancement into underlying cancellous bone as the screw is tightened. Should this occur, a washer should be inserted to increase surface area under the screw, to prevent excessive advancement.

A 38 year old female presents to clinic complaining of pain in the ball of her left foot. Physical exam a moderate hallux valgus deformity with a hypermobile first ray. Additionally sagittal plane contracture is noted of the second and third digits and an adductovarus deformity is noted on the fourth and fifth digits. There is a medial deviation of the second digit, however the second digit does not contact nor underlap to overlap the hallux. There is a somewhat diffuse painful area under the third and fourth met head and in the 3rd interspace. Additionally there is pain with a side to side compression of the right forefoot. Below is a ultrasound image of the patient's right foot. With the information provided and the clinical image below, what would be the provisional diagnosis to explain the patient's pain in the ball of her right foot? A. metatarsal stress fracture B. predislocation syndrome C. capsulitis third tmp joint D. interdigital neuroma

Correct answer: (D) Interdigital neuroma. Explanation: The foot described above is a foot prone to many problems. It is overpronated foot with a hypermobile first ray. It has led to a hallux valgus deformity and pathology related to lateral overload of pedal pressure due to the insufficiency of the first ray to bear its portion of weight. All the conditions listed above are possible with this foot type. However the key component in arriving to the correct answer lies in the diagnostic ultrasound picture. Highlighted by the yellow arrow is a sizable neuroma in the third interspace (Morton's neuroma). It can be seen protruding planetary beyond the metatarsal heads and occupying the entirety of the interspace. In contrast a metatarsal stress fracture cannot be diagnosed through this modality. Pre dislocation syndrome would be highly unlikely in this part of the foot. It typically affects the second tmp joint. Capsulitis of the third tmp joint is not visualized above. Thickening of the plantar capsule and/or accumulation of fluid under the metatarsal head is not present.

Dislocations of the first metatarsophalangeal joint in children are almost always treated by closed reduction. From the options below, what is a possible indication for open reduction? A. Dorsal dislocation greater than 5 mm B. Plantar dislocation C. Abductor tendon rupture D. Interposition of the sesamoid apparatus

Correct answer: (D) Interposition of the sesamoid apparatus. Explanation: Soft tissue interposition of the sesamoid apparatus can force the surgeon to convert the reduction from closed to open. (A) Dorsal dislocation of the proximal phalanx on the first metatarsal greater than 5 mm is not an indication for open reduction. (B) Plantar dislocation of the proximal phalanx on the first metatarsal is not an indication for open reduction. (C) Rupture of the abductor tendon is not an indication for open reduction.

When using a power saw or drill, it is important that the surgeon ensure that excess thermal energy does not build up in the bone. Why should burning of bone be avoided? Additional inflammation causes a decrease in phosphate and displacement of calcium in the bone matrix. It decreases phagocystosis at operative site. It results in necrotic bone in which white blood cells fail to migrate. It causes an increased zone of resorption at the osteotomy site.

Correct answer: (D) It causes an increased zone of resorption at the osteotomy site. Explanation: Thermal injury to the bone from sawing or drilling causes increased resorption of bone. In an osteotomy this leads to widening of the osteotomy and lack of apposition of the osteotomy surfaces. In drilling of bone the resorption leads to collapse of bone under the screw head and around the threads. This ultimately leads to premature loosening of the fixation. Thermal injury of bone does not affect phosphate or calcium metabolism in bone. Additionally the second and third choices essentially say the same thing but with different wording and both are not accurate.

What is the hallmark radiographic finding of skewfoot? An increase in the talocalcaneal angle in both the anterior-posterior and lateral projections Fixed calcaneal valgus Forefoot adduction Lateral subluxation of the navicular on the talar head

Correct answer: (D) Lateral subluxation of the navicular on the talar head. Explanation: The radiographic hallmark of skewfoot is lateral subluxation of the navicular on the talar head. This might be hard to see in the very young due to the late ossification of the navicular. (A) Although an increase in the talocalcaneal angle in both the anterior-posterior and lateral projections is present in the skewfoot, it is not the radiographic hallmark. (B) Although fixed calcaneal valgus is present in the skewfoot, it is not the radiographic hallmark. (C) Although forefoot adduction is present in the skewfoot, it is not the radiographic hallmark.

What local anesthetic property determines potency of the local anesthetic? A. protein binding B. pKa C. ionic potential D. lipid solubility

Correct answer: (D) Lipid solubility. Explanation: The physical and chemical properties that influence local anesthetic activity are lipid solubility, protein binding, and the physical-chemical parameter (pKa). Lipid solubility determines the potency of the local anesthetic. This is logical if one thinks in term the mechanism of action of local anesthetics. The more lipid soluble the anesthetic molecule is, the more easily it can penetrate the perineurium and cellular membrane to gain entry into the nerve cell and cause depolarization. That would mean that a smaller dose could provide a deeper degree of anesthesia. An example of this phenomenon is the addition of a butyl group onto mepivicaine. This leads to the formation of bupivicaine which is 28 more times lipid soluble and 4 times more potent than mepivicaine.

Which primary ossification center would be expected to appear after that of the cuboid? A. Talus B. Calcaneus C. 3rd metatarsal D. Navicular

Correct answer: (D) Navicular. Explanation: The primary ossification center of the cuboid appears between the sixth fetal month and one year. The ossification center of the navicular appears three months to five years after birth. (A) The primary ossification center of the talus appears between 3-5 fetal months and two months post-natal. (B) The primary ossification center of the calcaneus occurs between the third fetal month and one month post-natal. The calcaneus also has a secondary ossification center which generally appears between 5-12 years old. (C) The primary ossification center of the metatarsals appear between 2-4 fetal months. The metatarsals may have secondary ossificationc enters which appear between six to 24 months.

A 34 year old female presents for follow up consultation complaining of pain in the ball of her left foot. Her complaint is somewhat vague. She simply states it hurts when she walks. Denies trauma. Pt reports pain has been present for about six months and continues to get worse. Pt states the only thing that seems to make the pain better is when she is able to sit down and elevate her feet. Radiographs are negative for osseous pathology and pedal joint alignment is normal with the exception of a mild increase in the first inter metatarsal angle and hallux abductus angle. Initial treatment involved fitting and dispensing metatarsal pads and regular use of an NSAID. She reports mild relief with the initial treatment, but by the middle of the day, she reports the pain is almost unbearable. An MRI is ordered for further information. A representative image is below. What is the most likely diagnosis? A. capsulitis B. ganglionic cyst C. metatarsal stress fracture D. neuroma

Correct answer: (D) Neuroma. Explanation: The key to this question is the ability of the clinician to interpret the T1-weighted MRI image. The image above is most indicative of a morton's neuroma. It is a solid, uniform mass located in the third interspace, specifically between the third and fourth metatarsophalangeal joints. In the MRI image above, there is no identifiable loss of cortical integrity or edema with a metatarsal to suggest a metatarsal stress fracture. Furthermore, the third inter metatarsal space is an unlikely location for a ganglionic cyst and would not have such a uniform appearance. Lastly, capsulitis is unlikely as not diastasis is noted within the joints and there is no evidence of thickened synovial on the image above.

Placing an MBA implant that is too large in the appropriate position within the sinus tarsi will result in: A. Under-correction with pronated foot B. Under-correction with supinated foot C. Over-correction with pronated foot D. Over-correction with supinated foot

Correct answer: (D) Overcorrection with supinated foot. Explanation: If an MBA implant is too large within the sinus tarsi, it will result in overcorrection. This will result in a supinated foot with restricted motion preventing normal pronation. Normal joint function requires 4-5 degrees of calcaneal eversion. (A) Under-correction is more likely to occur with a smaller implant and may result in a pronated foot that is not fully corrected with the procedure. Undercorrection is generally conservatively managed with orthotic control. (B) A supinated foot with under-correction may be possible if the MBA implant is misplaced far medial within the sinus tarsi. (C) A pronated foot with over-correction may be possible if the MBA implant is misplaced far lateral within the sinus tarsi. This would lead to further pathologic pronation.

In regards to reduction goals of metatarsal fractures which of the following statements is true? A. Medial displacement of the capital portion is not well tolerated. B. Lateral displacement of the capital portion is not well tolerated. C. Dorsal displacement of the capital portion is well tolerated. D. Plantar displacement of the capital portion is not well tolerated.

Correct answer: (D) Plantar displacement of the capital portion is not well tolerated. Explanation: The metatarsal parabola and protrusion distance is an intricate orchestration of positions and pressure. The foot can tolerate medial and lateral displacement of the capital portions quite well, but whether referring to surgical position or reduction position, malposition in the sagittal plane is fraught with problems. Regardless of plantar malpositioning or dorsal malpositioning, plantar foot pressures become unevenly distributed. This results in pressure related problems such as calluses, stress fractures and pre dislocation syndrome.

Which of the following is a generally acceptable indication for amputation? A. Heel spur B. Grade I diabetic ulcer C. Pilon fracture D. Polydactyly

Correct answer: (D) Polydactyly. Explanation: Polydactyly, or the presence of a supernumerary digit(s), is indeed an indication for amputation. In fact this is the only indicated surgical approach for this condition. A Grade I diabetic ulcer is a relatively minor ulceration in the spectrum and by definition does not have infection nor necrosis so amputation would not be indicated. A heel spur albeit painful, is not an indication for an amputation. A pilon fracture is in itself not an indication for amputation. However in the setting of a poly trauma the pilon fracture might be part of a scenario where a Mangled Extremity Severity Score is high enough to indicate primary amputation.

When discussing intravenous anesthetics, what agent of intravenous anesthesia is best described as an agent that facilitates inhibitory neurotransmission mediated by GABA. It has a high lipid solubility which results in a rapid action and redistribution. When injected intravenously its actions only last minutes. It is an emulsion of soybean oil, glycerol and egg lecithin. What intravenous anesthetic agent is being described? A. Benzodiazepines B. Ketamine C. Etomidate D. Propofol

Correct answer: (D) Propofol. Explanation: Propofol is a short-acting, intravenously administered hypnotic/amnestic agent used for the induction of anesthesia. Its high lipid solubility attributes to this rapid onset of action as well as its short duration secondary to redistribution. A bolus of propofol will only produce effects that last for minutes. It does not cause nausea. Its rapid onset, short duration and amnesic effect have made it a very popular agent used in anesthesia. Pharmacokinetically it is metabolized by the liver. Its mechanism of action is that of a potentiator of GABA receptor activity and as a sodium channel blocker. Propofol is prepared as an emulsion of propofol, glycerol, soybean oil and egg lecithin.

What is the best surgical strategy for neuroma resection of adjacent interspaces on a patient with marginal lower extremity arterial circulation? A. Utilization of two separate incision spaces as far apart as possible B. Utilization of a single incision curved as it crosses the metatarsophalangeal joint C. Treat one neuroma by excision and the other non surgically D. Resect the more painful neuroma first, then resect the second neuroma 1-2 months after the primary incision has healed

Correct answer: (D) Resect the more painful neuroma first, then resect the second neuroma 1-2 months after the primary incision has healed. Explanation: When circulation is marginal, the more painful neuroma is excised first, and then the adjacent interspace neuroma is resected 1 to 2 months after the primary incision has healed. This prevents the foot's vascularity from being overwhelmed by the extensive dissection that is required with simultaneous adjacent neuroma resection. (A) Even though the surgeon would space the two separate incisions as far as possible, the degree of dissection required would significantly increase the chances of vascular embarrassment to the marginal blood supply. (B) With a single incisional approach for adjacent neuroma excision, extensive dissection and tension on the soft tissues is required. This could very well lead too great of a vascular insult. (C) A this point in the decision making process, all conservative measures have been exhausted before surgical intervention was considered. Non surgical treatment would be fruitless.

A 61 year old male presents for follow up three weeks status post an Austin osteotomy. He developed an early post-operative infection, which despite initially powerful, broad spectrum antibiotics and more recently culture-specific antibiotics, has failed to resolve. Most of the cellulitis is gone, however some redness and some serosanguineous drainage from the incision remain. Due to presence of a pacemaker and the recent nature of the surgery an MRI is contraindicated. This being the case a technetium and gallium nuclear scan are ordered. The technetium can come back positive. Likewise, the gallium scan comes in positive. What is the diagnosis? A. Chronic osteomyelitis B. Cellulitis C. Osteitis D. Septic joint

Correct answer: (D) Septic joint. Explanation: When an MRI is not possible, nuclear imaging is a viable alternative. When trying to discern between soft tissue infection from deeper bone infection, the combination of gallium and technetium scans are most informative. When both technetium and gallium are positive, the diagnosis is either acute osteomyelitis or a septic arthritis. When technetium is positive and gallium is negative, chronic osteomyelitis is present. Lastly, when a technetium scan is negative and a gallium scan is positive, the diagnosis is cellulitis. Technetium is a bone-imaging radionuclide while gallium is an inflammatory-imaging radionuclide. Gallium binds to white blood cells, plasma proteins, transferrin, ferritin, lactoferrin and siderophores and travels to areas of inflammation. Technetium binds to hydroxyapatite crystals and travels to area of high bone turnover.

A 12 year old boy presents to the office for follow up on calcaneal apophysitis of his right heel. Over the last three weeks he has tried elimination of athletic activity, icing, heel cups and naproxen all without success. An MRI was ordered last week, when he was not seeing relief, and it confirmed the diagnosis. What would be the next appropriate treatment? A. corticosteroid injection B. achilles tendon lengthening C. endoscopic plantar fasciotomy D. short-leg nonweigthbearing (NWB) cast

Correct answer: (D) Short-leg nonweigthbearing (NWB) cast. Explanation: The initial treatment of calcaneal apophysitis must include discontinuation of running and jumping activity until symptoms abate. Likewise, supportive strapping, heel cups, and intermittent icing can be helpful for symptom reduction. Finally, a short-leg cast immobilization for 2 to 4 weeks remains the "gold standard" in cases of refractory to less restrictive measures.

In regards to skin flap physiology which term below is best defined as the the decrease in stress when skin is held in tension at a constant strain for a given time? A. Stress B. Strain C. Creep D. Stress relaxation

Correct answer: (D) Stress relaxation. Explanation: There are four important concepts pertaining to flap physiology: stress, strain, creep and stress relaxation. Stress relaxation is the decrease in stress when skin is held in tension at a constant strain for a given time. This occurs over a matter of days to weeks and is due to an increase in skin cellularity and the permanent stretching of skin components. Creep refers to the increase in strain seen when skin is under constant stress. This occurs in a matter of minutes and is due to an extrusion of fluid from the dermis and a breakdown of the dermal framework. Strain refers to the change in length divided by the original length of the given tissue to which a force is applied. Stress refers to the force plaid per cross-sectional area.

During contact phase in the gait cycle, what is the position and the motion at the subtalar joint? A. 4-5 degrees internal and 4-5 degrees internal rotation B. 0-2 degrees in external and 4-5 internal rotation C. 0 degrees in external and 4-5 degrees on internal rotation D. Supinated 2-4 degrees, and pronation 4-6 degrees E. Pronated 2-4 degrees and supination at 4-6 degrees

Correct answer: (D) Supinated 2-4 degrees, and pronation 4-6 degrees. Explanation: Pronation at the subtalar joint during this period is a passive event. Plantarflexion of the ankle is resisted by contraction of the pretibial muscle group. This supinates the longitudinal axis of the midtarsal joint, which in open chain (swing) supinates the subtalar joint. This results in an inverted attitude of the calcaneus relative to the floor.

A 42 year-old female demonstrates hyperextension at the proximal interphalangeal joints and with flexion at the distal interphalangeal joints. What is the term for this deformity? A. Bouchard's node B. Heberden's node C. Boutonniere deformity D. Swan neck deformity

Correct answer: (D) Swan neck deformity. Explanation: In rheumatoid arthritis, subluxation and dislocation of the lesser toes create an imbalance between the intrinsic and extrinsic muscles and gives rise to progressive hammering of the lesser toes. These deformities also exist in the hands. Hyperextension at the proximal interphalangeal joints with flexion at the distal interphalangeal joints results in a swan neck deformity, while flexion at the proximal interphalangeal joint and hyperextension at the distal interphalangeal joint results in a boutonniere deformity. Osteoarthritis is characterized by bony prominences at the distal interphalangeal joints (Heberden's nodes) and at the proximal interphalangeal joints (Bouchard's nodes).

What is the best radiographic view to appreciate the presence of the middle talocalcaneal coalition? A. The frontal view B. The oblique view C. The medial oblique view D. The Harris-Beath view E. The Isherwood view

Correct answer: (D) The Harris-Beath view. Explanation: The middle talocalcaneal coalition is best appreciated in the axial view or Harris- Beath view. It will show the joint space obliterated if osseous coalition is present, gross joint space narrowing and subchondral sclerosis are seen if fibrocartilaginous. If fibrocartilaginous, the middle joint appears obliquely oriented, at 45 degrees. When the middle talocalcaneal calcaneal coalition is present, this landmark frequently appears rounded or even flattened inferiorly. Also, the shadows of the sustentaculum tali and the talar posteromedial process are normally separate and distinct.

These bones and articulations are not part of the ankle joint: A. The distal articular surface of the fibula. B. The distal articular surface of the tibia. C. The trochlear surface of the talus. D. The calcaneo-talar articular surface.

Correct answer: (D) The calcaneo-talar articular surface. Explanation: The subtalar joint. The bones of the ankle joint include the distal articular surfaces of the tibia and the fibula and the trochlear surface of the talus. The trochlear surface of the talus is depressed centrally, is broader anteriorly than posteriorly, and corresponds to a reciprocally shaped surface on the inferior aspect of the tibia. The medial and lateral surfaces of the body of the talus are seated in the ankle mortise by the two malleoli. The lateral malleolus is larger than the medial malleolus and extends further distally than the medial malleolus.

Which of the following is defined as the area of an AO/ASIF partially threaded screw that is void of threads and is located between the head and the beginning of the threads? A. The runout B. The core diameter C. The pitch D. The screw shaft

Correct answer: (D) The screw shaft. Explanation: Screw shaft refers to the region void of thread pattern between the screw head and the beginning of threads. The site where the shaft meets the threads is referred to as the runout, and it is the weakest point of a partially threaded screw.

Which of the following statements regarding lag screws is correct? A. Cortical screws come in only full threaded versions. B. A lag screw purchases both the proximal and distal cortices. C. Cancellous screws have a more narrow pitch. D. The thread must not cross the fracture line.

Correct answer: (D) The thread must not cross the fracture line. Explanation: The threads of a lag screw cannot cross the fracture/osteotomy line. If the threads do cross the fracture line then distraction of the fracture line will occur. Additionally with a lag screw the screw must pass freely and not engage the proximal cortex and only engaged the distal cortex. This will allow the screw to pull the distal fragment toward the proximal fragment as the screw head engages the proximal cortex. In regards to pitch of screw, it is the cortical screw that has the more narrow pitch to optimize the amount of thinner cortical bone that is purchased and cancellous screws have a wider pitch to grab more bone. Regarding the first statement, Cortical screws come in both full and partially threaded versions.

Which is the most important value of hallux proximal phalangeal osteotomies? A. They can shorten the hallux. B. They are indicated as an adjunct to correction for hallux abducto valgus. C. They correct hallux interphalangeus. D. They correct abnormal distal articular set angle. E. They correct abnormal hallux interphalangeal angle.

Correct answer: (D) They correct abnormal distal articular set angle. Explanation: Hallux proximal phalangeal osteotomies are not indicated for primary correction of hallux valgus. Rather, they are adjunct procedures. Although they do shorten the hallux, they correct hallux interphalangeus, abnormal hallux interphalangeal angle, and abnormal distal articular set angle.

Which of the following statements is not true regarding digital fillet flaps? A. They have consistent vascular supply. B. They provide durable soft tissue coverage. C. They are neurovascular pedicle flaps. D. They provide coverage of defects over 20 square centimeters. E. They can cover weightbearing structures.

Correct answer: (D) They provide coverage of defects over 20 square centimeters. Explanation: Digital fillet flaps are neurovascular pedicle flaps that provide durable, well- vascularized soft-tissue coverage for a variety of wounds, even plantar and weightbearing areas. They are limited in use in the coverage of a combined mean area under 15.6 square centimeters.

A 30 year old male who had surgical resection of a calcaneonavicular bar at age 11 exhibits fixed valgus deformity. The patient experiences subtalar joint pain with range of motion. Radiographs demonstrate recurrence of the calcaneonavicular bar, subtalar joint degeneration and talar beaking. What is the most appropriate surgical treatment? A. resection of coalition with interposition of extensor digitorum brevis muscle B. subtalar joint arthrodesis C. resection of coalition with subtalar arthroereisis D. triple arthrodesis

Correct answer: (D) Triple arthrodesis. Explanation: Due to the fact that the patient is an adult and has a rigid deformity that is a result from previous surgery, a triple arthrodesis is indicated. A previous resection of coalition was already attempted which failed, therefore this would be incorrect. A subtalar joint arthrodesis would be incorrect, because the deformity is rigid. There is evidence of subtalar joint arthritis in the form of subtalar joint degeneration and talar beaking which is not an indication for a subtalar joint. A combination of the resection of coalition and subtalar arthroereisis would also be incorrect for the reasons previously.

Utilizing the Gustilo classification of open fractures, where would you classify the injury below? A. Type I B. Type II C. Type IIIA D. Type IIIB

Correct answer: (D) Type IIIB. Explanation: The injury pictured above is a clear-cut example of a Gustilo IIIB fracture. Gustilo III fractures are best characterized an open fracture, highly contaminated, with a wound > 5 cm. There is extensive soft tissue damage including skin, muscle, nerve, and vasculature. A large crush component is seen. Gustilo III injuries can further be subdivided into three variants. IIIA is when there is adequate soft tissue coverage and it is a high-energy trauma. IIIB is when soft tissue coverage is not adequate for coverage and there is extensive periosteal stripping of exposed bone. IIIC is an open fracture with arterial injury regardless of bone or soft tissue injury. Furthermore type I is an open fracture with a clean wound < 1 cm, with little soft tissue involvement and no crush component. The wound is usually from bone protrusion inside to out. The fracture is usually short, short oblique, or transverse. Type II is an open fracture moderately contaminated, with a wound > 1 cm. There is no extensive soft tissue damage, skin flaps, or avulsions. A minimal crush component and moderate fracture comminution only.

The biggest disadvantage to the Wilson V-Y skin plasty in the management of Digiti Quinti Varus is: A. Vascular compromise and loss of the toe B. Scar contracture C. Overcorrection D. Undercorrection.

Correct answer: (D) Undercorrection. Explanation: In most cases of Digiti Quinti Varus, the degree of deformity simply exceeds the amount of skin advancement possible by the V-Y procedure. Therefore undercorrection is the biggest disadvantage of the Wilson V-Y skin plasty. Although scar contraction can cause a non purchasing fifth digit, this is completely avoidable and preventable. Vascular compromise of the digit in the V-Y procedure is of little to no concern as the procedure is done no deeper than the subcutaneous tissue so no real threat of vascular compromise exists.

86-year-old female undergoes a fifth metatarsal amputation due to peripheral vascular disease. 5 days post-operative the patient spikes a fever of 101.5 degrees Fahrenheit. Respiratory rate is 20, heart rate is 100 beats per minute; blood pressure is 120/85 mm Hg and SaO2 is 97% on room air. Examination is significant for suprapubic tenderness and CVA tenderness on the right side. What is the next best workup to confirm the most likely diagnosis? A. CBC with Differential B. CMP C. Chest X-Ray D. Urinalysis with culture and sensitivity

Correct answer: (D) Urinalysis with culture and sensitivity. Explanation: The most likely cause of this patient's post-operative fever on day 5 post-op is a urinary tract infection, which may have occurred from foley placement during surgery. Foley catheters are often left in after surgery for a day or so, particularly in older patients who may experience post-operative urinary retention. The best test to diagnose this is a urinalysis, with urine culture and sensitivity. Urinalysis may reveal positive leukocyte esterase, white blood cells, and positive nitrite. Red blood cells may also appear in some urinary tract infections or in patients with irritation from a foley catheter. The urine culture will identify the organism and sensitivity and is useful to direct antibiotic therapy in hospitalized patients who may have acquired an infection from a resistant organism. Chest X-Ray (C) can commonly be used if a pneumonia or atelectasis is suspected, but this is a more likely cause of post-operative fever in the first 24-48 hours. CBC with differential (A) may show an elevated WBC count but will not help to confirm the diagnosis. CMP (B) would not provide much information to help diagnosis a UTI. The causes of post-operative fever can be recalled by the mnemonic "5 W's:" Wind---pneumonia, atelectasis usually causes post-operative fever at 1st 24- 48 hours Water---urinary tract infection as a cause of post-operative fever typically is seen after post op day 3 Wound---wound infections usually cause post-operative fever after post op day 5 Wonderdrugs---especially anesthesia Walking---walking can help reduce deep vein thromboses; pulmonary embolus usually occures at Day 7-10

A 68 year old female patient presents to your office to discuss hammertoe surgery. She is relatively healthy with well controlled DM and hypertension. Which of the following laboratory tests are not needed on this patient? A. Chemistry Profile B. CBC C. EKG D. Urine Pregnancy Test

Correct answer: (D) Urine Pregnancy Test. Explanation: Laboratory testing is important to order prior to any surgical intervention. Chemistry profiles should be ordered in any patient with hypertension (especially those on diuretics), COPD, renal disease, sleep apnea. Complete blood count should be ordered prior to surgery in patients with history of fatigue (check hemoglobin and hematocrit), liver disease or coagulopathy (platelet count). EKG as a whole is recommended in women over the age of 50 and men over the age of 40 or in patients with heart disease. Urine pregnancy tests do not have to be ordered routinely if the patient is not of child bearing age.

This is a prescription brace for clubfoot. It connects at leg region, allows for growth in length, can be used for rotational deformities. A. Dennis Brown Bar brace B. Bebax shoe C. Mitchell/Ponseti brace D. Wheaton clubfoot brace

Correct answer: (D) Wheaton clubfoot brace. Explanation: It is a prescription brace, thermoplastic knee AFO, applied to the lower limb with hook and loop closure. The connection is at the leg region and allows for growth in length or rotational deformities. The patient cannot walk with the brace due to knee flexion at 90 degrees, indicated for mild to moderate, flexible clubfoot as initial therapy. It can also be used as maintenance after cast therapy. (See picture below) (A) The Denis Brown bar is a prescription bar for clubfoot, the shoes attach to the bar with screws, the screws or shoe clips should be set at 70 degrees abducted to follow the treatment after serial casting for severe clubfoot. It should be worn for 18-23 hours after casting. (B) It is a prescription brace, non-weight-bearing. It applies continuous corrective force. The angles are set by a prescribing physician. It has a plantar bar that can be adjusted up to three planes. The brace can be adjusted during treatment as needed using an Allen wrench, provided with shoes. This brace can be used for moderate and flexible to semi-flexible deformity or as a maintenance therapy, with little tibial deformity. (C) Developed by Dr. Ponseti and John Mitchell, the shoes attach to the bar with screws that attach to plate into shoe. Prescribed after serial casting cases, it should be worn for 23 hours, daily for 3 months, then while sleeping until 3.5 to 4 years old. The length of the bar is similar to the Denis Brown Bar. The bar should be between the heels. The shoe is set at 60 degrees abduction. The shoe is also in 15 degrees dorsiflexion when attached to the bar.

Which of the following is a deformity affecting the forefoot, mostly the 4th metatarsal? It is usually congenital and characterized simply by a shortening of one or more metatarsals due to a premature closing of the epiphyseal line at the distal end of the metatarsal. A. Syndactyly B. Metatarsal cavus C. Digitus adductus D. Digitus abductus E. Brachymetatarsia

Correct answer: (E) Brachymetatarsia. Explanation: Brachymetatarsia was first described by Kite in 1964 simply as a shortening of one or more of the metatarsals due to "... a premature fusion of the epiphyseal line at the distal end of the metatarsal". The true etiology, however, is unknown, with associations being made to traumatic and iatrogenic factors as well as being idiopathic. It has also been associated with such systemic disorders as pseudohypoparathyroidism, Turner's syndrome, Albright's hereditary osteodystrophy, and Down's syndrome. Most often, it is found in the fourth metatarsal and affects females and males at a 25:1 ratio. The deformity, which is congenital, becomes evident between the ages of 4 and 15 years of age. Because of the shortened metatarsal, the digit may dorsally displace over one of the adjacent metatarsals resulting in excess pressure and tyloma formation. Most patients come in with complaints of pain and/or cosmetic problems. Syndactyly (Syndactylism) is a congenital or acquired anomaly that is described as a webbing between adjacent digits. Polydactyly is a common congenital deformity defined as a developmental anomaly characterized by the presence of a supernumerary digit and/or metatarsal found on the hands or feet. Polydactyly may be fully developed, with total duplication of soft tissue and osseous structures. Metatarsal cavus is a metatarsal that is merely plantar declinated as a result of plantar buckling of the metatarsophalangeal joint caused by retrograde force from a contracted and deformed toe. It is an excess plantar declination of the metatarsals with the apex of the deformity at the Lisfranc's. If the deformity is at the apex, it is known as forefoot cavus. Digitus adductus is seen on the transverse plane and represents adduction of the toe at either the interphalangeal joint or metatarsal phalangeal joint. With the passage of time, sagittal plane deformity may also develop. It could be congenital with the middle phalanx less frequently affected. It could be acquired related to the metatarsophalangeal joint defragment or to iatrogenic or traumatic causes. Digitus abductus is abduction of the digit either at the interphalangeal or at the metatarsophalangeal joint. It is a transverse plane deformity that represents congenital deformity when it occurs at the interphalangeal joint.

During fracture fixation, which of the following fixation is providing compression and is also osteogenic? A. Steinman crossed fixation B. K-wire cross fixation C. Buttress plate D. DCP plate E. Lag screws

Correct answer: (E) Lag screws. Explanation: The lag screw involves a unique method of placement that results in approximation and subsequent compression of two osseous segments. Interfragmentary compression generates greater amounts of friction and thus reduces or prevents movement between segments. Lack of motion at the osseous junction, not compression, is osteogenic. Successful application of the lag screw technique can be achieved with either a fully threaded screw or a partially threaded screw.

What will lead to gapping when using a plate during fixation? A. Plate place on the tension side B. Plate place on the convex side of the long bone C. A plate that can withstand tensile forces D. An intact buttress of the opposite cortex E. Plate place on the concave side of the fracture

Correct answer: (E) Plate place on the concave side of the fracture Explanation: Plates ideally should be applied to the tension side in order to function as tension band plates. The plate must be applied to the tension side. If the plate is deficient under load, it will be subject to repetitive bending, will suffer fatigue and break. If a plate is placed on the convex side of the deformity and is placed under tension, it will function as a tension band. As the plate is progressively placed under greater and greater tension, the deformity will be gradually corrected until both the plate and the underlying segment of bone are perfectly straight. Such correction requires the use of a tension device.

Which of the following is true regarding intubation of a patient for general anesthesia? A. The sniffing position is when you extend the head and flex the neck B. It is common to suffer laceration of larynx during intubation C. After intubation, if no breath sounds bilaterally, you most likely intubated into the right main stem bronchi D. If you are going to use general anesthesia and intubate a patient, there is no need to keep the patient NPO for elective surgery

Correct: A Explanation: The best position to place the persons head and neck into for anesthesia is the sniffing position which is where you extend the head and flex the neck. This allows for alignment and is easiest to position the laryngoscope in this position. Intubation is generally very safe. Most patients only experience a mild sore throat which lasts for a few days. It is rare to suffer from croup, laceration and pneumothorax. After you intubate a patient, it is always important to listen for breath sounds. If you hear no breath sounds, you most likely are in the esophagus. If you hear breath sounds only on the right but not on the left, you are most likely in the right main stem bronchus. It is also important to remember that although you are intubating a patient which aids in decreasing the chance of aspiration of gastric contents when compared to IV sedation only, you should always be sure that the patient is NPO before any elective surgical case.

A solitary unilateral external fixator would be an appropriate fixation option for all the following except? A. Multiplanar equino varus deformity B. Pilon fracture of the ankle C. Open comminuted first metatarsal fracture D. Callus distraction for a congenitally short metatarsal

Correct: A Explanation: The foundation of this question lies in the recognition that only answer A requires fixation in multiple planes. Fixation of the pilon fracture requires purely axial fixation and furthermore the unilateral external fixture does not further violate the already tenuous soft tissue envelope of the injury. In the open comminuted first metatarsal fracture the unilateral external fixture can stabilize the fragments, avoid introduction of fixation in the contaminated injury site and allow access for subsequent wound care. Lastly, the unilateral external fixture is for all practical purposes the only way to perform callus distraction on a metatarsal. It allows the gradual distraction of the bony segments which is the premise of the procedure.

Which of the following instruments is not a periosteal elevator? A. Freer B. Metzenbaum C. Sayre D. Key

Correct: B Explanation: Periosteal elevators are used to separate periosteum from underlying bone and, occasionally, to provide leverage for retracting deep fascia, ligaments, and periosteum. These instruments have smooth, broad blades that vary in sharpness. The Metzenbaum scissors have narrow, slightly curved, blunt-tipped blades that are ideal for subcutaneous dissection and cutting lighter fascial and capsular structures. The Freer elevator is ideal for reflecting periosteum from the shaft of a metatarsal. The Sayre elevator has a short, heavy, rounded blade frequently used to distract the sesamoid apparatus from the first metatarsal head. Key elevators are excellent for rapidly reflecting periosteum from glabrous bony surfaces.

What is the most common cause of contracted lesser digits deformity: A. Extensor substitution B. Flexor substitution C. Flexor stabilization D. Trauma

Correct: C Explanation: Flexor stabilization, the most common cause of digital deformities is due to pronation. The flexor tendons are supinators of the rearfoot. During pronation, they fire earlier and longer in an attempt to stabilize the subtalar and midtarsal joints. Consequently, during the late stance-phase of gait, the FDL gains mechanical advantage over the small interosseous muscles. Additionally, the intrinsic muscles, which are also supinators of the midfoot, may become fatigued in the pronated foot. However, they also may be weak for a variety of reasons, including peripheral neuropathy or tarsal tunnel. Flexor stabilization is the most common cause of digital deformity. It is easily recognized by excessive gripping of the toes in stance with hammering or clawing of the lesser digits with adductovarus rotation of the fifth and often four digits.

The surgeon is performing a subtalar joint arthroscopy. Upon initial examination of the joint, immediate visualization of the anterior and middle facets is appreciated. Which of the following statements must be true? A. A non-invasive joint distractor was utilized B. Excessive insufflation of the subtalar joint has occurred C. Hoke's tonsil has been removed D. A defect is present in the interosseous talocalcaneal ligament

Correct: D Explanation: This question challenges the fundamental knowledge of the arthroscopic anatomy of the subtalar joint. The interosseous ligamentous structures completely obscures the view of the middle and anterior subtalar facets. The only way to see the anterior and middle facets is for a defect to be present in the interosseous talocalcaneal ligament. The noninvasive joint distractor would have no contribution in visualization of this aspect of the joint as the intact talocalcaneal ligament would prevent diastasis of the joint. Excessive insufflation of the joint, although it might injure the joint capsule, is not a strong enough force to distend the joint. Likewise, Hoke's tonsil is simply a discrete plug of adipose tissue that is easily manipulated with the arthroscopic instrumentation and does not obscure the view of any joint structure.

Which of the muscle strengths listed below demonstrates a muscle that is suitable for transfer? (Select all that apply) A. 2 B. 3 C. 4 D. 5

D - 5,C - 4. Correct answer: (C) (D) 4 and 5. Explanation: A muscle with the strength of 4 is suitable to be transferred. It is important to note that transferred muscles loses one grade in strength with the transfer thereby the transferred muscle would down grade down to a 3 after the transfer. A muscle with the strength of 5 is suitable to be transferred. It is important to note that transferred muscles loses one grade in strength with the transfer thereby the transferred muscle would down grade down to a 4 after the transfer. (A) A muscle with the strength of 2 is not suitable to be transferred. It is important to note that transferred muscles loses one grade in strength with the transfer thereby the transferred muscle would down grade down to a 1 after the transfer and be nonfunctional. (B) A muscle with the strength of 3 is not suitable to be transferred. It is important to note that transferred muscles loses one grade in strength with the transfer thereby the transferred muscle would down grade down to a 2 after the transfer and be nonfunctional.

Which of the following statements regarding the clinical photograph below are true? (Select all that apply) Latency period for this clinical scenario is shorter than that of callus distraction in an evans osteotomy. The bone in this area is relatively less active than the metaphyseal bone. It is easier to form a viable periosteal sleeve in the location around this osteotomy. An osteotomy in the proximal metaphysis would be more stable and therefore end in a more stable end result.

D - An osteotomy in the proximal metaphysis would be more stable and therefore end in a more stable end result.,C - It is easier to form a viable periosteal sleeve in the location around this osteotomy.,B - The bone in this area is relatively less active than the metaphyseal bone.. Correct answer: (B) (C) (D) Explanation: Due to the high proportion of cancellous bone in the calcaneus relative to that of the metatarsal, the latency period in the evans osteotomy is shorter than that of a metatarsal osteotomy. The osteotomy pictured above is in the diaphysis which is biologically less active than the metaphyseal bone. Dissection in the diaphyseal region is easier and the periosteum is thicker. This allows for easy dissection and creation of a periosteal sleeve. Additionally the mechanical strength of a structure varies with the square of the diameter when subjected to bending loads. Therefore a wider segment of bone (metaphysis) will tend to be more stable throughout lengthening and would lead to a more stable end result.

Which pediatric disorders are associated with a steppage gait? (Select all that apply) A. Cerebral palsy B. Duchenne's muscular dystrophy C. Polio D. Guillain-Barre syndrome

D - Guillain-Barre syndrome,C - Polio. Correct answer: (C) (D) Polio and Guillain-Barre syndrome. Explanation: Polio and post-polio syndrome patients can have an associated steppage gait. A steppage gait is one which the entire leg is lifted by the hip flexors to assist with ground clearance. Guillain-Barre syndrome can have an associated steppage gait. A steppage gait is one which the entire leg is lifted by the hip flexors to assist with ground clearance. (A) Cerebral palsy is not associated with a steppage gait, but rather a Hemiparetic gait. (B) Duchenne's muscular dystrophy is not associated with a steppage gait, but rather a Trendelenburg gait.

The talectomy: (Select all that apply) A. Is contraindicated in the pediatric population B. Is designed to allow patients to become "brace-free" after surgery C. Can be followed up with the Blair procedure to achieve a plantigrade foot D. Is an option for patients considering a BKA

D - Is an option for patients considering a BKA,C - Can be followed up with the Blair procedure to achieve a plantigrade foot. Correct answer: (C) (D) Can be followed up with the Blair procedure to achieve a plantigrade foot and is an option for patients considering a BKA. Explanation: The earliest indications for the talectomy procedure were in the pediatric population; specifically clubfoot, tumors, arthrogryposis multiplex congenita, myelomeningocele, and TB. Adult indications include trauma, failed ankle implant, neuroarthropathy, compartment syndrome, and sciatic nerve palsy, to name a few. Cooper and Capello followed 26 talectomies an average of twenty years post-talectomy; 24/26 were considered to have a satisfactory result. The average age of the patient at the time of surgery was 10.25 years. They did not come out with a specific age for talectomy. Regardless of the type of deformity before surgery, post-operatively the goal of a painless, plantigrade, and stable foot was achieved. They felt a talectomy was indicated in children in whom a triple arthreodesis was contraindicated due to their young age. Although major rearfoot fusions or tendon transfers have as a goal to have their patient be free from bracing, the deformities requiring a talectomy are so severe that reasonable goals post-talectomy include a plantigrade foot, pain-free, and amenable to bracing. Approximately 2-3cm of shortening can be expected, so some type of device would probably be necessary. Blair described the tibiocalcaneal fusion in 1943. Besides talectomy, pantalar or TTC fusion, in addition to the Blair fusion, are all options to a BKA.

What procedures are indicated for a rigid plantar flexed first ray? (Select all that apply) A. Dorsiflexory wedge osteotomy of the first metatarsal B. Cotton procedure (medial cuneiform opening wedge osteotomy) C. Fowler procedure (metatarsal head resection) D. Lapidus procedure (first tarsometatarsal arthrodesis)

D - Lapidus procedure (first tarsometatarsal arthrodesis),A - Dorsiflexory wedge osteotomy of the first metatarsal. Correct answer: (A) (D) Dorsiflexory wedge osteotomy of the first metatarsal and lapidus procedure. Explanation: A dorsiflexory wedge osteotomy of the first metatarsal is indicated in the case of a rigidly plantar flexed first metatarsal. This is typically fixated with a 3.5 mm cortical screw or a figure eight cerclage wire construct. A Lapidus procedure is indicated in the case of a rigidly plantar flexed first metatarsal. (B) The cotton procedure (medial cuneiform opening wedge osteotomy with the wedge based dorsally to plantarflex the medial column) is used to plantarflex the medial column of the foot and would be contraindicated in the rigid plantar flexed first ray. (C) The Fowler procedure (metatarsal head resection) is indicated in part of a metatarsus adductus deformity and not that is a plantarflexed first ray.

A metatarsal bar and a metatarsal pad both reduce pressure off the metatarsal heads. What is another modification that can reduce pressure off the metatarsal heads? (Select all that apply) A. Minimal arch fill B. Flexible orthotic shell C. Reverse Morton's extension D. Leaving anterior shell edge full thickness

D - Leaving anterior shell edge full thickness,A - Minimal arch fill. Correct answer: (A) (D) Minimal arch fill and leaving anterior shell edge full thickness. Explanation: Minimal arch fill on the positive will create a very conforming total contact orthotic. This will allow more pressure to be borne on the arch and the metatarsal heads will bear less weight for less time. Leaving the anterior edge of the orthotic full thickness, as opposed to beveling it, will produce a rocker effect. This will shorten the amount of time that the metatarsal heads bear weight. (B) A flexible orthotic shell would actually increase pressure on the metatarsal heads. To effectively offload the metatarsal head region a semi-rigid or rigid device is recommended. (C) A reverse Morton's extension would increase pressure on the metatarsal heads as it builds up the ground reactive force to metatarsal heads two though five.

Osteochondral grafting for the first MTPJ: A. Is best suited for large diffuse lesions of the metatarsal head. B. Results in fibrocartilage resurfacing. C. Will have a better success rate utilizing the talus, as opposed to the knee. D. Should utilize an MRI in pre-operative planning prior to the procedure.

D - Should utilize an MRI in pre-operative planning prior to the procedure.,C - Will have a better success rate utilizing the talus, as opposed to the knee.. Correct answer: (C) (D) Will have a better success rate utilizing the talus, as opposed to the knee and should utilize an MRI in pre-operative planning prior to the procedure. Explanation: Osteochondral grafting is best suited for the patient with a focal central lesion, confirmed with an MRI before surgery. Diffuse cartilage degeneration is a contraindication for osteochondral grafting. Following successful incorporation of the graft, hyaline cartilage is the result, as opposed to subchondral drilling techniques, which produce fibrocartilage which is less stable than hyaline cartilage. Autografting is best because of the number of chondrocytes present within the graft. The talus is better than the knee for procuring the graft. Knee grafts have thicker cartilage and may be inferior to the talus biomechanically. Also, obtaining the graft from the plantar/medial talar head may more closely resemble the radius of curvature of the first metahead.

A patient with a history of uncontrolled and undertreated hyperthyroidism had a complex bunionectomy postponed until the patient is stable and under anti-hyperthyroidism medications, as proceeding with surgery could lead to stress and thyroid storm. Which of the following are true concerning a thyroid storm? (Select all that apply) Can occur in the intraoperative period Occur in the first 48 hours postoperatively Diagnosis is impossible without laboratory results The mortality of thyroid storm is 10% to 75% and the patient must be monitored in a critical care environment

D - The mortality of thyroid storm is 10% to 75% and the patient must be monitored in a critical care environment,B - Occur in the first 48 hours postoperatively,A - Can occur in the intraoperative period. Correct answer: (A) (B) (D) Explanation: The most feared perioperative complication that usually arises from undiagnosed and undertreated hyperthyroidism is thyroid storm. Thyroid storm can occur at any time in the perioperative period, but it typically occurs either intraoperatively or in the first 48 hours postoperatively. The mortality of thyroid storm is 10% to 75% and the patient must be monitored in a critical care environment. Symptoms of the thyroid storm are nonspecific and include hyperexia (up to 41.1 degrees Celsius, or 106 degrees F), tachycardia, and delirium. Treatment of thyroid storm includes thionamides, beta blockers, and antipyretics. Acetaminophen is preferred over salicylates as the latter may exacerbate thyrotoxicosis by decreasing thyroid protein binding and increasing T3 and T4. The most common cause of thyroid storm in the perioperative period is sepsis.

A 75 year old female with a history of atrial fibrillation, hypertension, congestive heart failure, and stroke is scheduled to undergo a complex bunionectomy with hemi implant to the right foot secondary to long standing hallux limitus and pain. She has no prior VTE or DVT; she is currently on warfarin therapy. What is the best way to manage this patient's anticoagulation perioperatively? (Select all that apply) A. Perioperative bridging therapy B. Stop warfarin 5 days before surgery C. Start lower molecular weight heparin 12 hours after the last dose of warfarin D. Start lower molecular weight heparin 36 hours after last warfarin dose E. Administer last dose of lower molecular weight heparin 24 hours post procedure

E - Administer last dose of lower molecular weight heparin 24 hours post procedure,D - Start lower molecular weight heparin 36 hours after last warfarin dose,B - Stop warfarin 5 days before surgery,A - Perioperative bridging therapy. Correct answer: (A) (B) (D) (E) Explanation: This patient has a CHADS-2 score of 5 indicating a high annual risk of stroke (1 point for each for congestive heart failure, hypertension, age of 75 or greater, and Diabetes mellitus, 2 points for a history of stroke or transient ischemic attack (TIA). This case justifies a bridging therapy. So preoperatively, ensure that the patient does not have any contraindications to LMWH such as allergy to LMWH, history of heparin induced thrombocytopenia, extremes of weight, creatinine clearance of < 15 m/min, history of non-compliance, inability to administer injections. The bridging instructions should be as follows: stop warfarin 5 days before surgery if INR is 2-3, stop warfarin 6 days before surgery if INR is 3-4.5, start LMWH 36 hours after the last dose of warfarin, check INR the morning of surgery to ensure a INR < 1.5 and in some cases <1.2. Then restart the LMWH approximately 24 hours post procedure or consider thromboprophylaxis dosing of LMWH on post op day # 1. If patient is at high risk for bleeding restart warfarin at patient's usual dose on the surgical day, check INR daily until patient is discharged and periodically thereafter until INR is therapeutic, check CBC on post-op days 3 and 7 to monitor platelets, discontinue LMWH when INR is therapeutic for two consecutive days.


Related study sets

Criminal Justice Chapter 3 and 4.

View Set

5. Making a Radiographic Image: Projection Geometry & Characteristics of Diagnostic Radiograph Localization

View Set

Prep U Psychiatric-Mental Health Nursing Chapter 11: Anger, Hostility, and Aggression

View Set